Download as pdf or txt
Download as pdf or txt
You are on page 1of 185

iSummary SHORT NOTES

iSummary SHORT NOTES 1st edition

All copyrights reserved 2011

Egyptian fellowship of the Family Medicine

Terms of use
Dr Shaimaa :‫هذه اننسخة ممهىكة ل‬
‫وال جيىس نسخها نالستخداو انتجاري وال جيىس استخدامها من أي شخص غري مانك هذه‬
‫اننسخة وال جيىس رفعها عهى مىاقع االنتزنت من غري تصزيح من مؤنف انكتاب ومن خيانف هذه‬
‫انشزوط فقد خانف األمانة و يتحمم انسؤال أماو اهلل عش وجم يىو انقيامة‬

Preface
iSummary SHORT NOTES is a quick review of the family
medicine subjects

Features
Quick tables of all the diseases
Covers all important points in all topics
Covers all the family medicine branches

Intended audience
Primary care trainees and practicing physicians will find this
book a useful resource for common conditions seen in
ambulatory practice.
Candidates who are preparing to AKT exam of the Egyptian
fellowship of the Family medicine

Dedication
To my wonderful son Yasin who was bothering me during
writing of this book

Egypt, Cairo Nov 2011

1
iSummary SHORT NOTES

INDEX

N.B. Number 1 before any medication or procedure means the first choice, number 2 means
the second choice

2
iSummary SHORT NOTES

CARDIOLOGY
Disease Management
Dyslipidemia ●Psyllium hydrophilic mucilloid  mildly elevated LDL (130–159
mg/dL) with HDL ≥45 mg/dL
●Niacin moderately elevated LDL and either low HDL cholesterol
(≤35 mg/dL) or high TGs.   TG, ↓ LDL , HDL
●Bile Acid Binding Resins
HDL , ↓ LDL
●Fibrates   TG,  HDL
●Omega-3 fatty acids  TG
●Statins
 moderately or severely elevated LDL
cholesterol↓ LDL,  TG , moderately  HDL
●Ezetimibe ↓ LDL,  TG , moderately  HDL
Prinzmetal angina ●CCB drug of choice, Nitrates are also effective.
Stable angina ● 1.ß blockers ● CCB ● Nitrates ● Antiplatelet Agents (aspirin-
Clopidrogrel)
Systolic Heart ●Stage Arisk factors management (HTN, DM,..)
Failure ●Stage B ACEIs and β-blockersin recent or remote history of
MI
●Stage C ●loop diuretic , ACEI (or ARB if intolerant), β-
blocker ● Aldosterone antagonist
● Digoxin ● Hydralazine plus isosorbide dinitrate
● Implantable cardioverter-defibrillators (ICDs)
● Biventricular pacing
●Stage D● Control of fluid retention ●cardiac transplantation
Diastolic Heart ● 1.β-blockers ●verapamil, diltiazem (next best option) ●ACEIs
Failure ●diuretics
Hypertrophic ● 1.β-blockers ●CCB are sometimes useful
cardiomyopathy
Atrial fibrillation ● Cardioversion unstable patient, Anticoagulate for at least four
(AF) weeks postcardioversion
●Anticoagulationsee below
● Rate control β-blockers or diltiazem, verapamil. In
hemodynamically unstable patient with a ↓ EFdigoxin or
amiodarone.
● Rhythm controlFlecainide, propafenone, amiodarone, sotalol,
ibutilide.
Atrial flutter ●Lone atrial flutter (without structural heart disease)consider
anticoagulation.
●Unstable patients Cardioversion
●Stable patients ●Watchful waiting is an option ●anticoagulation
approach is similar to that of AF. ●Antiarrhythmics: Ibutilide,
flecainide, propafenone.●Overdrive pacing: A temporary pacemaker
●Cardioversion: Management of anticoagulation around
cardioversion similar to that of AF.
●Rate control with CCB, blockers, β-blockers, or digoxin.
●Long-term control Radiofrequency ablation, antiarrhythmics or

3
iSummary SHORT NOTES

pacemakers.
Supraventricular ●Stable Valsalva; carotid sinus massage
tachycardia (SVT) ● Adenosine may be given next if these maneuvers are unsuccessful.
● Verapamil, diltiazem, ß-blocker
●unstableCardioversion
Rheumatic fever ● 1.Aspirin
● Single IM benzathine penicillin or oral phenoxymethyl penicillin
for a 10-day course whether or not pharyngitis is present
Ventricular septal ●Small shunts do not require closure in asymptomatic patients
defect (VSD) ●Symptomatic children hypercaloric feeds, diuretics, and ACEIs.
●If symptoms persist despite maximal medical therapy or if there is
↑ pulmonary vascular resistance (affects approximately 50% of
patients)defect should be surgically or percutaneously repaired.
Surgery is contraindicated in Eisenmenger's syndrome.
●Influenza vaccine, RSV prophylaxis.
●Endocarditis occurs more often with smaller shunts; antibiotic
prophylaxis is mandatory for all patients.

Disease Diagnosis Investigation Management


Aortic ●Exertional ● ● Asymptomaticclose clinical
stenosis dyspnea, ↓ Echocardiography follow-up
exercise ● endocarditis antibiotic
tolerance, angina prophylaxis
pectoris, ● symptomatic Aortic valve
syncope, replacement
congestive heart ● Percutaneous balloon aortic
failure, sudden valvuloplastychildren, young
death adults
● Systolic
murmur
Mitral ●Asymptomatic ● TTE ● Diuretics mild symptoms
stenosis may be for 16 (transthoracic ● endocarditis antibiotic
years echocardiogram) prophylaxis
●Exertional ● TEE ● Percutaneous mitral balloon
dyspnea (most (transesophageal valvotomy symptomatic,
common and echoexclude left isolated MS, valve orifice is <1.7
often only atrial thrombus in cm
symptom) patients scheduled ● Surgical (“open”) valvotomy
● Diastolic for valvotomy ● Mitral valve replacement
murmur MS+ MR
Aortic ● Dyspnea on ● TTE ● Asymptomatic regular
regurgitation exertion, fatigue, clinical evaluation, assessment
(AR) and decreased of left ventricular function,
exercise ●Endocarditis antibiotic
tolerance prophylaxis.
● Blowing ●Symptomatic ACEIs, Long-
diastolic murmur acting nifedipine
● Widened pulse ● Surgical correction  before
pressure the onset of permanent left
ventricular damage, even in

4
iSummary SHORT NOTES

asymptomatic patients, patients


who remain symptomatic
despite optimal medical therapy
● Aortic valve replacement
progressive left ventricular
dysfunction and a left
ventricular ejection fraction
<55% or left ventricular end-
systolic volume >55% (even if
asymptomatic)
Mitral ● Chronic severe ● TTE detect and ● Asymptomatic regular
regurgitation MR fatigue, grade the severity. clinical evaluation, assessment
(MR) exertional ●TEEprior to of left ventricular function,
dyspea, and surgical repair or ●Endocarditis antibiotic
orthopnea replacement. prophylaxis.
● Acute severe ● Nitroprusside acute severe
MRLV failure MR
with acute ● Surgery  patient is
pulmonary symptomatic despite optimum
edema is medical management, LV
common dysfunction is progressive, with
(cardiogenic LV EV <60% (even if
shock), asymptomatic), AF, pulmonary
hypotension, and hypertension.
poor tissue
perfusion.
● Holosystolic
murmur
Mitral valve ● Asymptomatic ● ● Aspirinhistory of TIA or
prolapse ● Atypical chest Echocardiography lone AF.
(MVP) pain, palpitations, ●Warfarin: chronic AF, a
exaggerated previous thromboembolic event,
autonomic CVA, MR, or heart failure (target
symptoms or TIAs INR 2–3).
● Midsystolic ●No treatment if none of
click and a these complications are present.
murmur at the ●Beta-blockers chest pain.
apex
Tricuspid ● Dyspnea, right ● Diuretics and digitalis  treat
valve disease and left sided the associated right-sided
failure failure.
● Systolic ●ACE inhibitors LV
murmur dysfunction
● Tricuspid valve annuloplasty
● Tricuspid valve replacement
abnormal tricuspid valves not
amenable to annuloplasty or
tricuspid valve repair
Tricuspid ● Dyspnea with ● Percutaneous valvular
valve activity commissurotomy
stenosis ●  in the jugular ● Open commissurotomy

5
iSummary SHORT NOTES

vein with a large ● Valve replacement


a wave Bioprosthetic valves
Pericarditis ● Sharp constant ●Treat the cause (if possible)
pleuritic chest ●Symptomatic treatment
pain that  when NSAID for pain; steroids in
the patient is resistant cases. Colchicine can
supine, also be used.
respiration, and
↓ when the
patient leans
forward
● Pericardial
friction rub

 Routine tests on all newly diagnosed hypertensive patients

 Unless the history and physical examination suggest otherwise, initial


laboratory testing can be limited to:
 CBC: is helpful to rule out anemia or polycythemia.
 Potassium: to assess for hyperaldosteronism 1
 Creatinine: reflect renal function.
 Fasting glucose.
 Calcium.
 A fasting lipid profile including total cholesterol, HDL cholesterol, LDL
cholesterol and triglycerides.
 Complete Urinalysis: should be assessed for evidence of hematuria,
proteinuria, or casts suggestive of intrinsic renal disease.
 Resting electrocardiogram: assess for conduction abnormalities that
may have implications for antihypertensive medication choices.
 Obtaining a urinary microalbumin/creatinine ratio is optional.
1
Many patients with hyperaldosteronism will have normal serum potassium levels. Additionally, the
plasma aldosterone-to-renin level is usually high.

 JNC 7 Defined Compelling Indications for Use of Specific Anti-


hypertensive Agents in Complicated Hypertension
Indication Diuretic BB ACEI ARB CCB AA
Heart failure ✓ ✓ ✓ ✓ ✓
Post MI ✓ ✓ ✓ ✓
CAD risk ✓ ✓ ✓ ✓
Diabetes mellitus ✓ ✓ ✓ ✓ ✓
Renal disease ✓ ✓
Recurrent stroke prevention ✓ ✓

6
iSummary SHORT NOTES

 Other compiling indications


Condition Recommended drug Avoid
Ischemic heart disease BB, CCB none
Hypercholesterolemia ACEI, α blocker, high dose High dose BB without ISA
BB with ISA High dose diuretics
Pregnancy α methyl dopa Diuretics
Hydralazine ACEI
Asthma CCB BB
PVD CCB or other vasodilators BB
Young patient with BB none
hyperdynamic
circulation
Osteoporosis TZD none
Benign prostatic Α Blocker none
hypertrophy
Type 1DM ACEI BB
Isolated systolic HTN Diuretics, long acting CCB
Atrial arrhythmias diltiazem, verapamil
isolated systolic amlodipine, felodipine,
hypertension in elderly nifedipine

 Desired target of blood pressure

 For individuals with high blood pressure, the goal is a systolic BP consistently
below 140/90 mm Hg.
 The goal is 130/80 mm Hg for individuals with diabetes, chronic kidney
disease, coronary artery disease or coronary artery disease equivalents
(carotid artery disease, peripheral arterial disease, abdominal aortic
aneurysm) or high cardiovascular risk (10-year Framingham risk score ≥10%).
 Chronic kidney disease is defined as glomerular filtration rate (GFR) less than 60 mL/min/1.73
m2, baseline serum creatinine higher than 1.5 mg/dL in men or higher than 1.3 mg/dL in
women, or albuminuria, which is defined as higher than 300 mg/day on a 24-hour urine
specimen or 200 mg of albumin per gram creatine on urine spot check.

 Initial follow-up of hypertension

 Well patients with Stage 1 HTN should have follow-up visits every 1 to 2
months until the BP goal is reached without significant medication side
effects (i.e., side effects that are unacceptable to the patient or the
physician).
 Patients with Stage 2 HTN and/or complicating comorbidities should be seen
every 2 to 4 weeks until the BP is clearly coming under control without
unacceptable side effects.

7
iSummary SHORT NOTES

 Once the BP goals are reached and stable on a given therapeutic regimen,
follow-up can be stretched out to 3: 6 months, unless other conditions
dictate more frequent visits.

 Calcium channel blockers are useful in Raynaud syndrome, atrial fibrillation and
supraventricular tachyarrhythmias.
 ACE inhibitors must be used cautiously in patients with known renovascular
disease and, when used, may need dose adjustment due to reduced drug
clearance. When creatinine elevations exceed 30% above baseline, temporary
cessation or reduction of dose is warranted. (as well as ARBs)
 These agents should be used with extreme caution, if at all, in patients whose
serum creatinine level exceeds 3.0 mg/mL.

 Risk Category Determination and LDL Cholesterol Goals


Risk Category LDL Goal LDL Level at LDL Level at
Which to Begin Which to
Therapeutic Life- Consider Drug
style Changes Treatment
(TLC)
High-risk: <100 mg/dL ≥ 100 mg/dL ≥ 100 mg/dL or <
Coronary heart (<70 mg/dL 100 mg/dL
disease or optimal1)
equivalent3 (10y
risk ≥ 20%)
2+ Risk factors <130 mg/dL ≥ 130 mg/dL ≥130 mg/dL (>100
(10y risk 10% – (<100 mg/dL All patients regardless mg/dL: drug
20%) optimal) of LDL optional 2)
2 Risk factors (10y <130 mg/dL ≥130 mg/dL ≥160 mg/dL
risk ≤ 10%)
0 -1 Risk factor <160 mg/dL ≥ 160 mg/dL ≥190 mg/dL (160 –
189 mg/dL: drug
use optional)
1
The optional goal of <70 mg/dL is particularly targeted at patients at very high risk, including patients with a recent acute
coronary syndrome, and diabetic patients with poorly controlled glycemia and multiple risk factors.
2
When initiating statin therapy in these patients, the goal for LDL reduction should be 30%-40% from baseline.
3
Patients who are in the high-risk category either have CAD or a CAD risk equivalent.

 Metabolic syndrome is defined as the presence of three or more of the


following risk factors:
 Abdominal obesity (waist circumference ≥40 inches in men, ≥35 inches in
women).
 BP ≥130/85 mm Hg.
 Glucose intolerance (FPG ≥100 mg/dL).
 Dyslipidemia (TG ≥150 mg/dL or HDL ≤40 mg/dL in men, or ≤50 mg/dL in
women).

8
iSummary SHORT NOTES

 Dyslipidemia

 The single most important risk factor for CAD is elevated LDL cholesterol,
then a depressed HDL.

 SCREENING
 The most recent NCEP Adult Treatment Panel recommends that all patients
older than the age of 20 years be screened with a fasting lipid profile. This
includes a total cholesterol, HDL, LDL, and a triglyceride level. If these values
are normal, repeat screening should be performed in 5 years.
 The US Preventive Services Task Force (USPSTF) bases its screening
recommendations on the age of the patient.
 It strongly recommends (rating: A) routinely screening men 35 years of
age and older and women 45 years and older for lipid disorders.
 The USPSTF recommends (rating: B) screening younger adults (men 20–
35 years of age and women 20–45 years of age) if they have other risk
factors for CAD (tobacco use, diabetes, a family history of heart disease
or high cholesterol, or high blood pressure). They make no
recommendation for or against screening in younger adults in the
absence of known risk factors.

 MANAGEMENT
 Factors  HDL Exercise, weight loss, smoking cessation
 Gemfibrozil significantly reduces the glucuronidation of statins, which
decreases their elimination. This increases the risk of myopathy or
rhabdomyolysis and hepatotoxicity. When used in combination with
gemfibrozil, the doses for simvastatin and rosuvastatin should not exceed
10 mg daily.
 Statins are the most effective drugs for reducing serum levels of LDL;
ezetimibe is the next most effective.
 Fibrates have the greatest capacity to reduce serum triglycerides.
 Niacin raises serum levels of HDL significantly better than other
antilipidemic medications

 STATIN
 Pravastatin or fluvastatin may be least likely to cause side effects
 Perceived side effects from one statin can often be avoided by
switching to a different statin
 Liver toxicity can occur and is defined as an ALT elevation three times
the upper limit of normal (ULN) on two occasions at least 1 month
apart.
 Mild elevations in serum transaminase levels early during the course of
therapy are relatively common and usually resolve spontaneously. If
hepatotoxicity develops, statin therapy should be discontinued until
transaminase levels normalize and therapy with a different statin can

9
iSummary SHORT NOTES

be initiated. There is no documented evidence that the statins increase


the risk of liver failure.
 It is prudent to obtain a baseline ALT level and to recheck ALT 6 weeks,
12 weeks, 6 months, and annually after initiating treatment or after
increasing the statin dose.
 Warn patients and check CPK if muscle soreness occurs. The diagnosis
of myopathy is made when CK levels exceed 10 times ULN.
 Because of their relatively short half-lives (1 to 4 hours), lovastatin,
fluvastatin, pravastatin, and simvastatin should be taken after the
evening meal in order to intercept the peak activity of HMG-CoA-
reductase that occurs around midnight. Rosuvastatin and atorvastatin
can be taken at any time during the day or night because of their long
half-lives (approximately 19 hours and 14 hours, respectively).

 Ischemic heart disease

 CARDIAC ENZYMES
 The diagnosis of MI in a patient who had chest pain later than 48 hours
after presentation is best made with TI,
 whereas a recent-onset MI lasting less than 6 hours can best be made
with myoglobin, CK, and CK-MB.
 A negative TI after 12 hours of chest pain indicates a low chance for a
cardiac event in the near future (within 1 month).
 Abnormal cardiac enzymes allow a definite diagnosis of MI.

 A TREADMILL STRESS TEST can be a first-choice test to diagnose CAD by the


family physician. Women are more likely to have false-positive treadmill ECG
testing results than men and may be better candidates for pharmacologic or
echocardiographic stress tests.

 CORONARY ANGIOGRAPHY remains the gold standard in evaluation of ACS,


Most important indications Uncertain diagnosis of angina (coronary artery
disease cannot be excluded by noninvasive testing), Unstable angina, Acute
myocardial infarction, Angina after bypass surgery or percutaneous
intervention
 FIBRINOLYTICS are contraindicated in patients with unstable angina or non–
ST segment elevation myocardial infarction

 STABLE ANGINA
 Calcium channel blockers may be preferred in patients who cannot
tolerate beta blockers or have contraindications to their use. Short-
acting dihydropyridine calcium channel blockers (eg, immediate-release
nifedipine) should generally be avoided when treating angina,
hypertension, and other cardiovascular disease. The effects on blood
pressure are unpredictable, and mortality is greater. Use long-acting
preparations, such as amlodipine and extended-release nifedipine.

11
iSummary SHORT NOTES

 When prescribing a patch, it is important to have patch-free intervals of


10 to 12 hours (Irrespective of the mode of administration) to retain the
antianginal effect. Patients should be warned about the potential for
severe hypotension when sildenafil is taken within 24 hours after any
nitrate preparation has been taken.
 Unless the patient also has congestive HF, ACEIs are generally not
used as medical therapy for angina
 Clopidrogel is an effective alternative for patients who are unable to
take aspirin, It is also used with aspirin post PCI or UA/NSTEMI (for 1-12
months post PCI with bare metal stent or post UA/NSTEMI without
stenting)

 CARDIAC REHABILITATION
 Patient who has suffered an MI should increase activity level gradually
over a period of 6:8 weeks, patient can return to work by
approximately 8 weeks.
 Patient who have suffered uncomplicated MI may be safely started in
activity program by 3:4 weeks post-infractions
 Sexual intercourse can be resumed within 4:6 weeks of the infarction.
 Patients with uncomplicated MI or those who have undergone
uncomplicated PCI should not fly until at least 2 to 3 weeks have passed
and they are tolerating their usual daily activities.

 Heart failure

 Clinically, diastolic heart failure is as prevalent as LVSD, and the


presentation of clinically evident diastolic heart failure is indistinguishable
from clinically apparent LVSD. Elderly women, usually with a heavy
prevalence of hypertension, diabetes mellitus and obesity appear to be
most at risk
 The most valuable and cost-effective test in the diagnosis of HF is
echocardiography with Doppler imaging. The most important characteristic
found on echo is the EF which is usually < 40% in systolic HF.
 Diastolic dysfunction: EF is normal, but hypertrophy, RV enlargement,
pericarditis, and infiltrative disease may be seen
 Coronary angiography is recommended for patients with new-onset heart
failure of uncertain etiology, despite the absence of anginal symptoms or
negative findings on exercise stress testing

 Management
 Aldosterone antagonist (spironolactone and eplerenone ) For worsening
symptoms and survival in moderately severe to severe heart failure (NYHA
class III with decompensations)
 Digoxin reduces hospitalizations in patients with uncontrolled
symptomatic heart failure. in cases AF with a rapid ventricular rate
 Hydralazine plus isosorbide dinitrate Effective for persistent symptoms
and survival, particularly in blacks

11
iSummary SHORT NOTES

 Serum potassium needs to be monitored closely after initiation of


spironolactone, especially since it will generally be used with an ACE
inhibitor or ARB, both of which can increase the serum potassium.
potassium is maintained below 5.0 mEq/L. use of drugs (NSAIDs),
cyclooxygenase-2 (COX-2) inhibitors, and potassium supplements should be
avoided if possible
 Digoxin toxicity is manifested by cardiac arrhythmias, nausea, vomiting, and
yellow-green halos around visual images or lights. In severe cases of toxicity,
the drug should be withheld and digoxin-specific antibody fragments
(Digibind) can be given. Digoxin (Digibind) binds the digoxin and allows rapid
excretion
 Warfarin therapy is only indicated in heart failure patients with a history of
a thromboembolic event or those with paroxysmal or chronic atrial
fibrillation or flutter
 Aspirin can decrease ACEI effectiveness and potentially increase
hospitalizations from heart failure decompensation
 Therapies that adversely affect the clinical status of patients with
symptomatic heart failure should be avoided calcium channel blockers,
NSAIDs, antiarrhythmic drugs (except amiodarone and dofetilide),
Phosphodiesterase inhibitors (cilostazol, sildenafil, vardenafil, and
tadalafil), Thiazolidinediones and metformin
 Moderate dietary sodium restriction (3–4 g daily) with daily weight
measurement further enhance volume control and allow for lower and safer
doses of diuretic therapies
 Fluid restriction is necessary only when excessive or when volume status is
difficult to manage with diuretics and sodium restriction
 Exercise prescriptions (aerobic and light resistance training) are generally
safe in compensated HF.

 Valvular heart diseases

 Generally, patients with stenotic valvular lesions can be monitored


clinically until symptoms appear. On the other hand, patients with
regurgitant valvular lesions require careful echocardiographic monitoring
for left ventricular function and may require surgery even in the absence of
symptoms
 The most common cause of aortic stenosis (AS) is idiopathic. Individuals
born with a bicuspid valve are at most risk for developing AS. Symptoms do
not usually develop until the area is < 1 cm2 (the normal valve area is 3–4
cm2), jet velocity is > 4 m/sec, and the pressure gradient is > 40 mmHg.
 Patients with mild aortic stenosis who are asymptomatic can be followed by
echocardiogram every 3–5 years. Patients with severe disease should have
yearly echocardiography to evaluate for left ventricular dysfunction
 Hypertrophic cardiomyopathy  the most common cardiac abnormality
found in young athletes who die suddenly during vigorous physical activity
 Complication of MS Left atrial enlargement; AF → left atrial thrombus
formation and risk of embolism.

12
iSummary SHORT NOTES

 An annual history and physical examination, as well as a chest x-ray and


ECG, are recommended in asymptomatic patients. Endocarditis prophylaxis
in indicated in patients with MS; however, no further medical therapy is
indicated
 MR Control any concurrent dysrhythmia; often this is AF. Monitor LV
function with an annual echocardiogram.
 MVPIf the condition is symptomatic or associated with MR,
echocardiography should be repeated every year; otherwise, it may be
repeated every 2–3 years.
 According to AHA 2007 mitral valve prolapse is no longer considered high
risk and patients do not antibiotic prophylaxis.
 Once patients with MR and AR become symptomatic, the morbidity and
mortality increase significantly. Thus, these patients should be operated on
before they become symptomatic. Patients should have routine
echocardiography yearly if they have severe disease.
 Bioprosthetic valves Give 3 months of warfarin after placement; then
consider aspirin in high-risk patients.
 Mechanical valves All patients should be anticoagulated in light of the
higher risk of thrombus formation. The target INR depends on the type and
location of the valve.

 Atrial Fibrillation

 To determine the need for anticoagulation, estimate stroke risk with the
CHADS2 score:
 CHF
 Hypertension
 Age > 75 years
 Diabetes
 Stroke/TIA (two points each)
 Score 0Aspirin 325 mg daily.
 Score 1–2Indeterminate; based on the individual patient.
 Score 3+Warfarin; target INR of 2.5 (range 2–3) in most patients. Target
INR is 3.0 (2.5–3.5) with valvular disease, previous thromboembolism, or
mechanical valve.
 If warfarin is contraindicated, aspirin therapy is effective.

 Rheumatic fever

 Revised Jones criteria Evidence of previous streptococcal infection


(scarlet fever, +ve throat swab, and/or  ASO titre >200U/ml) and
2 major criteria or 1 major + 2 minor criteria
 Major criteria Carditis (arrhythmia, new murmur, pericardial rub, heart
failure, conduction defects), Migratory polyarthritis (flitting - red, tender
joints, Sydenham's chorea, Subcutaneous nodules, Erythema marginatum

13
iSummary SHORT NOTES

 Minor criteria Prolonged P-R interval on ECG (but not if carditis is one of
the major criteria), Arthralgia (but not if arthritis is one of the major
criteria), Fever,  ESR or  CRP, History of rheumatic heart disease or
rheumatic fever
 Recurrence may occur after further streptococcal infection or be
precipitated by pregnancy or the COC pill
 Prevention Phenoxymethyl Penicillin po or IM injection of benzathine
penicillin monthly until the age of 20 or 5 years after the last attack, if
cardiac involvement, continued until age 25y. Give sulfadiazine or
Erythromycin if patient is allergic to Penicillin.

 Bacterial endocarditis prophylaxis

 High risk patients + High risk procedures give prophylaxis


 High risk patients + low risk procedures (lower respiratory (including
flexible fiberoptic bronchoscopy), genitourinary (including vaginal
hysterectomy and vaginal delivery), or gastrointestinal tract (including
transesophageal echocardiography and endoscopy) give prophylaxis
(optional)

High risk patients High risk procedures


●A previous infection with ●Dental procedures involving dentogingival
endocarditis manipulation or endodonics (tooth extractions,
●A history of cardiac valve periodontal procedures, and cleaning of teeth
replacement surgery (prosthetic with removal of adherent plaque)
heart valve) ●Respiratory tract procedures, Tonsillectomy
●A history of surgically and/or adenoidectomy
constructed systemic-pulmonary Operations involving the respiratory mucosa
shunts or conduits ●Nasal packing, Cosmetic piercing of tongue or
●Complex cyanotic congenital oral mucosa in high-risk patients
heart disease such as single ●GIT procedures, Sclerotherapy for esophageal
ventricle conditions, transposition varices, Esophageal dilatation of stricture
of the great arteries, or tetralogy ●Endoscopic retrograde cholangiography with
of Fallot biliary obstruction, Biliary tract surgery,
Moderate-risk patients Gallstone lithotripsy
●Acquired cardiac valve disease ●Operations involving the intestinal mucosa
●Hypertrophic cardiomyopathies ●Genitourinary tract procedures, Cystoscopy,
●Other congenital abnormalities Urethral dilatation
not mentioned under high-risk ●Urethral catheterization or surgery if urinary
patients, above infection is present
●Mitral valve prolapse with ●Prostate surgery or biopsy
valvular regurgitation and/or
thickened leaflets

14
iSummary SHORT NOTES

Procedure Antibiotic
Dental, Oral, URT, or ●Amoxicillin, 2.0 g PO 1 hour before procedure or
Esophageal ●Ampicillin, 2.0 g IV or IM within 30 minutes before
Procedures procedure
If allergy ↓
●Azithromycin or clarithromycin, 500 mg PO 1 hour
before procedure or clindamycin, 600 mg PO 1 hour
before procedure. Or ●Clindamycin, 600 mg IV within 30
min of starting procedure.
Genitourinary or For high-risk patients
Gastrointestinal ●Ampicillin, 2.0 g IV or IM, plus Gentamicin, 1.5 mg per
Procedures kg IV (not to exceed 120 mg) within 30 minutes of starting
procedure, then Ampicillin, 1.0 g IV or IM 6 hours later
(or amoxicillin, 1 g PO).
If allergy ↓
●Vancomycin, 1.0 g IV (over 1 to 2 hours) completed
within 30 minutes of starting procedure plus gentamicin,
1.5 mg per kg IV or IM (not to exceed 120 mg) within 30
minutes of starting procedure.

For moderate-risk patients (treatment for these patients


is optional for gastrointestinal tract procedures) 
●Amoxicillin, 2.0 g PO 1 hour before procedure, or
●Ampicillin, 2.0 g IV or IM within 30 minutes of starting
procedure.
If allergy ↓
●Vancomycin, 1.0 g IV (over 1 to 2 hours) completed
within 30 minutes of starting procedure

 Congenital heart diseases


Symptoms of pathologic Features of innocent
murmurs murmurs the 8 S’s
●FTT, exercise intolerance, dyspnea ●Soft ●Systolic ●Short ●Sounds (S1 and
with exertion/diaphoresis, syncope, S2) normal ●Symptomless
dizziness, cyanosis, loss of ●Special tests (x-ray, ECG) normal
consciousness, and tachypnea with ●Standing/Sitting (vary with position)
feeds or activity. ●Sternal depression (pectus excavatum)
●Grade III or more.
●Diastolic murmur.
●Abnormal CXR or ECG
●The patient has a syndrome with a
high incidence of heart defects (e.g.,
Down, Turner’s, Marfan’s)

 Refer pathologic murmurs to a pediatric cardiologist; observe benign


murmurs.

15
iSummary SHORT NOTES

 Unless the child is febrile when the murmur is heard and it disappears once
afebrile, refer all children with murmurs for Echo or pediatric evaluation
whether the murmur is detected at routine screening, incidentally when
examining the chest for another reason, or when examined because
symptomatic.

16
iSummary SHORT NOTES

PULMONARY
DISEASES
Disease Management
Bronchial Quick relief
Asthma ●Short-acting inhaled β2-agonists:

●Anticholinergic:
● Systemic corticosteroids:

 Long term control


● Corticosteroids: Inhales fl

Systemic
● Mast cell stabilizers 
● Long-acting ß2-agonists
● Theophylline
● Leukotriene modifiers
COPD  Anticholinergic bronchodilators
●1. Tiotropium bromide ● Ipratropium
Bronchodilator ●LABA: , ●SABA:
 Corticosteroid (controversial)
 Theophylline
 Antibiotic therapy if↑ volume or a purulent appearance to
their sputum, if there are signs of pneumonia, sepsis, or other
bacterial infection
 Supplemental oxygen
 Mucolytic agents with exacerbations, winter months in
patients with moderate or severe COPD who are not on inhaled
steroids
Antitussives dextromethorphan
Pneumonia ●Healthy persons macrolide ( )

↓If resistance
●Fluoroquinolone ( ) or the
combination of a β-lactam plus a macrolide
●Gram-negative organisms (e.g., K. pneumonia, E. coli and H,
influenza) 1. Cephalosporin + azithromycin. 2. Quinolones
alone.
Active First 2 months  4 drugs isoniazid (INH), rifampin (RIF),
tuberculosis pyrazinamide (PZA), and ethambutol (ETH) or streptomycin
then either of the following:
●2:6 months INH and RIF alone (If there is no resistance to

17
iSummary SHORT NOTES

INH and RIF, if repeat sputum cultures are negative and there is
improvement in the patient’s clinical condition)
●2:9 months INH/RIF alone (if the patient remains
symptomatic, or if a follow-up smear or culture result remains
positive after 2 months of therapy)
●Pregnancy and lactation INH, RIF, and ETH for 9 months
with pyridoxine, 25 mg per day
Extrapulmonary ● As above for active tuberculosis but is continued for 9 months
disease ● Miliary TB, bone or joint TB, and TB meningitis in children and
infants  12 months of therapy with public health consultation.
Latent TB ●9 months of INH (daily or twice a week by DOT)
●4 months of RIFas an alternative in contacts of patients
with INH-resistant TB.
●9 months of INH For pregnant, HIV-negative women
Acute bronchitis ●Supportive fluids, rest ●± albuterol inhaler
●±Antibiotichigh risk patient
Bronchiectasis ● Persistent or ● CXR “tram ● Airway clearance
recurrent cough and tracks,” techniques chest
purulent sputum ● High- physical therapy,
production. resolution flutter devices
Hemoptysis CTgood ±Mucolytic agents
sensitivity ● Antibioticsin
exacerbations
amoxicillin, TMP-
SMX, or quinolone
with an
aminoglycoside if
Pseudomonas
aeruginosa is present.
●Bronchodilatorsβ-
agonists and
anticholinergics
●Surgical
resectionsevere
focal disease
Cystic Fibrosis ●FFT, pancreatic ● Sweat ● Acute
(CF) insufficiency, chloride exacerbations:
recurrent concentrations bronchodilators,
pancreatitis, > 70 mEq/L DNase to thin sputum,
sinusitis, intestinal antibiotics (at least two
obstruction, chronic with antipseudomonal
hepatic disease, coverage), and chest
vitamin (fat soluble) physical therapy.
deficiencies, and ● Longer-term
male therapy●aerobic
urogenital/infertility exercise, flutter
problems. devices, and external

18
iSummary SHORT NOTES

bronchiectasis. percussive vests


chronic or recurrent ●pancreatic enzymes
cough, sputum and the fat-soluble (A,
production, D, E, and K) vitamins
dyspnea, and ●Nebulized DNase.
wheezing

 Classification of Severity of Asthma


Mild Mild Moderate Severe
intermittent persistent persistent persistent
Symptoms ≤ twice a Symptoms ≥ twice Daily symptoms; daily Continual
week; otherwise a week, but less use of inhaled short- symptoms; limited
asymptomatic and than once a day. acting beta-2-agonist. physical activity;
normal peak frequent
expiratory flow (PEF) Exacerbation may exacerbations.
between affect activity
exacerbations.
Nocturnal symptoms Nocturnal Nocturnal symptoms Frequent nocturnal
≤ twice a month. symptoms ≥ twice more than once a symptoms.
a month. week.
Forced expiratory FEV1 or PEF FEV1 or PEF greater FEV1 or PEF ≤60% of
volume in 1 second greater than 80% than 60% but less than predicted.
(FEV1) or PEF ≥ 80% of predicted. 80% of predicted.
of predicted. PEF Variability >
30%
FEV1: forced expiratory volume in 1 second. PEFR: peak expiratory flow rate.

 BRONCHIAL ASTHMA

 Zone scheme: Peak flowmeters can also aid in patient self-assessment of


Asthma

 The green zone (PEFR = 80%–100% of the patient’s best score)


continue the usual course of medicine.
 The yellow zone (PEFR = 50%–80%) take additional medicine or call
the physician.
 The red zone (PEFR ≤ 50%) patient should both use the inhaler and
call the physician immediately

 Spirometry The essential criterion for a diagnosis of airway obstruction is


an FEV1/FVC ratio of <70%. An  in FEV1 ≥12% (or increase in FVC >15%)
post bronchodilator therapy suggests reversible airway obstruction/asthma.
FEV1 is the most sensitive in evaluating patients with asthma
 In children a chest x-ray is appropriate in the initial evaluation because of
the possibility of congenital abnormalities or, in the younger child,

19
iSummary SHORT NOTES

aspiration. When the diagnosis is uncertain, spirometry is the most useful


confirmatory test, but most children younger than 7 years cannot complete
the procedure.
 Management
 If the patient does not completely improve from the initial therapy, with
PEFR between 50% and 80%, the β2-agonist should be continued, and an
oral corticosteroid should be added. If the patient experiences marked
distress or if PEFR persists under 50%, the patient should repeat the β2-
agonist immediately. Further emergent treatment may be necessary
 Long-acting β2-agonists were more effective than leukotriene antagonists
as add-on-therapy to inhaled steroids. However, there is persistent concern
that long-acting β2-agonists can increase mortality when used as
monotherapy in the absence of inhaled corticosteroids
 Leukotriene inhibitors should be used only in asthma patients who are
already using a corticosteroid inhaler—or those who cannot tolerate
inhaled corticosteroid therapy.
 Use of short-acting β2-agonists more than two times a week in intermittent
asthma (daily or increasing use in persistent asthma) may indicate the need
to initiate (increase) long-term-control therapy
 One additional treatment that appears to have evidence for efficacy in
treating acute exacerbations of asthma is intravenous magnesium sulfate

SABA Ipratropium Inhaled Systemic LABA Theophylline Leukotriene Mast cell-


Corticosteroid corticosteroid modifiers stabilizing
Mild  as   prior to
Intermittent needed exercise

Mild  as  low dose ± ± Alternativ


Persistent needed e choice to
inhaled
steroids
Moderate  as  Medium- ±  ±
Persistent needed to high-dose
Severe  as  High-dose ±  ± ±
Asthma needed
Acute   
Asthma
Exacerbation

 COPD

 Spirometry is the key to making a formal diagnosis, as well as for staging the
severity of illness. postbronchodilator FVC of less than 80% of predicted
in a patient with evidence of airway obstruction (FEV1/FVC ratio less than
70%)
 Anticholinergic drugs are the most effective in long-term management of
COPD, Tiotropium improves health status and reduces exacerbations
compared to ipratropium.

21
iSummary SHORT NOTES

 Pneumonia

 The most common bacterial cause of community-acquired pneumonia is


Streptococcus pneumoniae
 Mycoplasma is the most common pathogen responsible for lung infections
in patients between 5 and 35 years of age.
 The follow-up visit to the office 3 to 4 days later will help to assess response
to therapy
 Early follow-up chest x-rays are mandatory in those who fail to show
clinical improvement by 5 to 7 days, because bronchogenic carcinoma can
present with the picture of a typical pneumonitis
 In patients with lobar or segmental pneumonias that do not clear with
antibiotic therapy, further evaluation with computed tomography may be
advisable to evaluate for an obstructing tumor. In addition, because of the
increased possibility of an underlying tumor associated with pneumonia in
patients older than 40 years, a follow-up chest film is indicated in 4 to 6
weeks in such individuals.
 Bacterial pneumonia can rarely late 3-5 months to clear on x-ray. Thus,
repeating the chest x-ray in 2 weeks is unlikely to show resolution. In elderly
patients, the chest x-ray takes longer to normalize than in younger patients.

 Pulmonary tuberculosis

 Screening
 An annual screening PPD (in asymptomatic individuals) is indicated for:
 Those with HIV infection.
 Health care workers, prison guards, and mycobacteriology laboratory
personnel.
 Those with a medical condition that ↑ the risk of active TB (e.g.,
diabetes, use of immunosuppressive medications, end-stage renal
disease, alcoholism, conditions associated with rapid weight loss or
chronic malnutrition).
 Homeless and IV drug users.
 Those who reside in a long-term care facility.
 A one-time screen is indicated for:
 Those with a single potential exposure to TB (repeat PPD in 6–12 weeks
if the exposure is recent).
 Those with an incidentally discovered fibrotic lung lesion.
 Immigrants and refugees from countries with a high prevalence of TB.

 Clinical diagnosis in endemic areas is often based on history of exposure,


clinical signs, acid-fast bacillus (AFB) smears, and chest x-ray findings.
Sputum cultures can confirm the diagnosis, and they also are important for
identifying patterns of drug resistance.

21
iSummary SHORT NOTES

 Management
 Children, pregnant women, and adults at risk for INH-induced neuritis
should receive pyridoxine (vitamin B6) daily
 In pregnancyStreptomycin should be avoided because of ototoxicity to
the fetus. PZA is not recommended because its teratogenicity is unknown.
 Lactating women who are taking antituberculous medication should
breastfeed before ingesting their medication. Bottle supplementation
should be used for the first feeding after dosing. (For infants whose mothers
were treated for active TB during pregnancy and who are themselves on
INH for treatment of LTBI, bottle-feeding is recommended.)
 Liver enzymes, bilirubin, creatinine, and a complete blood count/platelet
count should be obtained as baseline information before implementing the
standard regimens. While liver injury is a significant problem with isoniazid,
the drug need not be stopped unless the liver enzymes rise to >3 times the
upper limit of normal.
 If PZA is to be used, uric acid should be obtained. If ETH is included in the
regimen, obtain baseline and monthly visual acuity as well as red-green
perception testing to detect drug-induced optic neuritis
 Repeated sputum examinations, beginning with weekly smear quantitation,
are desirable until sputum conversion is documented.
 Latent TB For women at risk for progression of LTBI to disease, especially
those who are infected with HIV or who have likely been infected recently,
initiation of therapy should not be delayed on the basis of pregnancy alone,
even during the first trimester. For women whose risk for active TB is lower,
some experts recommend waiting 3 months postpartum because of the risk
of INH hepatotoxcity.
 Treatment of patients who have positive PPD and x-ray evidence of
tuberculosis but negative sputum smears depends on the level of clinical
suspicion for active tuberculosis.
 When suspicion is highmultidrug therapy should be initiated
pending results of culture.
 If cultures negative but the patient shows clinical or radiographic
signs of improvement after 2 months of treatment, then the
patient is assumed to have culture-negative TB, and treatment
should be completed using isoniazid and rifampin.
 If culture remains negative and there is no sign of clinical or
radiographic improvement, treatment discontinued after 2
months.
 For patients at low suspicion of TBno treatment is indicated pending
results of cultures.
 If cultures remain negative and the patient is asymptomatic with no
progression on chest x-ray,  treat as latent tuberculosis

22
iSummary SHORT NOTES

 Tuberculin skin testing (TST)

 Tuberculin skin testing the test is interpreted at 48 to 72 hours by


measuring the degree of maximum induration, not erythema; it is
acceptable to read the skin test up to 96 hours. False-positive PPDs can be
due to BCG vaccine and infection with other mycobacteria. However,
patients who are 1 year or more out from their BCG vaccine should be
treated as though they are true responders

 Three cutoff levels for determining a positive TST reaction are based on
sensitivity, specificity, and prevalence of tuberculosis in different groups:

 A ≥5-mm induration for HIV-positive individuals, recent contacts with


persons with active TB, individuals whose CXR findings are consistent
with old healed TB (fibrotic changes), and patients with organ
transplants and other immunosuppressed patients receiving more than
15 mg per day of prednisone for more than 1 month.

 A ≥10-mm induration for recent immigrants (within the last 5 years)


from high-prevalence countries; low-income minority populations;
residents and employees of correctional facilities, nursing homes, and
shelters; health care workers; injection drug users, individuals with
chronic medical conditions; and children younger than 4 years or
infants, children, and adolescents exposed to adults in high-risk
categories.

 A ≥15-mm induration for very low-risk patients (age older than 5 years,
no history of exposure, normal immune system, and low rates of TB in
the surrounding population).

 False negative skin tests may be seen in the following:

 Anergic states such as HIV and malignancy.


 Newly diagnosed pulmonary TB or severe extrapulmonary TB.
 Corticosteroids or immunosuppressive therapy.
 Concurrent viral infection.
 Poor nutrition.
 Because the prevalence of a positive PPD (Mantoux test) doubles between
the first and second tests in initial nonresponders, the U.S. Public Health
Task Force recommends a 2-step PPD by the Mantoux method for screening
high-risk populations (eg, individuals living in nursing homes). If the first PPD
is negative, a second test should be performed approximately 2 weeks
later in order to detect the “booster phenomenon.”

23
iSummary SHORT NOTES

 Bronchiectasis

 An abnormal and permanent dilatation of bronchi that is either focal or


diffuse and is the result of repeated cycles of infection → pooling of
secretions in the dilated airways. It is caused by CF in 50% of cases.

 Cancer lung

 Diagnostic evaluation can begin with chest x-ray, but a negative x-ray does
not rule out lung cancer, nor does it provide tissue diagnosis or staging.
High-resolution CT scanning (HRCT) is a more sensitive test for identifying
lung cancers even at asymptomatic stages

24
iSummary SHORT NOTES

GIT &
HEPATOLOGY
Disease Management
Irritable ●Cognitive-behavioral therapy
bowel ●Constipation ,
syndrome ; 70% and .

●Diarrhea

●Gas, bloating, or flatus , β-


●Abdominal painanticholinergics ( , ,
and others) and antispasmodics.
●Chronic pain tricyclic antidepressants, selective serotonin
reuptake inhibitors
● Dietary fiber
GERD ●Lifestyle modifications
●Antacids● Histamine type-2 receptor antagonists
(H2RA):
● Proton Pump Inhibitors (PPI):

● Prokinetic therapy
● Antireflux surgery
Non-ulcer ● Lifestyle modifications ●±H2RA or PPI or Metoclopramide
dyspepsia
Peptic ulcer ● Triple therapy PPI+ Amoxicillin/Metronidazole(if allergy) +
(H. Pylori) Clarithromycin/ Metronidazole(if allergy)14 days
● Quadruple therapyPPI + Tetracycline + Metronidazole +
Bismuth subsalicylate 14 days
● Sequential therapyPPI + Amoxicillin day 1:5 then
Clarithromycin + Tinidazole DAY 6:10
● Surgery failure of medical management (e.g., a gastric ulcer
that fails to heal after 12 weeks); suspicion of cancer; perforation;
outlet obstruction (failing to resolve after > 72 hours); and failure
to arrest bleeding.
Peptic ulcer ●PPI or an H2RA and the eradication of H. pylori, if present
(NSAIDs) ● Prostaglandin E1 analogue (if NSAIDs or aspirin can't
be stopped)
Hepatitis A ● Supportive carerest, attention to fluid and electrolyte balance
is indicated in severe cases.
● Immune globulin (IG) single IM dose of IG as soon as possible,
but not more than 2 weeks after the last exposure.
Hepatitis B ● Interferon-α ● lamivudine ●Adefovir dipivoxil
● Hepatitis B hyperimmunoglobulin  prevent symptoms after

25
iSummary SHORT NOTES

known and suspected exposure


Hepatitis C ● interferon 2b (PEG-IFN) or 2a ± ribavirin
Ascites ● Sodium restriction 2 g/day.
●1.Spironolactone and furosemide ● Triamterene and amiloride
and Bumetanide and torsemide (alternatives)
●Repeated large-volume paracentesis Albumin replacement is
controversial but should be considered if > 4–5 L is removed.
●Transjugular intrahepatic portosystemic shunt (TIPS):
Controversial, last resort for variceal hemorrhage
●Liver transplantation For refractory cases

Disease Symptoms/sig Diagnosis Management


n
Nonalcoholic ●Asymptomatic ●↑ALT and AST ● Gradual weight loss,
steatohepatitis ● ALT-to-AST ratio is > dietary fat restriction,
1. exercise
Hemochromatosi ● Slate-gray ● Hyperglycemia ●Phlebotomy
s pigmentation of the ● Fasting transferrin ●Deferoxaminerare
skin “bronze saturation > 45%; ly needed.
diabetes” ●↑ ferritin ●A normal diet, avoid
iron supplements,
vitamin C, shellfish
Wilson’s disease ● Kayser-Fleischer ●↓ copper and serum ● D-penicillamine or
rings ceruloplasmin trientene
↑ 24-hour urinary ● Oral zinc
copper excretion ●↓ of copper-
containing foods
(shellfish, organs,
legumes) is essential
● Liver transplant
Autoimmune ● Asymptomatic or ● Aminotransferases ● Prednisone with
hepatitis fulminant are ↑ (100–1000s). azathioprine or 6-MP
● young women ●↑gamma globulin
(SPEP).
●AutoantibodiesAN
A, ASMA (type I), and
ALKM (type II)
1° biliary ● Asymptomatic ● Alkaline ● Ursodeoxycholic
cirrhosis ● Fatigue and phosphatase > twice ●Cholestyramine 
pruritus, normal pruritus (rifampin is
Osteoporosis ● Antimitochondrial second line).
AB ●Liver transplantation
Hepatic ● Sleep ● Evaluate for a GI ●Correct the
encephalopathy disturbances bleed precipitating
progresses to eventNG lavage, K.
disorientation and ●Lactuloseretentio
confusion with n enema can be used
asterixis if oral lactulose is not
well tolerated.

26
iSummary SHORT NOTES

●Antibiotics
neomycin, rifaximin
●protein
restrictionevidence
does not support this

 IBS

 Rome II symptom criteria for IBS At least 12 weeks or more, which need
not be consecutive, in the preceding 12 months of:
 abdominal discomfort or pain that has two of three features:
 Relieved with defecation; and/or
 Onset associated with a change in frequency of stool (diarrhea or
constipation); and/or
 Onset associated with a change in form (appearance) of stool (loose,
watery, or pelletlike)

 GERD

 Drugs cause GERD calcium channel blockers, β-agonists, α-adrenergic


agonists, theophylline, anticholinergics, diazepam, narcotics, progesterone,
nitrates, and sedatives.
 In the absence of warning signs or alarm symptoms, the diagnosis of GERD
can be made clinically without additional testing. In the presence of warning
signs, upper endoscopy is recommended. Also endoscopy may be indicated
if patients have had chronic, untreated symptoms as their risk for Barrett
esophagus is higher
 An inadequate response to a 4- or 8-week trial of standard-dose PPI may
indicate that longer treatment is needed, more severe disease, or an
incorrect diagnosis. Additional benefit may be obtained by extending
treatment for another 4 to 8 weeks with the same or double doses of a PPI.

 Peptic ulcer

 Treatment with proton pump inhibitors (PPIs) significantly decreases the


sensitivity of endoscopy. Ideally, endoscopy should take place prior to
treatment, or PPIs should be discontinued for at least 4 weeks before
endoscopy
 Serologic IgG tests it is appropriate for patients never treated for H.
pylori in the past. It is not reliable to determine successful eradication of H.
pylori.
 Stool antigen test the stool antigen test has been recommended by the
European Helicobacter Pylori Study Group (EHPSG) as the preferred initial
noninvasive diagnostic test.

27
iSummary SHORT NOTES

 Urea breath test the urea breath test is the recommended standard to
determine whether H. pylori has been successfully eradicated. If an ulcer is
diagnosed endoscopically, a rapid urease test is the quickest means to
determine HP. Urea breath test should be delayed for 4 weeks following
treatment, as acid suppression can lead to false-positive results
 To confirm eradication after H pylori treatment for PUD, the patient should
ideally be retested for active H pylori 4 to 8 weeks after treatment, with
either the urea breath test or stool antigen test.
 Many tests for H. pylori (except serology testing) will be falsely negative if a
patient is on acid suppression therapy.
 Histology of tissue from at least two different sites  gold standard for
diagnosis of H pylori infection
 Treatment failure requires a second regimen, generally quadruple therapy
with alternative antibiotics. A second treatment failure requires specialty
referral with endoscopy, and culture for sensitivities.
 Consider unusual causes, particularly acid hypersecretory states. A fasting
gastrin level is indicated for multiple ulcers, ulcers resistant to therapy,
ulcer patients awaiting surgery, ulcers associated with severe esophagitis,
and patients with a family history of similar ulcer problems or other
endocrine tumors
 The American College of Gastroenterology has identified risk factors for
PUD that should be considered when starting someone on an NSAID. Use of
a PPI for prophylaxis should be considered for these patients  ●Prior
history of a gastrointestinal event (ulcer, hemorrhage) ● Age > 60 ● High
dosage of an NSAID ●Concurrent use of glucocorticoids or anticoagulants

 Diarrhea

 Investigation of an exact etiology of infectious diarrhea is recommended


for
 Profuse diarrhea with signs of dehydration, fever higher than 38.5◦ C
 Patients aged younger than 3 months or older than 70 years.
 Immunocompromised patients.
 Patients with persistent diarrhea or adults with more than six stools per
day lasting longer than 2 days
 Those at risk of transmitting infections (e.g., food handlers in food
service establishments, health care workers, attendees/residents,
employees of day care, or an institutional facility such as a psychiatric
hospital, prison, or nursing home).
 Severe abdominal pain in patients > 50 years of age, community
outbreaks, recent use of antibiotics, and hospitalization.

 If neither stool leukocytes nor blood is present, stool cultures are not
necessary because they are rarely positive in such patients
 Antibiotics if indicated ●Fluoroquinolones: For suspected bacterial
infection; give for 3–5 days. ●Administer a macrolide if resistance is

28
iSummary SHORT NOTES

suspected, especially in traveler’s diarrhea. ●Metronidazole: For cases


suspected to be related to Giardia or C. difficile. ●Always give antibiotics to
elderly patients, immunocompromised patients, and those with heart or
vascular grafts.

 Inflammatory bowel diseases

 CD transmural inflammatory infiltrate located at any level of the GI tract


from mouth to anus. CD often presents as segmental involvement of two or
more colonic areas; between these areas the colon is normal. Mainly mild
non-bloody diarrhea and mild chronic abdominal pain
 UC is limited to the colon, usually involves only the superficial layers of the
bowel, and is continuous in nature. UC virtually always involves the rectal
mucosa. rectal bleeding
 Sigmoidoscopy is the first diagnostic test to perform in the patient with
bloody diarrhea
 Colonoscopy is often the only method of diagnosing early ileal or colonic CD
 IBD increases the risk of colon cancer, so more frequent and earlier
colonoscopies are recommended. Recommendations vary, but most
recommend colonoscopy every 1–2 years in patients with UC after 8 years
with disease. Screening in those with Crohn is less well established. The risk
of colon cancer is greater in UC than in Crohn

 Predisposing conditions to Cancer colon

 Adenomatous polyps: Present in 35% of those > 50 years of age. The risk of
progression to cancer ranges from < 4% to > 10% depending on size and
histologic features.
 Familial adenomatous polyposis (FAP): Polyps appear by age 15,
adenomatous polyps by age 35 and colorectal cancer by age 50. Colectomy
or proctocolectomy is indicated before age 20.
 Hereditary nonpolyposis colorectal cancer (HNPCC): An autosomal-
dominant syndrome; also ↑ risk for endometrial and other cancers.
Associated with a 70 –80% lifetime risk of colorectal cancer. If cancer is
found, treatment is subtotal colectomy.
 A villous polyp has a higher malignant potential; if larger than 2 cm, it has a
50% chance of containing invasive cancer.
 Surveillance in survivors
 Colon cancer: Evaluate every 3–6 months for 3–5 years with a history
and exam, FOBT, LFTs, and CEA. Colonoscopy should be performed 6–
12 months after surgery and every 3–5 years thereafter.
 Rectal cancer: As with colon cancer, but sigmoidoscopy every 6–12
months for three years.
 AFP> 500 µg/L in high-risk patients is virtually diagnostic of HCC

29
iSummary SHORT NOTES

 Liver diseases

 The incubation period●HAV15 to 45 days (average 4 weeks) ●HBV


between 60 and 90 days, with a range of 45–180 days ●HCV between 2
and 26 weeks but most commonly is 6–7 weeks
 HAV The virus is excreted in the stool during the first few weeks of the
illness prior to onset of symptoms. Children can shed HAV virus in their stool
for several months after the onset of clinical disease
 Most people infected with HCV (85%) develop a chronic infection. The
incidence of significant liver disease is 20–30% for cirrhosis and 4% for liver
failure; over 1–4% of patients with chronic infection develop HCC annually.

 The diagnosis of acute HCV disease can be made 1 to 2 weeks after


infection through detection of HCV RNA using reverse transcriptase
polymerase chain reaction (RT-PCR)
 HCV antibody is detectable by 5 to 6 weeks postexposure in 80% of infected
patients and by 12 weeks in 90%.
 HBsAg is found in serum 30 to 60 (about 6 weeks) days after HBV infection.
 Loss of HBe Ag (with or without conversion to anti-HBe antibody positivity)
indicates decreased viral replication and less of a risk of progression to
cirrhosis. Loss of HBe Ag may occur spontaneously; it is also the therapeutic
endpoint of antiviral treatments for HBV infection (interferon, lamivudine,
adefovir).
 Quantitative HCV PCR is not a reliable means for diagnosing HCV infection
because currently used methods are insensitive at low levels of viremia;
thus, infection cannot be ruled out if the level of HCV viremia is below the
lower limit of detection of the test.
 Qualitative HCV PCR is the most sensitive test for the presence of HCV RNA,
with a limit of detection that is lower than that of quantitative PCR. It is
useful to establish the presence of viremia, but is more expensive than
antibody testing and thus not a first-line test.

 When the transaminase values reach or exceed 10,000 IU/mL, the most
common causes are drug injury, exposure to hepatotoxins, or ischemia.
 When transaminase levels are in the range of 1500 to 3000 IU/mL, acute
viral hepatitis is the major concern.
 If the transaminase values are lower than 300 IU/mL, chronic viral hepatitis,
metabolic disorders, and alcoholic hepatitis are of general concern.
 In more than 70% of patients with alcoholic hepatitis, the AST/ALT ratio is
higher than 2 and elevations in the γ-glutamyltransferase (GGT) level are
seen.

 Prevention of HAV ●immunoglobulin during the incubation period not


>2 weeks after the last exposure. People vaccinated with at least one dose
of hepatitis A vaccine more than a month prior to exposure do not need IG.
● Vaccination 3 doses for children > 1 year, 2 doses for adolescent and
adult. Travelers who receive the vaccine may assume to be protected 4

31
iSummary SHORT NOTES

weeks after receiving the first dose, although the second dose is needed for
long-term protection. If travel is anticipated in less than 4 weeks,
immunoglobulin may be given in a different site for additional protection
 Repeat testing of all infants born to HBV-infected mothers at 9–15 months
with HBsAg and anti-HBs.

 Alcoholic hepatitis: AST usually > 300–400 and 2× ALT


 Prophylaxis of Spontaneous bacterial peritonitis
 Prior SBP: Norfloxacin or TMP-SMX QD.
 GI bleeding in the setting of cirrhosis: Norfloxacin or TMP-SMX × 7 days
(there is an ↑ risk of SBP in these patients).
 Ascites and ascitic protein < 1 g/dL OR bilirubin > 2.5 mg/dL:
Norfloxacin or TMP-SMX QD.
 Precipitating causes of Hepatic encephalopathy  GI bleeding (↑ urea
levels and ammonia levels), ↑ protein intake, hypokalemia (↓ renal
ammonia excretion), alkalosis (favors NH4 + → NH3, which crosses the
blood-brain barrier), infection, or constipation (↓ GI clearance of
ammonia). Exacerbated by hypoxia and sedatives/tranquilizers.
 1° prophylaxis for variceal hemorrhage: Prophylaxis with nonselective β-
blockers (propranolol, nadolol) is preferable to prophylactic banding
therapy. Nitrates are not recommended.
 A liver biopsy is usually indicated if a treatable cause of cirrhosis, such as
Wilson disease or hepatitis C is suspected.
 ALT is found predominantly in the liver and is a more specific indicator of
liver damage.
 Surveillance for hepatocellular carcinoma (HCC) should be performed every
6 to 12 months with ultrasound and serum alpha fetoproteain (AFP). (SOR
A)

 Hepatitis b viral serologies for different phases of infection

31
iSummary SHORT NOTES

 Jaundice
 hemolysis or a defect of bilirubin
conjugation or transport.
Elevated unconjugated bilirubin (>80% to 85% of total) hemolysis or hereditary conjugating defects
(Gilbert syndrome or Crigler-Najjar syndrome)
 Obtain a complete blood count (CBC), blood smear, reticulocyte count, and lactate dehydrogenase
(LDH) → May be hemolysis.
 If laboratory results are not consistent with hemolysis, suspect a defect in conjugation, especially
if the total bilirubin is less than 6 mg/dL and conjugated bilirubin is normal.
 Gilbert syndrome is the most common conjugating defect (7% of population): bilirubin is
usually less than 3 mg/dL but increases with fever, fasting, or stress though rarely exceeding
6 mg/dL; patients are asymptomatic and have normal liver histology.
 Crigler-Najjar syndrome is rare. In Type I bilirubin levels rise to 50 mg/dL and death occurs in
infancy. In Type II bilirubin values may reach 20 mg/dL but no severe sequelae result except
jaundice.

Elevated conjugated bilirubin (>50% of total)  congenital defect in conjugated bilirubin transport
(Rotor syndrome or Dubin-Johnson syndrome)
 These conditions appear in childhood or adolescence with bilirubin levels up to 25 mg/dL, but cause
no clinical sequelae. They follow an autosomal recessive inheritance pattern.
 Patients with Rotor syndrome demonstrate visualization of the gallbladder on oral
cholecystogram (OCG).
 In Dubin-Johnson syndrome the gallbladder is not seen on OCG; pathognomonic black pigment
is found on liver biopsy.

 hepatocellular injury
 Acute or chronic viral hepatitis diagnosed by viral hepatitis serology.
 Alcoholic hepatitis clinically resembles viral or toxic hepatitis, but AST is usually greater than ALT (a
reversal of the usual ratio); diagnosis is based on a history of heavy alcohol intake, absence of other
causes of hepatitis, and liver biopsy.
 Hereditary liver disease:
 Wilson disease is confirmed by low ceruloplasmin levels and Kayser-Fleischer rings or liver
biopsy.
 Hemochromatosis is suspected in patients with a history of hepatomegaly, idiopathic
cardiomyopathy, skin pigmentation, loss of libido, diabetes mellitus, or arthritis; elevated
transferritin saturation and ferritin levels suggest the diagnosis, which is confirmed by genetic
testing or liver biopsy.
 α1-Antitrypsin deficiency is associated with pulmonary disease and confirmed by decreased α1-
antitrypsin levels.
 Congestive and ischemic diseases including right-sided congestive heart failure, constrictive
pericarditis, Budd-Chiari syndrome (hepatic vein or inferior vena cava obstruction), portal vein
thrombosis, veno-occlusive disease, and hypotension are causes of jaundice and hepatocellular
injury; diagnosis is based on other physical findings.
 Liver diseases in pregnancy that cause hepatocellular injury are acute fatty liver of pregnancy and
toxemia.
 Drug-induced hepatitis can be confirmed by drug levels (e.g., acetaminophen), agent-specific
patterns of hepatotoxicity, and, occasionally, liver biopsy.
 Autoimmune hepatitis is suspected when antinuclear antibodies, smooth muscle antibodies, or

32
iSummary SHORT NOTES

antimitochondrial antibodies are present; liver biopsy is helpful.

 cholestasis
This may be extrahepatic cholestasis
 If extrahepatic cholestasis is suspected based on history and physical examination, possible causes
include choledocholithiasis, malignancies (pancreatic, bile duct, lymphoma, metastases), biliary
stricture, sclerosing cholangitis, chronic pancreatitis, biliary atresia, and other rare conditions (Asian
cholangiohepatitis, ascariasis, hemobilia).

Or may be an intrahepatic cholestasis


 If intrahepatic cholestasis is suspected based on history and physical examination, order specific
laboratory tests.
 Acute and chronic hepatitis (viral, alcohol, and drug-induced).
 Cirrhosis is most commonly caused by long-term alcohol use and viral infections, especially
hepatitis C. Rarer causes include genetic and metabolic diseases (Wilson disease,
hemochromatosis, and α1-antitrypsin deficiency) and autoimmune diseases (primary biliary
cirrhosis, primary sclerosing cholangitis, and lupoid hepatitis). Liver biopsy may be helpful if the
diagnosis or cause is unclear.
 Chronic cholestatic syndromes include primary biliary cirrhosis and primary sclerosing
cholangitis. Primary biliary cirrhosis is usually seen in middle-aged women; antimitochondrial
antibodies are elevated in more than 90% of patients; liver biopsy confirms the diagnosis.
 Primary sclerosing cholangitis may be seen as an isolated finding or in association with
inflammatory bowel disease, other fibrosclerosing syndromes, or AIDS; diagnosis is confirmed
by ERCP or magnetic resonance cholangiogram.
 Benign recurrent intrahepatic cholestasis is diagnosed by recurrent episodes, family history,
and absence of obstruction on cholangiography.
 Cholestasis of pregnancy is usually seen in the third trimester and often recurs in subsequent
pregnancies or in association with estrogen use.
 Cholestasis can also be seen with sepsis, parenteral nutrition, postoperative state, or neoplasm.

 Transaminases may also be elevated. 5′-nucleotidase and GGTP are usually elevated; if not,
consider a bone source.

 Screening of colon cancer (according to level of risk)

 Average risk: No family history or one second- or third-degree relative with


cancer. Begin screening at age 50.

 Elevated risk: Refers to patients with a first-degree relative diagnosed with


cancer/adenoma at > 60 years of age or two or more second-degree
relatives with cancer only. Begin screening at age 40.
 High risk: Describes patients with a first-degree relative diagnosed with
cancer/adenoma at < 60 years of age or two first-degree relatives with
cancer/adenoma. Begin at age 40 or when the patient is 10 years younger
than the family member’s age at diagnosis; repeat every five years.

33
iSummary SHORT NOTES

 Adenomatous polyp on screening colonoscopy: Repeat colonoscopy in


three years.

 FAP (Familial adenomatous polyposis): Genetic testing offered to first-


degree family members after age 10; if inconclusive, sigmoidoscopy
performed beginning at age 12. Upper endoscopic evaluation of the
duodenum/periampullary area every 1–3 years.

 HNPCC (Hereditary nonpolyposis colorectal cancer): Offer genetic


counseling and testing if the family history is suggestive. If testing is
positive, screen with colonoscopy every 1–2 years starting at age 25 or
when the patient is five years younger than the age of the family member at
diagnosis. Also screen annually for endometrial cancer from 25 to 35 with
endometrial aspirate or transvaginal ultrasound (or consider prophylactic
TAH-BSO).

 IBD: No consensus; the general recommendation is colonoscopy every 1–2


years beginning eight years after a diagnosis of IBD.
 Adenomatous colon polyps, either pedunculated or sessile, are associated
with transformation to cancer. Hyperplastic polyps are considered benign.

34
iSummary SHORT NOTES

RENAL DISEASES
Disease Diagnosis Investigation Management
Nephrotic ● Loss of ● Proteinuria>3.5 ●Sodium and fluid
Syndrome appetite, fatigue g/d restriction
and malaise, ●Oval fat bodies or ●Control peripheral edema
edema (Swelling “Maltese crosses” Loop diuretics.
of eyelids and ●serum albumin < ●ACEIs/ARBs  slow
face)… 3g/dl proteinuria.
● hyperlipidemia, ●Lipid-lowering therapy
lipiduria target LDL generally < 100.
●Anticoagulant therapy
(while patients have
nephroyic proteinuria
and/or albumin level <
20g/L)
●Prednisone, and, in some
cases, cytotoxic drugs.
●Treat the underlying
disease
Nephritic ● Hematuria, ● RBC casts ● Bed rest & protein
syndrome hypertension, ● Proteinuria<3.5 restriction (BUN or
(PSGN) renal g/d creatinine )
insufficiency, ●Oliguria ●Fluid overload and
and edema ●ARF over days to hypertensiondiuretics
weeks and other antihypertensive
as needed.
●Immunosuppressive if
heavy proteinuria or rapidly
decreasing GFR is present.
●If acute renal insufficiency
develops and volume
overload is unresponsive to
diureticshemodialysis
Disease Management
Spontaneous ●1.Cefotaxime + albumin infusion
bacterial
peritonitis
Asymptomatic ●Pregnant  ● Amoxicillin 7 days ●Nitrofurantoin or a
bacteriuria cephalosporin if allergy to penicillin
Uncomplicated ● Symptomatic phenazopyridine
bacterial ● Single-Dose AB Fosfomycin, TMP-SMX
cystitis ● Short-Term (3-day) AB 1. TMP-SMX - Cephalexin
,Ciprofloxacin , Lomefloxacin , Nitrofurantoin , Nitrofurantoin SR,
Norfloxacin , Ofloxacin

35
iSummary SHORT NOTES

●Longer-term treatment (7-day)  fail to respond to a 3-day


regimen, Children, men, and pregnant women
●Pregnant  amoxicillin, nitrofurantoin 7 days
●Prophylactic antibiotics (TMP-SMX, Nitrofurantoin, Cranberry
tablets) for 6 months to 1 year in women with ≥ 3 infections
in a year
Complicated ●≥10-14 days AB ● Ampicillin and gentamicin IV ●
cystitis Fluoroquinolone ● Piperacillin and tazobactam IV ● Ticarcillin
and clavulanate IV ● Meropenem IV
Pyelonephritis ● 1.fluoroquinolone
●Aminoglycosides; aminopenicillins such as amoxicillin ±
clavulanic acid 7–14 days, ticarcillin, or piperacillin;
cephalosporins; fluoroquinolones 7–14 days; or, in extreme
cases, imipenem

 Membranous nephropathythe most common cause of nephrotic syndrome;


presents with proteinuria with occasional microhematuria, and
hypercoagulability (renal vein thrombosis)
 Poststreptococcal glomerulonephritis (PSGN) The leading cause of acute
nephritic syndrome. Classically seen 2–3 weeks after pharyngitis, ear or skin
infection

 Hematuria

 Microscopic hematuria ≥ 3 RBCs/hpf in centrifuged urine on 2 of 3


properly collected urine specimens. Further evaluation is indicated for
single specimens with > 100 RBCs/hpf.
 The first step in approaching asymptomatic microscopic hematuria is to
verify that it is a consistent finding. Patients should have repeat urinalysis,
and if two separate follow-up samples are normal, no further evaluation is
needed. If hematuria is consistently found, the next step is to risk stratify
the patient based on history, physical, and baseline laboratory findings.
Obtain urine cytology and consider a referral to urology for cystoscopy,
especially for patients at risk for malignancies ● Cigarette smoking ●
Occupational and environmental exposures (aluminum, certain paints and
dyes, petroleum, rubber and textile industries) ●Analgesic abuse
(phenacetin) ●High dietary consumption of fried meats and fats ● Chronic
cystitis ● Schistosomiasis ● Pelvic radiation and chemotherapy
(cyclophosphamide)

36
iSummary SHORT NOTES

 EVALUATION OF MICROSCOPIC HEMATURIA

After microscopic hematuria has been identified (2 of 3 urine samples with 3 or more RBC/HPF), the
AUA recommends the following evaluation:
 Infection identified → treat with antibiotics and repeat urinalysis
 RBC casts, proteinuria, or elevated creatinine → begin evaluation for glomerulonephritis and
consider referral to a nephrologist
 No infection or primary renal disease identified in first 2 steps → urine cytology, bladder
cystoscopy (if at risk for bladder cancer based on environmental exposures and/or >40 years), and
CT scan (helical CT if stones suspected, contrast-enhanced CT if stones not suspected)
 If entire thorough diagnostic evaluation negative → follow-up urinalysis, urine cytology, blood
 Proteinuria
pressure, and serum creatinine every 6–12 months

In type 1 DM, patients should be started on an ACE inhibitor if they screen


+ve for microalbuminuria, even if normotensive.
Adults with proteinuria need their Urine protein/urine creatinine (UPr/Ucr)
determined. Those with values outside the normal range should undergo an
evaluation for CKD. Patients with dipstick proteinuria but normal-range
UPr/UCr values should be rechecked at periodic follow-up examinations.

 Proteinuria Values
Test Protein Value
Dipstick ≥1+ if urine specific gravity ≤1.015 or ≥2+ if urine urine
specific gravity >1.015
UPr/UCr Children >0.5 (age 6mo to 2yr) or > 0.25 (>2 yr)
Adults >0.2
24-hour urine Children>4 mg/m2/hr or >100 mg/m2/day Adults 30-
assay 300 mg/24 hr—microalbuminuria or >300 mg/24 hr—
(normal <0.15 g albuminuria
per day)

 The absence of morning proteinuria, as evidenced by a normal UPr/UCr


ratio, supports the diagnosis of benign orthostatic proteinuria

 Renal failure

 Remember that a BUN/Cr ratio >20 generally indicates a prerenal cause of


azotemia. A BUN/Cr ratio <20 is suggestive of intrinsic renal disease or
urinary outlet obstruction.
 Slow the progression of CKD ●Reduction of proteinuria with ACEIs/ARBs:
Beneficial for both diabetic and nondiabetic nephropathy. Also
nondihydropyridine calcium channel blockers (diltiazem or verapamil) slow
the progression of CKD. ●Treatment of hypertension: Treat hypertension to
< 130/80 with ACEIs (first-line therapy) or diuretics (second-line treatment).
●Lipid lowering with an LDL target < 100 (statin) ● Protein restriction
(controversial) and smoking cessation ● Maintain careful fluid balance (may

37
iSummary SHORT NOTES

need to restrict fluids if edema is present) ●Restrict sodium, potassium,


and phosphate. ●Treatment of anemia (erythropoietin). ●Diuretics if
significant fluid overload is present. ● Control of renal osteodystrophy with
calcium and vitamin D supplementation.
 In chronic renal failure due to any cause, absolute clinical indications for
initiating dialysis  persistent nausea and vomiting, pericarditis, fluid
overload, uremic encephalopathy, accelerated hypertension, bleeding due
to uremia, serum creatinine >12 mg/dL, and severe electrolyte
abnormalities.

 Urinary tract infection

 The American College of Obstetrics and Gynecologists recommends


screening all pregnant for asymptomatic bacteriuria with urine culture
(>105 organisms) at the initial prenatal visit and in the third trimester.
●Follow-up culture after treatment and monthly for the duration of the
pregnancy, and consideration of prophylactic nitrofurantoin following a
bout of pyelonephritis
 Recurrent cystitis is defined as three or more UTIs in the course of 1 year
(or two episodes in 6 months).
 Among uncomplicated UTIs, 80%–85% are caused by E coli
 Empiric antibiotics after the history can be a cost-effective method of
treating cystitis. The gold standard test is the urine culture and sensitivity
 Indications of urine culture in case of cystitis  ● in children, even in
uncomplicated cystitis ●After a patient self-administers antibiotics ●For
those who do not respond to initial therapy ●For patients who have
recurrent symptoms soon after therapy ●Patients with complicated UTIs ● if
pyelonephritis is suspected
 Risk factors for TMP-SMX resistance  recent antibiotic exposure, use of
TMP-SMX during the previous 6 months, recent hospitalization, diabetes
mellitus, ≥ 3 UTIs in the past year, and possibly use of oral contraceptive
pills or estrogen replacement therapy. If resistance is likely, a 3-day course
of a fluoroquinolone is reasonable. Use of fluoroquinolones as first-line
therapy should be discouraged.
 In cystitis Patients should return to the doctor if symptoms have not
resolved or markedly improved within 48 hours of starting antibiotic
treatment

 Risk factors of complicated cystitis


 Any cystitis not resolved after 3 days of appropriate antibiotic
treatment.
 Any cystitis in a special population, such as● A man. ●A patient
with anomaly (reflux) or stones. ●indwelling urinary catheter,
urinary tract instrumentation within the past 2 weeks ● Recent
systemic antibiotic use. ● Immunosuppression (e.g., poorly
controlled diabetes mellitus). ●A pregnant woman.

38
iSummary SHORT NOTES

 Any cystitis involving multiple resistant bacteria.


 High postvoid residual urine volume.
 Renal impairment and renal transplant.

 In pyelonephritis Imaging studies generally are not required unless in the


following ●Severely ill. ●Immunocompromised. ●Not responding to
treatment. ●Patients in whom complications are likely (e.g., pregnant
patients, diabetics, and those with nephrolithiasis, reflux, transplant
surgery, or other GU surgery).

39
iSummary SHORT NOTES

ENDOCRINOLOGY
Disease Diagnosis Investigation Management
Adrenal ● Weakness, weight ● A low plasma ● Hydrocortisone
insufficiency loss, cortisol of <3 mcg/dL and
hyperpigmentation, either in the morning fludrocortisone
craving for salt or at a time of stress
● short ACTH
stimulation test

Diabetic peripheral ● Pregabalin ● Duloxetine ● Venlafaxine XR ● Gabapentin


neuropathic pain ● Tricyclic antidepressants ● NSAIDs ● capsaicin cream
Hyperparathyroidism ● Parathyroidectomy ● aggressive hydration ● loop
diuretic ● etidronate, plicamycin, and calcitonin

 DIABETES MELLITUS

 Screening
 ADA recommends screening every 3 years beginning at age 45 or sooner
and more frequently in those with the following risk factors:  ●Family
history of diabetes. ●Hypertension. ●Dyslipidemia (especially  TG and ↓
HDL level). ●Obesity. ● High-risk ethnic or racial groups (African American,
Hispanic, Native American). ●Previous history of impaired glucose
tolerance. ● Gestational diabetes or birth of a child weighing more than 9
pounds (4 kg). ●Habitually physically inactive. ●Cardiovascular disease.
Polycystic ovarian disease.

 Medications that can predispose to type 2 DM beta blockers, atypical


antipsychotics, protease inhibitors, high-dose thiazides, or niacin, and
corticosteroid.
 Diagnosis
 Diagnosis of DM if  ● Two fasting plasma glucose (FPG) levels of over 126
mg/dL, A glucose tolerance test (75-g load) in which any blood glucose
value between time 0 and 2 hours exceeds 200 mg/dL.
 Impaired fasting glucose  Fasting plasma glucose ≥100 mg per dL and
<126 mg / dL
 Impaired glucose tolerance  2-hour postload (75 g anhydrous glucose)
plasma glucose of ≥140 mg per dL and ≤199 mg per dL
 Impaired fasting glucose, or impaired glucose tolerance should be
rescreened yearly.

41
iSummary SHORT NOTES

 Management
 A reasonable workup for the diagnosis of type 2 DM include laboratory
evaluation of fasting or random plasma glucose, HbA1c, fasting lipid profile,
serum creatinine, and urinalysis for ketone, glucose, protein, and
microalbuminuria. ● ECG should be performed at the onset of
microalbuminuria.

 ACE inhibitors have a special role in preserving renal function in type 1 DM


patients. Calcium-channel blockers are warranted in cases of intolerance of
or contraindications to ACE inhibitors or inadequate blood pressure control
with these agents. Angiotensin II receptor blockers (ARBs) are useful when
ACE inhibitors are not tolerated, the most common reason being cough.
 Current recommendations are to maintain total cholesterol below 200
mg/dL, triglycerides below 150 mg/dL, and LDL cholesterol below 100
mg/dL or below 70 mg/dL in the presence of cardiovascular disease.
 Statins are the drugs of choice in treating hyperlipidemia in diabetic
patients. Most patients who have an insufficient response or cannot
tolerate statins can now be managed with ezetimibe
 ACE inhibitors reduce cardiovascular (CV) events in diabetic patients older
than 55 years, with or without hypertension, who have one other typical CV
risk factor

 Feet should be examined at every office visit and patients instructed in good
foot care. Prevention of skin breakdown and infections is the best
treatment
 All patients should be screened yearly with a microalbumin test. The best
test to evaluate for microalbuminuria is the urine microalbumin/creatinine
ratio. Its advantages include ease of use, relatively low cost, and good
correlation with 24-hour urine collections
 An annual dilated ophthalmologic examination is indicated for all diabetic
patients

 Initial management of type 2 diabetes mellitus

 Minimally decompensated presentation  obesity and mild to


moderate hyperglycemia with or without symptoms.  patient
education, training, and motivation; an individualized hypocaloric diet
with diet training; and an exercise plan tailored to the individual.
Emphasize permanent lifestyle modification.

 Moderately decompensated presentation Obesity, severe


symptomatic hyperglycemia (fasting blood glucose >300 mg/dL), 
Temporary insulin therapy with daily glargine/detemir or prebreakfast
and bedtime dosing of NPH insulin at a starting daily dose of 0.5 unit
per kilogram of weight Prescribe an individualized hypocaloric diet with

41
iSummary SHORT NOTES

diet training and an exercise plan. The long-term goal is tapering and
withdrawal of insulin, assuming that glycemic control can be
maintained with diet and oral agent therapy.

 Severely decompensated presentation  Clinical picture shows a


severely symptomatic patient with blood glucose levels exceeding 500
mg per dL, marked dyslipidemia (serum triglycerides >1,000 mg/dL),
and hyperosmolality with absence of ketosis Intravenous fluids and
insulin (similar to DKA) in the hospital setting are necessary for acute
reversal of the metabolic derangement, followed later by a switch to
daily insulin glargine/detemir or prebreakfast and bedtime dosing of
NPH insulin.

 Patients with DM whose plasma glucose values are over 300 mg/dL should
not exercise until their control has improved and their blood glucose levels
have decreased. Vigorous exercise may be contraindicated in those with
proliferative or severe diabetic retinopathy. Self-monitoring of blood
glucose is useful during exercise. If blood glucose is low at the start of
exercise (≤100 mg/dL), a carbohydrate snack is indicated

Drug Side effects contraindications


Metformin ● GI nausea, ● Hypoxia, acidosis from any reason,
vomiting, anorexia, serious kidney or liver disease, and
flatus, diarrhea and anyone who is seriously ill or not
a metallic taste in taking anything by mouth
the mouth ●Creatinine > 1.5 mg/Dl, COPD,
●Lactic acidosis asthma, or advanced congestive
● Interfere with B12 heart failure
(as well as folate) ● Discontinued on the day of any
metabolism iodinated dye-load procedure or
surgery and should be withheld for 48
hours after the procedure. It should
not be restarted until a normal
posttest/surgery serum creatinine is
confirmed.
● Withheld during treatment for
pneumonia or acute MI
Sulfonylureas ● Hypoglycemia ● Significantly liver or kidney disease
(Glyburide, and weight gain ● Sulfa allergy
Glimeperide,
Glipizide)
Non sulfonylureas ● Hypoglycemia
secretagogues
(Rapaglinide,
Nateglinide)
Thiazolidinediones ●Idiosyncratic ● New York Heart Association class III
(Pioglitazone, hepatic dysfunction or IV congestive heart failure

42
iSummary SHORT NOTES

Rosiglitazone) ●Pedal edema


● Dilutional anemia
● Weight gain
●  LDL cholesterol
●extremity
fracture risk in
females

α-Glucosidase ● Flatus and nausea ● Cirrhosis, ketoacidosis, IBD, or other


Inhibitors ● Flatulence chronic intestinal disorders
● Elevation in ● Serum creatinine > 2.0 mg/dL
transaminases

 Metformin●For an overweight patient with a fasting glucose level of


about 150 mg/dL, we use metformin as a first-line drug. ●The drug of choice
in children with type 2 diabetes and has been used successfully in children
10 years of age and older. ●There is no evidence for its safety in younger
children. ●Metformin has been used in patients with impaired glucose
tolerance and shown to decrease the incidence of DM in that group. ●A
category B drug, it may be used in pregnancy but not during lactation
 Glimepiride It has a more rapid onset and longer duration of action than
other sulfonylureas, but induces less hypoglycemia, and may be the best
choice in patients with known coronary disease. Glimepiride and the
meglitinides may be used in mild azotemia because of their higher hepatic
excretion.
 Glipizide  preferred in patients with renal abnormalities
 Meglitinides disadvantage is that generic preparations are not available.
● They are particularly useful in patients whose fasting glucose levels are
well controlled but who have high post-prandial values or for patients who
eat few or irregular meals. These drugs are more expensive than oral
sulfonylureas but may prove useful in patients with renal impairment.
●They may be taken immediately before meals. If a meal is skipped, the
dose is skipped as well.
 α-Glucosidase They work only when taken with food, so they do not
cause hypoglycemia when used alone. They are particularly useful in
patients with erratic or poor eating habits, because they are taken only
with meals. ● Patients taking these agents will have trouble treating
hypoglycemic attacks with complex carbohydrates and so should have oral
glucose tablets readily available ● Serum transaminase levels must be
followed every 3 months for the first year.
 Thiazolidinedione TZDs are metabolized by the liver, so they may be used
in patients with renal failure, but Liver function tests should be monitored
every 2 months for the first 12 months in patients on these agents and
periodically thereafter
 Type 1 diabetics should monitor blood glucose at least four times a day.
 Patients with type 2 diabetes can monitor once or twice a day based on
their degree of control, but they need to include fasting, bedtime, and pre-

43
iSummary SHORT NOTES

and 2-hour post-prandial readings. Also recommended at least 2-hour


postprandial determination after meals containing newer foods; in this way,
the patient can self-estimate what and/or how much he or she can or
cannot eat and help monitor diet.

 Visit frequency is based on control of diabetes and the patient's


understanding and comfort.

 Patients initiating insulin therapy may require daily contact, by phone


or e-mail.
 Patients with poor control or making frequent changes may require
weekly to monthly visits.
 Patients with well-controlled diabetes usually need visits only
quarterly.

 Adult patient with DM should receive a yearly flu vaccine, a pneumococcal


vaccine every 5 to 7 years, and a tetanus vaccine every 10 years

 Diabetes screening recommendations

 Common endocrine disorders

 The most common cause of primary adrenal insufficiency is autoimmune


adrenalitis (Addison disease).
 ● Glucocorticoid administration is a more common secondary cause of
adrenal insufficiency
 No increase in serum cortisol or a blunted response after ACTH
administration confirms adrenal insufficiency
 ACTH-producing tumors account for 80% of cases of Cushing syndrome.
 If the serum ACTH level remained normal despite a high steroid load, there
must be an ACTH-producing neoplasm somewhere in the body

44
iSummary SHORT NOTES

 Cushing syndrome diagnosis ● central weight gain, moon facies, buffalo


hump ●24-hour urinary free cortisol test ● An overnight dexamethasone
suppression test ● Midnight cortisol level determination

 Primary hyperparathyroidism  The most common cause is a benign


solitary parathyroid adenoma (80% of all cases) ● Hypercalcemia (serum
calcium level > 10.5 mg/dL when corrected for serum albumin level) is the
most important clue to the diagnosis. ●An elevated PTH level in the
presence of hypercalcemia confirms the diagnosis of primary
hyperparathyroidism ● Any medications or other products that increase
calcium levels, such as estrogens, thiazides, vitamins A and D, and milk,
should be avoided.

 Hypoparathyroidism● The most common cause is the removal of the


parathyroid glands during a thyroidectomy or following surgery for primary
hyperparathyroidism. ● Symptoms associated with hypocalcemia include
tetany, carpopedal spasms, paresthesias of the lips and hands, and a
positive Chvostek sign or Trousseau sign ● patients with
hypoparathyroidism have low serum calcium and elevated serum
phosphate levels, with a normal alkaline phosphatase level. The key finding
is a low to absent PTH value

 Prolactinomas (the most common pituitary microadenoma)


 Pituitary adenoma ● A pituitary tumor <1 cm is considered a
microadenoma, and tumors ≥1 cm are considered macroadenomas.
●Treatment implications are slightly different. In both cases, medical
therapy with a dopamine agonist (eg, bromocriptine, cabergoline, etc) is
indicated. ●If the tumor is < 20 mm and is not causing any signs or
symptoms, observation may suffice. ●Surgery should be reserved for those
patients who fail to respond to or cannot tolerate medical therapy

 Sheehan’s syndromeInfarction of the pituitary that occurs postpartum


following substantial blood loss in childbirth. Presents with inability to
lactate, lethargy, and amenorrhea

45
iSummary SHORT NOTES

HEMATOLOGY
 Comparison of Anemia of Inflammation and Iron Deficiency
Anemia
Iron Deficiency Anemia of Inflammation
Low serum iron level Low serum iron level
Elevated total iron-binding capacity (TIBC) TIBC normal or reduced
Transferrin saturation low (<15%) Transferrin saturation low (15%-20%)
Serum ferritin level low (<15 ng/mL) Serum ferritin level normal or
elevated
Microcytic, hypochromic red blood cells Normocytic to microcytic RBCs
(RBCs)
RBC protoporphyrin level elevated RBC protoporphyrin level elevated

 Anemia

 In general, anemia is defined in adult males as Hb <13 g/dL and in adult


menstruating females as Hb <12 g/dL. Anemia of childhood occurs at Hb
<10.5 g/dL. Anemia of pregnancy occurs at Hb <11 g/dL in the first and third
trimesters or Hb < 10.5 g/dL in the second trimester. (CDC)

 Macrocytic anemias, those with MCV above 100 fL, are classified as
megaloblastic or nonmegaloblastic.
 Megaloblasts,  vitamin B12 deficiency and folic acid deficiency.
 Nonmegaloblastic alcoholism, hypothyroidism, and chronic liver
disease.
 Normocytic anemia (MCV between 80 and 100 fL) can be due to hemolytic
or nonhemolytic causes.
 Hemolysis  spherocytosis, G6PD deficiency, sickle cell disease,
autoantibodies, alloantibodies (in, for instance, transfusion reactions),
or a nonimmune process such as malaria or hypersplenism.
 Important nonhemolytic causes of normocytic anemia include poor
production of RBCs due to aplastic anemia, renal insufficiency, and bone
marrow infiltration.

  red cell distribution width with a low mean corpuscular volume is


suggestive of iron deficiency anemia. Conversely, the RDW will be normal in
thalassemias. The most sensitive test for diagnosis of iron-deficiency anemia
is the serum ferritin level.
 Men or women older than 65 years with iron deficiency require GI
endoscopy to screen for malignancy, (SOR B) as do men and non-

46
iSummary SHORT NOTES

menstruating women younger than 65 years with unexplained iron


deficiency. (SOR B)
 The differential diagnosis of microcytic anemia includes thalassemia,
sideroblastic anemias, some types of ACD, and lead poisoning.
 Prenatal vitamins with iron are recommended during pregnancy.
Supplemental iron therapy is recommended at Hb <10 g/dL.
 The regimen for management of iron deficiency in children is 3 to 6
mg/kg/day of elemental iron in divided doses for 3 months.

 thalassemias are usually characterized by an elevated RBC count & Low


reticulocyte production
 Hb electrophoresis should be ordered for any patient with suspected
thalassemia.

 Anemia as a result of chronic renal failure is the most common cause of


anemia of chronic disease
 Recombinant erythropoietin should be considered for all patients with
anemia of renal failure or patients undergoing chemotherapy

 Vitamin B12 deficiency is almost always caused by malabsorption related to


atrophic gastritis.
 There is evidence suggesting that early vitamin B12 deficiency can be
diagnosed by elevated levels of homocysteine or methylmalonic acid.
 Treatment of vitamin B12 deficiency requires monthly parenteral
treatment of vitamin B12 in doses of 100–1000 mcg, usually administered
daily or every other day for the first few weeks, followed by maintenance
doses every 1–3 months. Once vitamin B12 levels have been reestablished,
oral therapy (1 mg/day) can then be substituted. Treatment often also
consists of concurrent administration of folate, 1–5 mg each day
 Patients with confirmed vitamin B12 deficiency from PA must receive
vitamin B12 therapy for the duration of their lives. Intramuscular dosing is
1000 µg/d for the first week, then 1000µg weekly until hematologic values
normalize or for at least 6 months if neurologic complications exist, then
1000 µg monthly for life.

 The most common cause of folic acid deficiency is inadequate intake.


 Drugs such as anticonvulsants and oral contraceptives also predispose to
folate malabsorption. Folate antagonists include certain chemotherapeutic
agents, antiviral drugs (e.g., azidothymidine [AZT] and zidovudine),
methotrexate, trimethoprim, nitrous oxide, primidone, and phenobarbital.
Individuals on methotrexate should be supplemented with 1 mg of folic
acid daily
 Pregnancy, hemolytic anemia, psoriasis, dialysis, alcoholics  folate
deficiency
 With folate deficiency serum and RBC folate levels are low, In cases where
diagnosis remains in doubt, an elevated homocysteine level, in spite of a
normal methylmalonic acid level, suggests folate deficiency.

47
iSummary SHORT NOTES

 You must exclude B12 deficiency before replacing folate, because folate
replacement can reverse the anemia but will permit progression of
neurologic effects of B12 deficiency
 Supplementation with oral folic acid—from 1 to 5 mg daily—is used to treat
deficiency. Total correction occurs within 6–8 weeks
 Special tests can also be used to evaluate patients for HS. The osmotic
fragility test
 The peripheral blood smear in G6PD deficiency shows characteristic Heinz
bodies

 Sickle cell anemia  Folic acid should be supplemented, 1 mg orally every


day. ● Patients with sickle cell disease should be immunized against
Streptococcus pneumoniae, Haemophilus influenzae, hepatitis B, and
influenza according to CDC. ●Antibiotic prophylaxis with penicillin should
be given to children between the ages of 2 months and 5 years because of
the high risk of pneumococcal infection. A typical regimen is oral penicillin,
125 mg twice daily until age 2 to 3, then 250 mg twice daily. ●Regular
ophthalmologic examination should be done every 1 to 2 years beginning
at age 10 years because of a high incidence of retinopathy
 Pulmonary complications are the most common causes of death in patients
with sickle cell disease.
 Sickle cell anemia  The peripheral blood smear shows sickling of half of
the RBCs (the gold standard diagnosis). Howell–Jolly bodies and target cells
are also present on the smear
 Patients with sickle cell anemia should have their hemoglobin and
hematocrit levels maintained at a moderately low level because this is
protective against some of the vaso-occlusive complications related to
viscosity
 The use of hydroxyurea promotes increased levels of fetal hemoglobin, and
fetal hemoglobin is associated with decreased frequency and severity of
pain crises

 A positive direct Coombs test helps diagnose autoimmune hemolytic


anemia. Corticosteroids are often considered the treatment of choice,
especially when autoantibodies are warm

 Bleeding disorders

 Bleeding time will be prolonged (more than 7 minutes) in qualitative


platelet dysfunction, thrombocytopenia, and von Willebrand disease
 Prothrombin time (PT) and activated partial thromboplastin time (aPTT)
evaluate coagulation factors
 Hemophilia A is an X-linked disease due to deficiency of coagulation
factor VIII. ● Laboratory findings are prolonged aPTT and decreased factor
VIII assay ● Administer purified or recombinant factor IX concentrate
during acute bleeding or prophylactically prior to invasive procedures (or

48
iSummary SHORT NOTES

fresh frozen plasma [FFP] if specific factor concentrates are unavailable). ●


chronic prophylactic Cryoprecipitate, Desmopressin acetate, Purified IV
human and porcine factor VIII concentrates

 Hemophilia B is an X-linked bleeding disorder due to deficiency of


coagulation factor IX. ● Laboratory findings are prolonged aPTT and
decreased factor IX assay. ● Administer purified or recombinant factor IX
concentrate during acute bleeding or prophylactically prior to invasive
procedures (or fresh frozen plasma *FFP+ if specific factor concentrates are
unavailable)

 Vitamin K deficiency●Vitamin K deficiencies may occur with warfarin use,


postsurgical states, antibiotic therapy, biliary obstruction, liver disease,
nutritional deficiencies, and malabsorption syndromes, such as
inflammatory bowel diseases and ingestion of nonabsorbed fat substitutes
in diet foods. ● The PT is prolonged. The aPTT may be prolonged if the
deficiency is severe. ● Mild deficiencies may be corrected with vitamin K, 10
to 20 mg PO. ● Severe bleeding should be managed by transfusion of
FFPalong with vitamin K administration
 liver disease The PT and aPTT are prolonged ● Vitamin K 10 to 20 mg SC
can be tried ● FFP may transiently improve hemostatic function for the
actively bleeding patient
 ●Von Willebrand disease The most common inherited bleeding disorder
●↓ plasma vWF antigen, ↓ plasma vWF activity, ↓ factor VIII activity, ↑ or
normal PTT, and ↑ bleeding time ● Oral contraceptives may be given to
women. They mimic the hormonal changes of pregnancy, increasing vWF
and factor VIII levels ● Desmopressin (DDAVP) increases vWF
concentrations two- to fivefold ● Cryoprecipitate or factor VIII concentrate
in severe disease

 ITP ● Most cases of acute ITP resolve spontaneously within 6 months. ●


Prothrombin time and partial thromboplastin time are normal, but bleeding
times are increased. Most patients can simply be observed. The only
indication for treatment is bleeding. ●In more severe cases with platelet
counts below 20,000 mm3 or with active bleeding, corticosteroids, γ
globulin, or other immunosuppressant agents that transiently elevate the
platelet count can be used, but they do not alter the course of the illness
 Chronic ITP ● Initial treatment is traditionally prednisone

 Leukemia & lymphoma

 Leukemia  ●Peripheral smear: ↑ circulating blasts as well as anemia and


thrombocytopenia ● Bone marrow: ↑ blasts
 Acute lymphoblastic leukemia (ALL) the most common childhood
malignancy with a peak incidence at age 2 to 5 years. Characteristic
appearance of the bone marrow aspirate with >20% lymphoblasts

49
iSummary SHORT NOTES

 Acute myelogenous leukemia (AML)Characteristic appearance of bone


marrow aspirate with >20% myeloblasts
 HODGKIN’S LYMPHOMA ●A painless, enlarged, rubbery lymph node
(70% of patients - most common presenting complaint). ● Excisional lymph
node biopsy provides definitive diagnosis

51
iSummary SHORT NOTES

RHEUMATOLOGY
 Rheumatoid arthritis ●The early use of DMARDs is beneficial, it should be
initiated within 3 months of diagnosis ● Methotrexate is typically used as first-
line treatment in cases of moderate or severe RA or in addition to another
DMARD in refractory cases of mild RA
 In methotrexate Monitor complete blood count (CBC) and liver-associated
enzymes every 4 to 8 weeks. Supplement with folic acid, 1 mg per day ● Women
and men on methotrexate should use an effective form of contraception, and
continue contraception for 3 months after stopping methotrexate.
 ●Annual eye exam to assess for hydroxychloroquine-related retinal toxicity.
●Hand x-rays are recommended at 2-year intervals

 SLE  Antibodies to double-stranded DNA (anti-ds DNA) or the Smith nuclear


antigen (Anti-Sm) are highly specific for SLE. ANA testing is close to 100%
sensitive but is not specific, whereas antibodies to dsDNA and Smith are specific
but not sensitive (can be confirmatory if positive)

 In order to diagnose fibromyalgia, 11 of 18 trigger points must be tender,


symptoms must be present for at least 3 months, and other rheumatic
conditions must be excluded. ● Pain is the cardinal symptom and is widespread,
the pain must be above and below the waist, on both sides of the body, and
along the axial skeleton

51
iSummary SHORT NOTES

NEUROLOGY
Disease Management
Migraine ●Specific  Naratriptan PO, Rizatriptan PO, Sumatriptan SC, IN, PO,
Zolmitriptan PO, DHE
●Non-specific Acetaminophen + aspirin + caffeine PO, Aspirin
PO, Butorphanol IN, Ibuprofen PO, Naproxen sodium PO,
Prochlorperazine IV
Migraine ●Medications of high efficacyamitriptyline (and likely other
Prophylaxis tricyclics), divalproex sodium, and propranolol/timolol.
●Medications of lower efficacy  atenolol/metoprolol/nadolol,
nimodipine/verapamil, aspirin/naproxen/ketoprofen, fluoxetine,
ACE inhibitors, gabapentin, feverfew, magnesium, and vitamin B2
Tension ● Aspirin, acetaminophen, or NSAIDs
Headache
Chronic ● Tricyclic antidepressant ● Calcium channel blockers and β-
tension blockers
headaches
Cluster ●100% oxygen at 6 L/min, DHE, intranasal lidocaine, butorphanol
Headache and the triptans (sumatriptan)
●Prophylaxis Verapamil, ergotamine, lithium, divalproex
sodium, methysergide, topirimate, indomethacin, beta blockers,
TCAs, SSRIs, and
Temporal ● High-dose prednisone ● Aspirin ↓ the risk of stroke or visual
arteritis loss
Rebound ● Discontinue the analgesics causing the rebound headaches.
headache ●Acute attackduring withdrawal with triptans or ergotamines.
●ProphylaxisTCAs, SSRIs, β-blockers, anticonvulsants.

 The Glasgow Coma Scale

Eye opening Spontaneous 4 To pain 2


To voice 3 None 1
Best verbal response Oriented 5 Incomprehensive 2
Confused 4 None 1
Inappropriate words 3
Best motor response Obeys command 6 Flexion 3
Localizes pain 5 Extension 2
Withdraw 4 None 1
Total score = Eye opening + best verbal + best motor response scores

52
iSummary SHORT NOTES

 Seizure

 The most common cause of seizures is cerebrovascular disease


 In adults with a first seizure, between 30% and 60% will go on to have a
second seizure. In patients who have a second seizure, the likelihood of
going on to have a third is 80%–90%; therefore, after a second unprovoked
seizure, treatment is recommended.
 During pregnancy, the drug that has controlled the seizures should be
continued. If the patient has been seizure free, an attempt to stop
medication is warranted. Folic acid and vitamin D supplements may reduce
birth defects and vitamin K in the month prior to delivery can reduce
neonatal hemorrhage.
 A consideration of discontinuation of medication can be made after a
seizure-free period of 2 to 4 years. This decision should be confirmed by a
lack of seizure activity on EEG
 It is not recommended that women discontinue AEDs when pregnant, but
simplifying to monotherapy is recommended if possible
 For grand mal seizures, valproic acid seems to be the drug that is most
effective and thus the best drug to start with. For partial seizures (eg,
temporal lobe epilepsy), lamotrigine seems to be the best drug to start with
 STATUS EPILEPTICUS A neurologic emergency consisting of continuous
seizure activity lasting > 10 minutes or repetitive seizures lasting > 30
minutes without a return to a baseline neurologic level between seizures ●
For adults, and as a second choice for childrenphenytoin ● the drug of
choice in children and may be used in adults Lorazepam

 Headache

 Neuroimaging should be considered in patients with non-acute headache


and an unexplained abnormal finding on neurologic examination
 The first-line of treatment for a moderate to severe migraine headache
accompanied by vomiting in the absence of contraindication is
subcutaneous sumatriptan. If sumatriptan fails, intravenous
dihydroergotamine and intravenous metoclopramide is an underused but
highly effective treatment
 Serious reactions to triptans include coronary vasospasm, anaphylaxis, or
hypertensive crisis in patients with known CAD, hypersensitivity to triptans,
or uncontrolled hypertension.
 Contraindication to triptans agonists: IHD/angina pectoris, previous MI,
uncontrolled hypertension, basilar artery migraine, hemiplegic migraine,
and concomitant use of MAO inhibitors, SSRIs, or lithium
 Avoid narcotics for the treatment of migraine headaches
 Tension headacheAvoidance of habituating, caffeine-containing over-the-
counter or prescription drugs as well as butalbital-, codeine-, or ergotamine-
containing preparations (including combination products) is recommended

53
iSummary SHORT NOTES

given the significant risk of developing drug dependency or medication-


overuse headache

 ISCHEMIC STROKE Ischemic lesions do not usually appear in non-contrast CT


during the first 24–48 hours after stroke ●Potential tPA candidates can receive
tPA within three hours of initiation of stroke symptoms. Head CT must show no
hemorrhage, and bleeding disorders must be ruled out ● If the patient is not a
tPA candidate and has no bleeding disorder, start antiplatelet therapy. In
general, unless systolic blood pressure remains above 220 mmHg or diastolic
pressure remains above 120 mmHg, elevated blood pressure should not be
treated within the first 48 to 72 hours after an acute ischemic stroke
 Kernig’s sign (passive extension of a flexed knee producing pain and resistance)
and Brudzinski’s sign (passive flexion of the neck causing flexion of the hips and
knees)  Meningeal signs: K is for Kernig’s and Knee. Remember, you move the
Knee to test for Kernig’s
 If the CT is normal but Subarachnoid hemorrhage is still suspected, obtain an LP
to examine CSF for xanthochromia or blood. Gradually treat mean arterial BP >
130 mmHg but maintain SBP > 120 mmHg
 TRANSIENT ISCHEMIC ATTACK Abrupt onset of focal neurologic symptoms
lasting only 5–20 minutes and < 24 hours. ●Anticoagulation: For cases with a
cardiac source of embolization. Rule out coagulopathy. Start IV heparin and
switch to warfarin on an outpatient basis

 GUILLAIN-BARRÉ SYNDROME symmetrical, rapidly progressive, ascending


muscle weakness that usually starts in the legs.● Campylobacter jejuni
gastroenteritis is the most frequent antecedent infection plasmapheresis or
immunoglobulin
 MYASTHENIA GRAVIS muscle weakness that worsens with activity and
improves with rest Acetylcholinesterase inhibitors, Thymectomy

54
iSummary SHORT NOTES

55
iSummary SHORT NOTES

OBESITY
Underweight: BMI <18.5 Obesity Class I: BMI 30.0 to 34.9
Normal: BMI 18.5 to 24.9 Obesity Class II: BMI 35.0 to 39.9
Overweight: BMI 25.0 to 29.9 Obesity Class III: BMI ≥40

 Paroxetine and tricyclic antidepressants, monoamine oxidase inhibitors, and


mirtazapine are most likely to cause weight gain
 valproic acid and gabapentin have been most closely tied to weight gain.
Lamotrigine is thought to be weight-neutral whereas topiramate may actually
promote weight loss
 Initially set a 4- to 6-month goal for weight reduction of 10% or a reduction of
BMI by two units.
 Orlistat use by patients with diabetes has been demonstrated to reduce weight
and decrease hemoglobin A1c over placebo. There can be decreases in the
absorption of fat-soluble vitamins, so it is recommended that people taking
orlistat take a daily multivitamin.

BMI 18≤25, no Healthy diet and physical exercise


adjusted risk:
BMI 25≤27, As above plus low calorie diet
moderate risk:
BMI 27≤30, high As above
risk:
BMI 30≤35, very As above plus medication (Orlistat )and very low
high risk: calorie diet
BMI 35≥40, As above and consider surgical interventions when
extremely high risk: obesity-related comorbidities are present
BMI 40+, extremely As above plus surgical interventions
high risk:

56
iSummary SHORT NOTES

INFECTIONS
Disease Management
HIV ●Nucleoside reverse transcriptase inhibitors (NRTIs):

● Non-nucleoside reverse transcriptase inhibitors (NNRTIs)

● Protease inhibitors (PIs)

● Fusion inhibitors
Prophylaxis Against AIDS-Related Opportunistic Infections
Pneumocystis ●If CD4 count < 200/mm3 or a ● TMP-SMX or dapsone +/−
jiroveci cystic history of oral thrush. pyrimethamine or
pneumonia (PCP) ● Prophylaxis may be stopped pentamidine nebulizers or
if CD4 > 200 for ≥ 3 months on atovaquone.
HAART.
Mycobacterium ● CD4 count < 50/mm3. ● Azithromycin or
avium complex ●Prophylaxis may be stopped clarithromycin or rifabutin.
(MAC) if CD4 > 100 for ≥ 3 months on
HAART.
Toxoplasma ● CD4 count < 100/mm3 and TMP-SMX or dapsone
●Toxoplasma IgG positive. −/+pyrimethamine or
Prophylaxis may be stopped if atovaquone
CD4 > 100–200 for ≥ 3 months
on HAART.
Rabies ●1% Lidocaine through intact skin
● Copious wound irrigation with 500 cc normal saline
● Wound closure ●by Secondary Intention if: Puncture
Wounds, Infected, >24 hours ● Sutured closure if: Wound <8
hours old, located on face
● Tetanus Vaccine
● Human Rabies immune globulin + Rabies Vaccine (0, 3, 7,
14, 28)
● Antibiotic1. Amoxicillin-Clavulanate 7 days if no
cellulitis(14d)
Encephalitis ● Supportive measures
● Empiric IV acyclovir  in suspected cases of encephalitis
without an obvious source
Syphilis ● Penicillin given in the long-acting benzathine IM
Brucellosis ● Doxycycline + streptomycin, rifampin or gentamicin. For ≥
6w
Cholera ● Fluid and electrolyte replacement
● Antibiotics●Adult Tetracycline, Ciprofloxacin,

57
iSummary SHORT NOTES

doxycycline, erythromycin ● Child erythromycin, TMP-SMX,


Furazolidone
Typhoid fever ● Antibiotics1. Fluoroquinolones if resistance 
Chloramphenico, Amoxicillin, TMP-SMX
Malaria ● Mefloquine ● Pyrimethamine with sulfadoxine ● Primaquine
Giardiasis ● 1.Metronidazole ● 1.Tinidazole ● Furazolidone
● 2.albendazole and mebendazole ● 2.nitazoxanide
● Paromomycin  in pregnancy
Amebiasis ● Symptomatic metronidazole, tinidazole ● If failure
iodoquinol, or paromomycin
● Asymptomatic intraluminal infection iodoquinol, or
paromomycin ● Nitazoxanide (no clinical trials)
Enterobiasis ● Single dose of mebendazole, Some authorities recommend
a second course of treatment in 2 weeks
● Alternatives albendazole, pyrantel pamoate 
pregnancy
● Most authorities recommend treating other family members
or close household contacts
Toxoplasmosis ● Most infections in immunocompetent and nonpregnant
patients no antimicrobial therapy
● Acute infection pyrimethamine + sulfadiazine+ folinic
acid
● Corticosteroid may be added for patients with chorioretinitis
and congenital infection
● Clindamycin, may be substituted for the sulfadiazine in cases
of allergy or intolerance
● Suspected primary infection during pregnancy
Clarithromycin and azithromycin
●Confirmed primary infection during pregnancy and after the
18th week of pregnancy pyrimethamine + sulfadiazine+
folinic acid for 3 weeks and then alternated every 3 weeks
with the spiramycin till delivery
Tapeworm ● e.g.,Taenia solium, Taenia saginata, Hymenolepis nana,
Diphyllobothrium latum
● Intestinal tapeworm praziquantel, Albendazole,
nitazoxanide  the entire family should be treated
● Invasive tapeworm infection Albendazole
Schistosomiasis ●Intestinal schistosomiasis: S. mansoni
●Urinary schistosomiasis: S. haematobium
●Acute schistosomiasis Praziquantel
Ancylostoma ●Mebendazole, albendazole, pyrantel pamoate
duodenale

58
iSummary SHORT NOTES

 HIV

 HIVMarkers used to measure progression are CD4 count and HIV RNA
viral load. The CD4 count measures the degree of immune compromise and
predicts the risk of opportunistic infections. Viral load measures HIV
replication rate, gauges the efficacy of antiretrovirals, and predicts long-
term progression ●ELISA/enzyme immunoassay (EIA) to diagnose HIV by
detecting antibodies to the virus in serum. In most cases, ELISA will be
positive three months after infection. Roughly 95% of infected patients will
be positive by six months. Tests must be confirmed by Western blot ● CD4
cell count should not be used for screening
 The current recommendation start antiretroviral treatment in all patients
who are symptomatic. Treatment of asymptomatic patients should be
started when the CD4 count is 200–350 cells/mm3. Start HIV treatment in
patients with a viral load of > 55,000 copies/mL. ● Typical regimens use
three drugs, generally consisting of two NRTIs plus one PI or NNRTI
 During pregnancytwo NRTIs (including AZT) plus nevirapine or a PI.
Consider starting after 10–14 weeks of gestation to minimize the risk of
teratogenicity. Efavirenz is contraindicated during pregnancy
 Postexposure prophylaxis (PEP)  Combivir (zidovudine plus lamivudine)

 Meningitis

 Once you suspect bacterial meningitis, rapid diagnostic evaluation and


emergent treatment are imperative, including lumbar puncture and blood
cultures. If lumbar puncture is going to be delayed, give appropriate
empiric antimicrobial and adjunctive therapy without delay ●All patients
should receive a third-generation cephalosporin. In patients > 1 month of
age, vancomycin should be added to cover possible penicillin resistant S.
pneumoniae infection. Ampicillin should be added in very young, elderly, or
immunocompromised patients to cover L. monocytogenes. Consider
corticosteroids as adjunctive therapy
 Prophylaxis  Rifampin can be used for all age groups for prophylaxis;
ciprofloxacin is acceptable treatment for adults older than 18 years.
Ceftriaxone can also be used for prophylaxis especially in pregnant

 Syphilis

 Syphilis● Primary syphilis chancre ● Secondary syphilis rashes may


last up to 1 year ● Latent syphilis may last for many years, asymptomatic
but positive treponemal antibody test ● Late or tertiary syphilis occurs
10 or more years after the initial infection, systemic involvement
 Screening tests for syphilis include a VDRL test and the rapid plasma reagin
test. If positive, a confirming fluorescent treponemal antibody absorption
test should be performed

59
iSummary SHORT NOTES

 Brucellosis Blood Culture (70% sensitive in acute illness), Discharge Culture,


Bone Marrow Aspirate (90% sensitive in acute illness), Brucella Serology
 Typhoid fever Relative Bradycardia , Rose Spots ●Blood Culture: Best Test
Sensitivity in first week, Salmonella serology (Widal's Test): Poor Test
Specificity, Low Test Sensitivity (70%)
 Malaria  Peripheral blood smear with Giemsa or Wright stain, Rapid blood
dipstick testing
 Giardiasis  microscopic detection of G. lamblia trophozoites, immunoassays
for G. lamblia antigen in stool specimens
 Amebiasis stool specimens
 Enterobius vermicularis adhesive tape on the skin and then examining it
under a microscope. Eggs are rarely laid in the intestine, so inspection of fecal
specimens is not useful

 Toxoplasmosis  the most common presentation is self-limited focal


nontender adenopathy in the head and neck region ● IgM is detectable 2 weeks
after infection and usually becomes negative in 6 to 9 months but may remain
elevated for several years ● Immunoglobulin (Ig) G remains elevated for life;
hence a positive test does not distinguish acute from past infection. ● CBC and
platelet counts need to be monitored for marrow suppression once or twice a
week while the patient is taking daily pyrimethamine ● Newborns with
congenital Toxoplasma infection should get a year of treatment

 Schistosomiasis Serologic assay for schistosomal antibodies OR antigens,


Stool examination ova ● Eosinophilia

 Ancylostoma duodenale Anemia, Microscopy for Ova and Parasites,


●Eosinophilia

 Recommendations for Tetanus Prophylaxis


Clean minor All other
wounds wounds*

History of tetanus Td TIG Td TIG


prophylaxis
Uncertain or less than 3 Yes No Yes Yes
doses
3 doses or more
Last dose within 5 years No No No No
Last dose within 5 to 10 years No No Yes No

61
iSummary SHORT NOTES

 VACCINATION PROPHYLAXIS (animal bites)

 TETANUS
 It is important to determine the patient’s tetanus vaccination status.
 Patients who have completed the primary three-shot regimen will need
a booster shot if it has been 10 or more years since their last shot.
 A wound that is large and dirty needs a booster if it has been longer
than 5 years since their last one.
 If the patient has not completed the primary three-shot series, that
series should be started when the patient presents for treatment. If the
wound is small and clean, beginning the vaccination series is adequate.
If the wound is large and/or dirty the patient should be given be given
human tetanus immune globulin (250 units for adults and children). The
vaccine and immune globulin should be administered at separate sites
that are distant from the other so that they do not interfere with each
other.
 If the patient has not had a primary series of tetanus immunizations,
administer tetanus immune globulin and start the primary tetanus
series.

 RABIES
 Rabies prophylaxis includes both vaccination and human rabies immune
globulin (RIG).
 If a patient has been previously immunized against rabies, a single
booster injection is adequate. If there is no history of vaccination and
the patient has a high-risk bite, then the patient is given the vaccine on
days 0, 3,7,14, and 28. Additionally the patient should be given RIG as a
single dose of 20 IU per kg. As much of the RIG as possible is infiltrated
around the wound, and the rest injected IM at a site distant from the
vaccine.
 The difficult part is determining which bites need rabies prophylaxis.
The best source for an answer to this is the local public health
department. Some basic guidelines include bites from any dog, wild
animal, or bat that is not available for observation. If the animal is
available, prophylaxis can be held for up to 10 days to allow for
observation and examination to determine if the animal was carrying
rabies. One absolute indication is for people who have been in a room
with a bat and it cannot be determined if a bite, scratch, or mucous
membrane exposure has occurred (Advisory Committee on
Immunization Practices recommendation). This includes small children,
the disabled, intoxicated persons, and anyone who awakens to find a
bat in the room. Bats have very small sharp teeth and it can be very
difficult to determine if a bite occurred.
 All bats should be considered rabid unless available for observation and
testing.

61
iSummary SHORT NOTES

 The appropriate treatment of heat exhaustion/heat stroke (heat stroke being


defined as CNS dysfunction with a change in the level of consciousness) is cool
water-soaked blankets and towels with fans aimed at the patient. This allows
evaporative cooling and also conductive heat loss (to the water in the towels).
Antipyretics are generally not effective because by this point the patient’s
endogenous thermoregulation is kaput. ● Packing patients in ice is
contraindicated. Total body immersion in ice water is useful but packing the
person in ice actually reduces cooling for two reasons.

62
iSummary SHORT NOTES

GERIATRIC
 Pressure ulcers

 To prevent pressure ulcersschedule regular and frequent repositioning for


bed and chair-bound individuals. Turn at least every 2–4 hours on a pressure-
reducing mattress or every 2 hours on a nonpressure reducing mattress. Also,
maintain the head of the bed ≤30 degrees to reduce pressure on the sacral area
 Constipation  Dietary and behavioral changes constitute the first-line
approach to correcting constipation. Increasing fiber is a good first-line approach
and should be encouraged. The daily recommended fiber intake is 20–35 grams
● Laxatives are third-line therapy and include lactulose and sorbitol  chronic
constipation. Stool softeners (e.g., docusate) on opiates or who are
bedridden. Stimulant laxatives (e.g., senna, cascara) short-term use. Enemas
fecal impaction is present
 Falls ● Older people should have their risk of falls assessed periodically (e.g.,
annually) (SOR A) ● the most useful assessment of integrated function is the Get
Up and Go test. Completion of the test in <10 seconds represents no risk and
can be expected from nonfrail elders. ● Vitamin D deficiency has been
associated with an increased risk of falls, and empiric supplementation can
reduce the risk

 Incontinence

 Transient factors inadequate mobility, motivation, or dexterity;


confusion; and medications (diuretcs, autonomic agents, psychotropics).
Other causes include delirium, infection, inflammation, impaction and
polyuria (diabetes, volume overload)
 post-void residual volume (PVR)Normally, <50 mL of residual urine is
present after voiding. Volumes of >200 mL are abnormal. Intermediate
volumes (50 to 200 mL) are considered equivocal, and the test should be
repeated
 α-adrenergic antagonists (e.g., terazosin) intrinsic sphincter
deficiencystress incontinence
 Overflow incontinence can be caused by medications that relax the bladder
detrusor muscle (e.g., anticholinergic agents, calcium-channel blockers.
Discontinue anticholinergics and narcotics that may → bladder relaxation;
stop decongestants such as pseudoephedrine that may ↑ sphincter tone

63
iSummary SHORT NOTES

Type Symptoms/sig Investiga Management


ns tions
Urge ● Most common ●1. PVR, if ● Kegel exercises
incontinence type >200, ● Bladder training
● Urge to urinate proceed to treatment of choice
but cannot get to ultrasound ● Anticholinergics
the toilet in time ,
● Nortriptyline Check
ECG first, as it may worsen
QT-interval prolongation
● Topical estrogen
Stress ● Loss of urine ● PVR > 450 ● Kegel exercises
incontinence during a stress ● α-adrenergic agonists
maneuver (e.g., 
sneezing, coughing,
laughing) ● Duloxetine
● Estrogen
● Surgery is an effective
treatment for women with
pelvic prolapse
Overflow ● Unpredictable ● Acute urinary retention
incontinence dribbling of urine indwelling or intermittent
or weak urine catheterization
stream ● α-blockers Prazosin or
terazosin

 Dementia

 The Mini-Mental Status Exam is the best instrument for screening for
dementia ● age is the greatest risk factor for developing AD ● In order to
diagnose dementia, impairment in memory must be present ● In general,
neuroimaging is recommended if dementia occurs in the following
scenarios: onset <65 years, sudden onset, presence of focal neurologic
signs, and suspicion of normal pressure hydrocephalus ● Vitamin E may
slow progression of Alzheimer’s, Estrogen and NSAIDs have shown no
benefits
 Lewy body dementia parkinsonism, cognitive fluctuations, autonomic
dysfunction, visual hallucinations
 Frontotemporal dementia younger age group. Focal atrophy of frontal
and temporal lobes is a characteristic finding, Inappropriate social behavior,
Language dysfunction and behavioral abnormalities
 Normal pressure hydrocephalus (NPH) dementia, gait ataxia, and
urinary incontinence

64
iSummary SHORT NOTES

 Parkinson's disease ●Levodopa/carbidopa: Most effective medication for PD.


Dopaminergic agonists may be preferable for younger patients. For older
patients, carbidopa/levodopa is preferred

65
iSummary SHORT NOTES

OBSTETRIC
 Medical disorders during pregnancy
Disease Management
Nausea and vomiting ● Lifestyle changes ● Ginger
of pregnancy ● Vitamins containing folate
● Pyridoxine (vitamin B6)± doxylamine
● Antiemetics  metoclopramide or promethazine
● Antihistamines, anticholinergics, and corticosteroids
Hyperemesis ● Lifestyle changes ● Ginger
gravidarum ● Antiemetics vitamin B6, doxylamine, H2 blockers,
promethazine, dolasetron
●IV/enteral fluids and nutrition if severe dehydration
and/or > 5% weight loss
●Corticosteroid
Physiologic bleeding ●Pelvic exam ●Ultrasound ●Expectant management
(1st trimester)
Spontaneous ●Pelvic exam ●Ultrasound ●serial serum β-hCG (if
abortion passage is uncertain)
● D&Cprolonged or heavy bleeding
Threatened abortion ●Pelvic exam ●Ultrasound and/or β-hCG
● Expectant management
Inevitable abortion ●Pelvic exam; removal of tissue at the os may stop
bleeding.
●D&C is indicated for significant cramping or blood loss;
otherwise ● Treat with expectant management.
Septic abortion ●Stabilize with IV fluids; ●Broad-spectrum antibiotics;
● D&C. ● Elective abortion or other invasive procedures.
Missed abortion ● Ultrasound to confirm diagnosis; ● choose between
(“blighted expectant management × 2 weeks (15–75% will pass
ovum”) spontaneously), medical management with misoprostol,
or D&C/D&E for prolonged retention/unsuccessful
medical management/patient preference.
Leg cramps ● Magnesium lactate or citrate
(pregnancy) ● Stretches, Regular exercise and supportive stockings

Heartburn (GERD) ●Nonpharmacologic


●Only for refractory symptoms Antacids, histamine
antagonists (B), PPI (C)
Iron deficiency ●60 to 120 mg of elemental iron
anemia
Asthma ●Similar to that in the nonpregnant asthmatic woman
Transient gestational ● Methyldopa
hypertension ●If not controlled add nifedipine or hydralazine

66
iSummary SHORT NOTES

● Labetalol/
Preeclampsia ●Mild preeclampsia  ●Bed rest, labetalol,
hydralazine, or nifedipine. ●During labor and delivery
intravenous magnesium sulfate for seizure prophylaxis.
●Severe preeclampsia● Immediate delivery
●corticosteroid for lung maturity (48 delay) ●
hydralazine or labetalol; give MgSO4 ● Postpartum
therapy with magnesium sulfate for 12:24 hours
Eclampsia ● MgSO4 ● benzodiazepines
HELLP syndrome ● Corticosteroids ● Hypertensive medications ● platelet
transfusion if platelet count is < 20,000 or there is
maternal bleeding
● Delivery at > 34 weeks of gestation or in the setting of
nonreassuring fetal status or severe maternal disease
Gestational diabetes ● Initial therapy diet, exercise
● If fasting blood glucose cannot be maintained below 105
mg/dL and 2-hour postprandial blood glucose below 120
mg/dL insulin therapy
● Glyburide

Asymptomatic ● Cephalosporins, ampicillin, and nalidixic acid


bacteriuria
Pyelonephritis ● Hospitalization IV antibiotic (cephalosporin or an
extended spectrum penicillin, addition of an
aminoglycoside ( ) for severe cases). ●Oral
antibiotics are then prescribed for 7 to 10 additional days.
Hyperthyroidism ●1. Propylthiouracil ● Methimazole ● β-blockers
●Thyroidectomy may be considered after the first
trimester
Hypothyroidism ● Levothyroxine
Cholestasis of ●Synthetic bile acidscholestyramine; hydroxyzine for
pregnancy symptomatic relief
●Deliver by 38 weeks or sooner if severe and fetal lung
maturity is established
Toxoplasmosis ● Pyrimethamine and sulfadiazine
Syphilis ● Penicillin
Rubella ● Pregnancy termination offered
Herpes ● Acyclovir (controversial)
HIV ●Same as non-pregnant women ● Zidovudine
Group ß ●IV penicillin G/Ampicillin before delivery (Cefazolin,
streptococcal Clindamycin, Erythromycin) if allergy
infection
Active management ● Oxytocin or ● Misoprostol and ergotamine
of 3rd stage of labor
Induction of labor ● Prostaglandin E2 gel is administered vaginallycervical
ripening

67
iSummary SHORT NOTES

● Oxytocin labor induction


● Misoprostol cervical ripening and labor induction
Cord prolapse ●Immediate delivery Cesarean delivery preferred if the
cervix is not fully dilated
● Vacuum or forceps if the cervix is completely dilated
Postpartum ●Oxytocin: It should never be administered as an
hemorrhage undiluted IV bolus, due to the risk of hypotension and
cardiac arrhythmia.
●Methylergonovine maleate (Methergine) use
cautiously if hypertensive
●Carboprost tromethamine (Hemabate)Caution
should be used in women with asthma, as Hemabate can
cause bronchoconstriction.
●Misoprostolcan be administered to women with
asthma or hypertension.
● IV fluids or blood products (packed red blood cells) if
necessary.

Disease Symptoms/Sign Diagnosis Treatment


s
Ectopic ●Abdominal pain with ●Positive β-hCG ● IM
pregnancy spotting, usually ●Serial, quatitative methotrexate if
occurring 6 to 8 weeks serum β-hCG testing small (<3.5-cm mass),
after the LMP ●Abdominal/transvag <8w, no rupture
inal ultrasonography ● Laparoscopic
injection of
methotrexate
● Surgical removal
Hydatidiform ●Vaginal bleeding ●Ultrasound ●Immediate uterine
molar +/− anemia; pelvic ●β-hCG is ↑. evacuation
pregnancy pressure or pain; ●Tissue pathology serial β-hCG
hyperemesis
gravidarum;
hyperthyroidism;
and early-onset
preeclampsia.

Placenta ●Asymptomatic, or ●Ultrasound ●If no bleeding


previa ●Painless vaginal Complete pelvic
bleeding late in the rest until resolution
second or third of the previa is
trimester confirmed on serial
ultrasounds or until
delivery.
●If actively
bleeding Admit
and stabilize the

68
iSummary SHORT NOTES

mother; immediate
C-section delivery if
fetal/ maternal status
is nonreassuring.
●If bleeding
resolves
Conservative
inpatient
management with
bed rest,
corticosteroids, and
serial ultrasounds; C-
section once fetal
lung maturity is
confirmed at
approximately 34
weeks.
Placental ●Constant severe ●Diagnosis is clinical; ●Stabilize the mother
abruption uterine contractions ultrasound is (IVs, type and cross,
+/− vaginal bleeding insufficiently sensitive transfuse PRN);
(80%) to rule out abruption. continuous internal
fetal monitoring.
●If nonreassuring
maternal/fetal
status Immediate
C-section.
●If term and stable:
May deliver vaginally
in the OR.
●If preterm and
stable: Inpatient
conservative
management.
Vasa previa ●Painless vaginal ● If term and/or
bleeding with ROM, unstableImmediat
●Fetal heart rate e C-section.
anomalies (a ●If preterm and
sinusoidal pattern is stableInpatient
classic). conservative
management; C-
section when fetal
lungs are mature.
Polyhydramnios ● Uterine size ● Ultrasound to ● Monitor AFI every
measuring large for evaluate amniotic 1–3 weeks
dates fluid index ● Amnioreduction 
●↓ fetal movement if symptomatic and
severe
●Give
indomethacin  if
severe and/or
preterm

69
iSummary SHORT NOTES

Oligohydramnio ● Uterine size less ●Inadequate AFI ● Maternal


s than dates measured on hydration;
●↓ fetal movement ultrasound; amnioinfusion if
●Amniocentesis for necessary for
karyotype analysis adequate
ultrasound
assessment of the
fetus.
● Biweekly AFI
measurements.
●Labor induction
at term or
earlier if there is
evidence of
nonreassuring
fetal status.

Preterm labor ●Fetal fibronectin ● Antibiotics


specimen between 24 ● Corticosteroids if
and 35 weeks prior to 34 weeks
●Bed rest
●Tocolytics 
1.magnesium
sulfate, terbutaline,
prostaglandin
inhibitors, and CCB
Premature ●Presents with ●Positive nitrazine; ●Look below
rupture of leaking or gushing ferning of amniotic
membranes of clear fluid from fluid.
the vagina

Intrauterine ●Discrepancy between ●Ultrasound fetal ●Bed rest in a lateral


growth the fundal height and growth and recumbent position
retardation the expected ●Biometry
(IUGR) gestational age measurements
serial ultrasounds
during a 2-to 4-week
period
●Doppler flow
velocimetry if
placental insufficiency
is suspected
Pruritic ● Extreme pruritus, ● Rash starts on ● Topical steroids;
urticarial with erythematous the abdomen and antihistamines
papules and papules coalescing to spreads to the
plaques of plaques within striae extremities
pregnancy
(PUPPP)

71
iSummary SHORT NOTES

 Estimated date of delivery

 Nagle's rulesubtracting 3 months and adding 7 days to the first day of


the last menstrual period. If a woman's menstrual periods are 35 days
apart, add 14 days rather than 7 days. If her periods are 21 days apart,
add 0 days rather than 7 days
 Pelvic ultrasound (the most reliable method) Biparietal diameter or
femur length (most accurate at 12–15 weeks)
 The most accurate ultrasound measurement is a crown-rump length
performed between 9 and 12 weeks, which is accurate to within 3–5
days.

 Nutrition & Weight gain in pregnancy

 Supplementation of 30 mg of elemental iron given in the form of simple


salts such as ferrous sulfate, ferrous gluconate, or ferrous fumarate is
recommended.

 The Centers for Disease Control and Prevention (CDC) recommend all
low-risk fertile women to take 400 μg of folic acid per day.
 Women at increased risk for offspring with NTDs should take higher
prepregnancy doses (4 mg/day): personal or family history of neural tube
defect, maternal insulin-dependent diabetes, and possibly adolescents.
 Women who are taking anticonvulsants with no personal or family history
of NTDs are advised to take 1 mg of folate supplementation, but there
are no large studies to confirm these dosage recommendations.

 Calcium is required for fetal skeletal development, particularly in the last


trimester. During pregnancy and lactation, elemental calcium intake
should include at least 1000 mg per day, divided in two doses taken with
food
 Vitamin D supplementation is not routinely recommended, but it has
been linked to neonatal hypocalcemia and should be considered in
women with limited exposure to sunlight; however, high doses are toxic.

 Proteins are a critical part in the fetus’s proper brain development. As


such, pregnant women are advised to ingest an additional 5 to 6 g of
protein daily above the nonpregnant state.

 The average weight gain in women with a normal BMI (19 to 24.9) is 25
to 35 pounds (11.5 to 16 kg).
 Women who enter pregnancy with BMI < 19.8 should gain a greater
amount of weight during pregnancy, e.g., 28 to 40 pounds (12.5 to 18 kg).

71
iSummary SHORT NOTES

 Overweight (BMI 25 to 29.9) and obese (BMI >30) women should be


advised to gain less weight during pregnancy e.g., 15 to 25 pounds for the
overweight (7 to 11.5 kg) and <15 pounds for the obese (<7 kg)

 Prenatal labs/studies

 hemoglobin and hematocrit anemia


 raid plasma reagin  syphilis
 Rubella antibody titer; varicella titer if unsure of history;
 Hepatitis B (HBsAg).
 A maternal serum antibody test (if Rh -ve, give RhoGAM at 28 weeks and
postpartum (at the time of delivery) to prevent Rh isoimmunization) and
blood group.
 A complete UA with culture.
 Testing for human immunodeficiency virus (HIV) is offered and highly
recommended because perinatal transmission can be decreased with
appropriate treatment.
 During the pelvic exam, a Papanicolaou (Pap) smear (if not done in the
past 6 months).

 A one-hour 50-g glucose challenge for diabetic screening in high-risk


patients.
 Purified protein derivative (PPD) screening should be offered for women
at risk of TB.
 Obtain cervical cultures for gonorrhea (GC) and Chlamydia (Chl) in high-
risk women (age <25 years; unmarried; Black, a history of STIs or STDs,
new or multiple sexual partners, cervical ectropion, and inconsistent use
of barrier contraception; and living in communities with high infection
rates).

 Screening for Down syndrome and trisomy 18 between 11 weeks 0


days and 13 weeks and 6 days HCG (human chorionic gonadotropin)
and PAPP-A (plasma-associated pregnancy protein A),nuchal thickness
(Ultrasonography at 10 to 14 weeks' gestation can measure nuchal
translucency as a screening test for Down syndrome.)

 Immunizations

 Hepatitis B, tetanus/diphtheria (if indicated) Waiting until the second


trimester of pregnancy to administer Td is considered a reasonable
precaution.
 Inactive influenza vaccine is recommended during pregnancy. It should
be administered annually between October and December to all pregnant
women who will be in the second or third trimester of pregnancy during
the influenza season. High risk groups, such as women with asthma,

72
iSummary SHORT NOTES

diabetes, and cardiovascular disease, should be given regardless of their


trimester.

 Influenza (live-attenuated), measles, mumps, rubella, and


varicellacontraindicated
 Delay pregnancy at least 1 and ideally 3 months after administration of
Varicella and rubella vaccines

 Abortion

 Threatened abortion: vaginal bleeding that occurs in pregnancy before


20 weeks of gestation (cervix closed)
 Inevitable abortion: spontaneous rupture of membranes or dilatation of
the cervix before 20 weeks of gestation.

 Complete abortion: spontaneous and complete passage of all products of


conception.
 Incomplete abortion: a spontaneous but incomplete passage of the
products of conception (fetus, placenta, or membranes).
 Missed abortion: retention of all products of conception after the death
of the fetus.
 Recurrent spontaneous abortion is a term reserved for women who have
had 3 or more first-trimester losses. This diagnosis should prompt further
evaluation for a cause.

 Women with threatened abortion are reassured somewhat that the loss
is neither imminent nor inevitable. Bed rest is encouraged, but no
evidence supports its value in prevention of miscarriage.
 Women who are D negative and at less than 13 weeks of gestation
should receive 50 μg of D immunoglobulin intramuscularly when the
abortion is diagnosed to prevent sensitization. In women beyond 13
weeks of pregnancy, 300 μg is used. This treatment can be omitted if the
father is known to be D negative also.

 Ectopic pregnancy

 It is the leading cause of maternal mortality in the first trimester and


accounts for 10% to 15% of all maternal deaths.
 IUDs do not increase the absolute risk of ectopic pregnancy. In fact, the
incidence of ectopic pregnancy is lower in IUD users than in non-IUD
users. However, if in the rare chance an IUD users does become pregnant,
the pregnancy is more likely to be extrauterine.
 If the β-hCG level exceeds 1,500 to 2,000 mIU/mL and no intrauterine
pregnancy is found with vaginal ultrasound (or 3600 mIU/mL with
abdominal US), an ectopic pregnancy should be suspected

73
iSummary SHORT NOTES

 A progesterone level <5 ng/mL is nonreassuring and a level >25 ng/mL is


reassuring.

 Molar pregnancy (hydatidiform mole)

 Serum hCG determination should be monitored every 2 weeks after


evacuation of the uterus until the value drops to nonpregnant values and
then every 1 to 2 months for 1 year.
 In addition, a chest x-ray should be performed at the time of evacuation
and 4 to 8 weeks after evacuation to check for metastasis.
 The lungs are the most common sites for metastasis. Patients should
avoid pregnancy for at least 1 year after the development of a molar
pregnancy.

 Second- and third-trimester labs/studies

 Perform a “quadruple screen” at 15–18 weeks to measure maternal


serum α-fetoprotein, beta subunit of hCG, unconjugated estriol, and
inhibin A levels (helps predict the risk of Down syndrome, neural tube
defects, and trisomy 18)
 Obtain a one-hour 50-g glucose challenge test at 26–28 weeks.
 Obtain a vaginal/rectal swab for group B streptococcus (GBS) at 35 to 37
weeks.

 Women of advanced maternal age are women 35 years or older at the


time of the anticipated birth of the baby should be given specific
counseling and offered diagnostic testing in the form of amniocentesis or
chorionic villus sampling to ascertain the well-being of their fetus.
 Genetic amniocentesis is typically performed at 14 to 20 weeks of
gestation, but it can also be done any time after 20 menstrual weeks.
 Chorionic villus sampling (CVS) can be performed at 10 to 13
menstrual weeks
 Down Syndrome Alpha-fetoprotein ↓, Estriols↓ β-hCG 
 Open Neural Tube Defect Alpha-fetoprotein 
 The most common reason for an abnormal serum AFP is an
inaccurate estimated gestational age.
 Women with abnormal screening tests should be assessed for
accurate gestational age and should be referred for genetic
counseling, targeted ultrasonography, and possible amniocentesis.

74
iSummary SHORT NOTES

 PRETERM LABOR uterine contractions → cervical change prior to 37 weeks of


gestation

 Preterm premature rupture of membranes (PPROM) PROM that occurs prior


to 37 weeks of gestation.

75
iSummary SHORT NOTES

 Medications in pregnancy

 Avoid  aspirin, NSAIDs, sulfonamide (in 3rd trimester), Tetracyclines,


ACEIs, Benzodiazepines

 Gestational and chronic Hypertension

 Gestational HTN BP ≥ 140/90 on two occasions six hours apart, with


onset after 20 weeks of pregnancy, in the absence of other causative
factors.
 Chronic hypertension hypertension precedes pregnancy or is
diagnosed before 20 weeks of gestation.
 It is generally accepted that a blood pressure higher than 130/80 mm Hg
any time during pregnancy is abnormal.
 Therapy to decrease maternal blood pressure should be instituted or
modified to keep maternal diastolic blood pressure lower than 100 to 105
mm Hg.

 Preeclampsia

 Hypertension (>90 mm Hg diastolic and >140 mm Hg systolic after 20


weeks on two separate occasions taken 6 hours apart; also blood
pressure elevations of 30 mm Hg systolic or 15 mm Hg diastolic over
baseline)
 Proteinuria (>300 mg/24 hours or 1+ on urine dipstick)

 Gestational Diabetes
 Ultrasonography to assess fetal size should be performed every 4 to 6
weeks. Hemoglobin A1c can be checked every 4 to 6 weeks
 Check a two-hour 75-g glucose tolerance test six weeks postpartum, and
check fasting blood glucose yearly thereafter.
 The two major forms of birth trauma encountered in gestational diabetes
are clavicular fractures and brachial plexus injuries.

 Screening for Gestational Diabetes


Test Abnormal Glucose Level (mg/dL)
50g glucose, 1-hour challenge
Nonfasting ≥140
100g glucose, 3 hour challenge
Fasting ≥105
1 hour ≥190
2 hour ≥165
3 hours ≥145

76
iSummary SHORT NOTES

 Women with a prior history of DVT have a 7% to 12% risk of recurrence during
pregnancy. Heparin (in regular or low-molecular-weight form) is indicated for
prophylaxis and should be started as early in pregnancy as possible. Women
receiving warfarin as maintenance therapy for DVT should be switched to
heparin before conception, because warfarin is teratogenic.

 Seizure  It is advisable to aim to use the best single agent for the seizure type
at the lowest protective level.

 FETAL HEART PATTERN


 Late decelerations uteroplacental insufficiency
 Early decelerations fetal head compression.
 Variable decelerations cord compression or other vagal response
 Absent variability: fetal sleep cycle, anesthetic medications, or fetal
acidosis
 Tachycardia (>160 beats/min) maternal fever, drugs, or fetal hypoxia.
Other less common causes of fetal tachycardia include fetal
hyperthyroidism, fetal anemia, fetal heart failure, and fetal
tachyarrhythmias.
 Bradycardia (<120 beats/min) normal in the postterm infant or
indicative of severe hypoxia, maternal systemic lupus erythematosus, or
fetal heart block.

 Epidural anesthesia

 Complication of Epidurals: epidurals may lengthen labor and result in


increased incidence of fetal malposition, increased need for oxytocics, and
increased rate of operative vaginal delivery (but not of caesarean section).
The most common complications, however, are maternal hypotension and
postdural puncture headache.
 Contraindications to spinal/epidural BK PAIN
 Blood pressure too low(uncorrected hypovolemia), Koagulopathy
(bleeding disorder), Pressure (↑ ICP), Anatomic back problems, Infection
of the soft tissue overlying the epidural injection site, No (the patient
refuses)

77
iSummary SHORT NOTES

GYNECOLOGY
Disease Management
Bleeding from ●Reassure ●change formulation? ● adding exogenous
Contraception estrogen?
Fibroids ● Gonadotropin-releasing hormone (GnRH) agonist
(preoperatrive)● OCPs● DMPA ●NSAIDs
● Myomectomypreserve fertility
● hysterectomy ● uterine fibroid embolization
Anovulatory ● Estrogen, OCPs, and cyclic progesterones
Bleeding ● Mefenamic acid, ibuprofen, and naproxen
● Hysterectomy and endometrial ablation
Dysfunctional UB ● Within 2 years of menarche expectant management
OR OCPs
● Menorrhagia OCPs, NSAIDs, levonorgestrel IUDs, luteal
phase progesterone, and danazol(most effective)
● Endometrial ablation and hysterectomy if structural
cause, failure of medical therapy
Dysmenorrhea ●1.NSAID ● oral contraceptives ●DMPA ● levonorgestrel
IUD
● Danazol or leuprolide acetate refractory causes of
secondary amenorrhea caused by endometriosis
● Calcium channel blockers ● Glyceryl trinitrate
● Terbutaline, oral guaifenesin, magnesium, thiamine,
aspirin, B12 and fish oil supplements
Premenstrual ●Life styleexercise, diet ● Multivitamins, calcium, and
syndrome magnesium ● Pyridoxine (vitamin B6) < 50mg/day
● Prostaglandin inhibitors: Mefenamic acid
●Diuretics: metolazone, spironolactone
●GnRH: nafrelin, leuprolide
●Hormonal contraception
●Bromocriptine
●Alprazolam, Buspirone
Premenstrual ●SSRIs
Dysphoric Disorder
Menopause ●Hormone Replacement Therapy
●Behavioral ● Calcium 1200 to 1500 mg with vitamin D
daily
● Androgens ● Synthetic steroids
●Antidepressants Venlafexine, fluoxetine, citalopram,
and paroxetine
● Anticonvulsants Gabapentin, combination of
belladonna and phenobarbital
● Antiadrenergic agents Clonidine
● Black cohosh

78
iSummary SHORT NOTES

Hyperprolactinemia ● Microadenomas  Cabergoline or bromocriptine,


Pergolide, transsphenoidal surgery
●Stop offending drug
Galactorrhea ●Treat underlying cause
Hirsutism ● Unnecessary in mild cases, except for strong patient
preference
● Electrology, bleaching, laser, topical cream, or mechanical
means.
● Oral contraceptives, long-acting GnRH analogues, and
insulin sensitizers
● Spironolactone, flutamide, cyproterone acetate, or
finasteride.
● Low-dose steroids in mild or late-onset congenital
adrenal hyperplasia.

Disease Diagnosis Investigation Management


Polycystic ●Primarily a clinical ●↑ Testosterone, ● Anovulation/infertility
ovarian diagnosis consisting androstenedione, Weight loss; clomiphene
syndrome of a combination of LH, estradiol, citrate +/− metformin
oligomenorrhea estrone, and ● Oligomenorrhea OCPs
(90%) and fasting insulin; an (choose the least
hyperandrogenism. ↑ ratio of LH to androgenic progestins;
●Amenorrhea (40%), FSH (> 3:1); and ↓ avoid norgestrel and
infertility (40%), sex hormone– levonorgestrel)
hirsutism (80%), acne, binding globulin. ● Hirsutism traditional
obesity (50%), hair removal techniques ,
possible ovarian ● Ultrasound spironolactone +OCPs,
enlargement, and >eight follicles < OCPs + progestin with
acanthosis nigricans, 10 mm in antimineralocorticoid
20% of patients are diameter, but activity
asymptomatic imaging is not
necessary or
sufficient for the
diagnosis.

Chlamydia ●Asymptomatic 80% ●Test sample from ● Doxycycline or


●mucopurulent urine or cervical ● Azithromycin
discharge and a swab using  ●Alternatives
friable cervix. nucleic acid erythromycin, levofloxacin,
amplification tests, or ofloxacin
genetic probe, or
culture.
Gonorrhea ●Asymptomatic ●Test sample from Any of the following
●Mucopurulent urine or cervical ● cefixime ●ceftriaxone
discharge and a swab using  ●ciprofloxacin, ofloxacin,
friable cervix. nucleic acid levofloxacin no longer

79
iSummary SHORT NOTES

amplification tests, recommended by the CDC


genetic probe, or
culture.
Bartholin cyst ●Vulvar Pain worse ● Word Catheter
with walking and placement
intercourse ● Marsupialization  Cysts
● Firm swelling at only
posterior vaginal ● Excision Gland Abscess
introitus
Pelvic ● Lower abdominal ●Clinical ● Inpatient IV AB
inflammatory pain +/− fever +/− Doxycycline + cefoxitin,
disease (PID) nausea and anorexia. followed by doxycycline for
● Cervical motion a total of 14 days
tenderness. Clindamycin+ gentamicin,
followed by either
doxycycline, clindamycin,
to complete 14 days of
total therapy.
●Outpatient AB 
Cefoxitin+ Probenecid or,
or third-generation
cephalosporin +
doxycycline for 14 days
with or without
metronidazole for 14 days.
Ofloxacin or levofloxacin
for 14 days, with or
without metronidazole for
14 days.
Genital herpes ● Painful vesicles +/− ● Clinical ● Oral acyclovir,
(HSV-1 or -2) systemic symptoms ● Consider viral valacyclovir, or
(fever, myalgia) +/− culture or Tzanck famciclovir.
vaginal discharge. smear from
● Tender grouped vesicular fluid.
vesicles on an
erythematous base.
Ruptured vesicles
appear as shallow
ulcers or abrasions.
Inguinal
lymphadenopathy is
also seen.

Genital warts ●Painless growths on ●Clinical ●Topical imiquimod or


(HPV) the genitals, anus, or podofilox; podophyllin;
perineum cryotherapy; trichloroacetic
●fleshy, skin-colored acid; electrocautery; or
“cauliflower”papules excision.
or plaques on the
vulva, vagina, or
cervix.

81
iSummary SHORT NOTES

Chancroid ● Painful genital ●Clinical ● Azithromycin,


(Haemophilus ulcer. ●Gram stain may ceftriaxone, ciprofloxacin
● Tender papule, help. Rule out HSV (avoid in pregnancy).
ducreyi) pustule, or ulcer with and syphilis.
an erythematous
edge. Tender inguinal
lymphadenopathy
Pubic lice or ●Itch in the pubic ●Clinical ● Topical permethrin,
“crabs” hair. May also affect malathion, or lindane;
(Phthirus the axillae and other decontaminate bedding
pubis) hair-covered areas. and clothing.
●Tiny white nits are
attached to hair.
Adult lice may be
seen.

Molluscum ● Dome-shaped ●Clinical ●Most resolve


contagiosum pearly papules spontaneously and do not
(painless and require treatment.
nonpruritic)with ●Imiquimod, cryotherapy,
central umbilication curettage, and
are seen. electrodesiccation.
Endometriosis ● Infertility ● Laparoscopy is ● Combined hormonal
● Dysmenorrhea, required for contraceptives or
dyspareunia, and low definitive diagnosis progestin-only methods
back pain that + lesions must be ● Danazol
worsens during histologically ● GnRH agonists (IM
menses confirmed leuprolide, SQ goserelin,
nasal nafarelin)
● Surgical treatment for
infertile patients
● Hysterectomy with
oophorectomy
intractable pain who no
longer want to become
pregnant.
Ovarian ●Usually sudden- ● Ultrasound ● Laparoscopy
Torsion onset, severe, ● Laparoscopy conservative treatment
unilateral pelvic pain consists of uncoiling the
● unilateral, tender torsed ovary.
adnexal mass
● Salpingo-
oophorectomy severe
vascular compromise,
peritonitis, or tissue
necrosis.

81
iSummary SHORT NOTES

 Summary of Differential Diagnosis of Vaginal Discharge


Feature Physiologic Bacterial Candidal Trichomoniasis Atrophic
Discharge Vaginosis Vaginitis Vaginitis
Organism ●Gardnerella ●Candida albicans ●Trichomonas
vaginalis and vaginalis
anaerobes
Risk factors ●Douching, ●Diabetic, pregnant, ●estrogen
antibiotic use, and HIV, antibiotics, deficiency
use of an IUD, long-term steroids,
and pregnancy. or
immunosuppressive,
OCPs, and
diaphragms, tight-
fitting/poorly
ventilated clothes.
Transmission ●not sexually ●not sexually ●sexually
transmitted transmitted transmitted
Symptoms ●None or ●None or ●Itching, burning ●Asymptomatic ●Itching,
change in malodorous pain, dyspareunia, Malodorous soreness.
amount or discharge and/or discharge discharge Burning
consistency of No odor dyspareunia
discharge bleeding or
spotting
Signs ●Normal pool ●Thin, Grey- ●“cottage cheese” ●Grey-green frothy ●Tissue pallor,
in posterior white discharge Curdlike yellow or discharge dryness,
vaginal fornix adherent to white discharge ●“strawberry decrease in
vaginal walls adherent to vaginal cervix” in < 25% vaginal rugae
wals, vulvar ●Thin, watery
erythema discharge
pH ●3.8 to 4.5 ●>4.5 ●3.8 to 4.5 ●6 to 7 ●6 to 8
10% KOH ●Negative ●Positive whiff ●Pseudohyphae or ●Negative ●Negative
test Amine budding yeast
(fishy) odor
wet mount ●Few white ●Clue cells ●Occasionally ●mobile, flagellated ●Red and white
microscopic cells, reveals budding trichomonads, blood cells, many
examination predominance yeast many WBCs bacteria
of long rod- ●Parabasal cells
shaped bacteria
Gram stain ●Gram-positive ●Lack of gram- ●Gram-positive ovals ●Trichomonads ●Lack of gram-
rods positive rods or tubes positive rods
Culture ●Normal flora ●Not indicated ●90% sensitive but ●90% sensitive ●Not indicated
positive in 20% of using Diamond's
asymptomatic liquid medium
women
Treatment ●Reassurance ●Metronidazole ●Fluconazole 150 ●Metronidazole 2-g ●Mild symptoms
gel 0.75% 5D mg tablet dose of (Treat do not require
●Clindamycin ●Miconazole or sexual partener) treatment.
cream 2% 7D clotrimazole, are ●Dryness 
●Oral creams or vaginal vaginal
metronidazole suppositories lubricants

82
iSummary SHORT NOTES

500mg 7D Terconazole, ●Topical or oral


tioconazole, and estrogen
butoconazole replacement
●Pregnant topical (<6M IN
azole agents for 7 postmenopausal)
days
Recurrence ●≥3 episodes/1 ●≥4 episodes/year ●higher dose or
year ketoconazole (200 longer duration of
●Longer course mg twice a day for 5 metronidazole
of oral days, or daily for up
metronidazole to 6 months)
or a course of ●vaginal boric acid
oral suppositories (600
clindamycin mg daily for 14 days)
(300 mg twice a can be used.
day for 14 days).
KOH, potassium hydroxide; WBC, white blood cell.

 Contraception
Method Side effects Risks/Contraindications
COCs ●Breakthrough bleeding (BTB)  ●Pregnancy
due to noncompliance, during the ●Postpartum less than 6 weeks and
first 3 months switched to a breastfeeding
different COC formulation, NSAIDs ●Age >35 years and heavy smoker (>15
●Amenorrhea normal cigarettes per day)
consequence different COC ●Systolic BP >160 mm Hg, diastolic BP
formulation > 99 mm Hg
●Bilateral breast tenderness ●Hypertension with vascular disease
switch to a COC with a lower ●Diabetes with neuropathy,
estrogen dose retinopathy, nephropathy, or vascular
●Nausea take the COC at night or disease
with food ●History of DVT or pulmonary
●Weight gain embolism
●Headaches lower estrogen dose ●Major surgery with prolonged
COC, stop COC if progressive immobilization
headache. ●History of ischemic heart disease
●Decreased libido rule out ●History of stroke
depression, switch to a different ●Complicated valvular disease (with
COC formulation atrial fibrillation, pulmonary
●Hypertension (small degrees hypertension, bacterial endocarditis)
elevation or overt) ●Severe headaches with focal
neurologic symptoms
●Current breast cancer
●Active viral hepatitis, severe cirrhosis,
benign or malignant liver tumors
●Current gallbladder disease in
women who are already susceptible.
PROGESTIN-ONLY ● Breakthrough bleeding and
PILLS spotting are more common and

83
iSummary SHORT NOTES

persistent than with COCs.


●Amenorrhea is also more
common than with COCs.
●Less likely to cause headaches,
blood pressure elevation, and
depression
● Breast tenderness
Transdermal ●Similar as COCs ●Similar as COCs
contraceptive ●Local skin irritation
patch ●Reversible application site
hyperpigmentation
Vaginal ● Vaginitis (13.7%), headache ● Theoretically the same as those for
contraceptive (11.8%), and leukorrhea (5.9%). COC pills.
ring ● Theoretically the same as those
for COC pills.
● Lower incidence of BTB and
nausea than that observed for low-
dose COC pills.
Injectable depot ● Irregular bleeding for the first 6 ● Loss of bone density (±reversible
medroxyprogeste months after discontinuation)
rone acetate ●Amenorrhea can persist for ●↓HDL
(Depo Provera) months after discontinuation
● Headaches, breast tenderness,
dizziness, and palpitations
●± weight gain
●Depression
Implanon ● Irregular bleeding and ●Infection, expulsion, and local
(etonogestrel amenorrhea Over time, bleeding reactions is rare
implant) usually becomes more regular.
● Localized bruising can occur
after insertion or removal.
● Ovarian cysts, headaches, and
acne

Copper T 380a ● Increase menstrual bleeding, ●The risk of uterine infection is higher
intrauterine cramping, and the risk of anemia in the first 20 days after IUD insertion
device NSAIDs. Bleeding typically decreases ● History of Wilson's disease or allergy
over time. to copper.
●Recent history of PID (within the past
3 months) or current PID.

84
iSummary SHORT NOTES

 Preferred Contraceptive Options for Select Patient Types


Patient Type Preferred Comments
Contraceptive Options
Adolescent DMPA, implant, or COC plus
condom
Potential compliance DMPA, implant, patch, ring, or IUD
problems
HIV and STI risk Condom plus any other form of
contraception
History of PID Barrier methods or COC plus
condom
Postpartum and DMPA, implant, POP, or IUD. LAM COCs can decrease
lactating appropriate up to 6 months if quantity and duration of
breast-feeding nearly exclusively lactation
Smoker >35 years old Barrier methods, DMPA, implant, COCs, patch, or ring not
POP, or IUD recommended
Diabetic Barrier methods, DMPA, or implant COCs appropriate in
young, normotensive,
well-controlled diabetics
Hypertensive Barrier methods, DMPA, implant, COCs appropriate in
IUD, or POP young, well-controlled,
nonsmoking
hypertensives
History of stroke/TIA Barrier methods or IUD Estrogen-containing
contraceptives
contraindicated
History of Barrier methods or IUD Estrogen-containing
thromboembolism contraceptives
contraindicated
Coronary artery disease Implant or IUD Estrogen-containing
contraceptives
contraindicated
Mitral valve prolapse DMPA, Implant, IUD, or COC if
asymptomatic
Polycystic ovarian COC COCs can suppress
syndrome (PCOS) androgen excess
Sickle-cell disease DMPA or barrier methods DMPA can decrease
frequency of crisis
Gallbladder disease DMPA, implant, or IUD COCs can accelerate
progression
Taking anticonvulsants DMPA or IUD Efficacy of COCs, POPs,
or other hepatic and implants may be
inducers reduced
Desires long-term DMPA, implant, or IUD
reversible
contraception
Desires short-term Barrier methods, COCs, POPs,
reversible patch, or ring
contraception

85
iSummary SHORT NOTES

Desires convenience DMPA, implant, or IUD


Desires permanent Vasectomy or tubal ligation
contraception

COC, combined oral contraceptive; DMPA, depot medroxyprogesterone acetate; IUD, intrauterine
device; PID, pelvic inflammatory disease; POP, progestin-only pill

 Puberty

 At the onset of puberty males have testicular enlargement (first sign


the appearance of pubic hairenlargement of the penis and spermarche.
Skeletal and muscle growth are late events in male puberty.
The first objective sign of puberty in girls is the beginning of the height
spurt.  development of breast buds between ages 8 and 11 years. The
height spurt correlates more closely with breast developmental stages
than with pubic hair stages.
 Delayed sexual maturation is defined as follows:
 In boys: no testicular development by age 14 years, no pubic hair by age
15 years, and more than 5 years between the initial and completed
growth of the genitalia
 In girls: no breast development by age 13 years, no pubic hair before
age 14 years, and no menstruation within 5 years of the development of
breast buds, or if menstruation does not occur by age 16.
Precocious puberty is defined as the presence of breast development or
pubic hair before 8 years of age.
Pubertal delay is defined as absence of breast development before the
age of 13 years or the lack of menstruation 5 years after breast growth.

 Abnormal uterine bleeding

 Hyperthyroidism  Oligomenorrhea (most common)


 Hypothyroidism menorrhagia (most common)
 Endovaginal ultrasonography is a sensitive test for patients with
postmenopausal bleeding whether or not they are using hormone
replacement therapy. Therefore, postmenopausal patients with an
endometrial stripe thicker than 4–5 mm should have a histologic biopsy.
 A transvaginal ultrasound is a reasonable first step in postmenopausal
bleeding.
 In patients younger than 40 years, endometrial cancer is usually seen in
obese patients, patients who are chronically anovulatory, or both.
Therefore, a patient who presents with an anovulatory pattern of bleeding
for more than 1 year should be evaluated for hyperplasia and neoplasm
with an endometrial sample.
 In addition, the evaluation of women >35–40 years who present with a new
onset of menorrhagia should include endometrial sampling, because the
incidence of endometrial cancer increases after age 35.

86
iSummary SHORT NOTES

 D&C is useful in patients with cervical stenosis or other anatomic factors


that prevent an adequate endometrial biopsy. It is not effective as the sole
treatment for menorrhagia.
 treat an acute episode of heavy uterine bleeding
 Premarin used intravenously will temporarily stop most uterine
bleeding, regardless of the cause. The dose commonly used is 25 mg of
conjugated estrogen every 4 hours.
 After one or two doses, if bleeding has not slowed or if patient is
unstable, intrauterine balloon placement for tamponade, or dilatation
and curettage (D&C) should follow.
 After acute bleeding is controlled, the physician should add a progestin
to the treatment regimen to induce withdrawal bleeding. To decrease
the risk of future hyperplasia or endometrial cancer, a progestin is
continued for 10–14 days each cycle.
 In dysfunctional uterine bleeding  Pelvic ultrasound should be
performed only in the evaluation of pelvic disorders discovered during
clinical examination, in women who weigh 90 kg or more, or in women who
are 45 years of age or older. Dilation and curettage (D&C) is generally not
therapeutic in cases of heavy menstrual bleeding.

 Amenorrhea

 Primary amenorrhea is defined as failure of menses to appear by age 16 in


a patient with normal 2° sexual characteristics or by age 14 in a patient with
no 2° sexual characteristics.
 Secondary amenorrhea is defined as the absence of menses for three
consecutive months in women with previous normal menstruation or for
nine months in women with prior oligomenorrhea.
 Patients with anorexia nervosa often need to reach 90% of their normal
weight prior to the return of menses.

 Vaginal discharge

 Complications of BV PID, premature rupture of membranes, preterm


delivery, postpartum endometritis, postcesarean endometritis,
posthysterectomy cuff cellulitis, and postabortion infection
 Treatment of asymptomatic BV. These women do not need treatment
unless they are at risk for HIV infection or undergoing a procedure such as
hysterectomy or therapeutic abortion, pregnant with a high-risk pregnancy
(prior preterm delivery).
 In Vaginal Candidiasis 
 Fungal culture (should only be considered for resistant symptoms or
recurrent episodes)

87
iSummary SHORT NOTES

 Patients developing itching or burning within a few days of topical


treatment likely have an allergy to topical azole treatment. Treatment
with topical nystatin or an oral agent can be used for these women.
 drug interactions with oral fluconazole Warfarin, STATINs, oral
hypoglycemics (topical azoles are the better choice)
 Patients treated with long-term keotoconazole should be monitored
for hepatic toxicity
 Women with recurrent candidal infections should be considered for HIV
and diabetes testing. Diets low in concentrated sugars may also be
beneficial.
 it is not necessary to treat asymptomatic women who have evidence of
Candida Pap smear, wet prep, or vaginal culture
 Prevention prophylactic antifungal medications (history of frequent
candidal infections who require antibiotic treatment), Male sexual
partners should be examined for Candida species and treated if
infection is present
 Trichomoniasis
 Complications abnormal Pap smears. When present in pregnant
women, premature rupture of membranes and premature birth,
increase risk of postoperative infection, especially posthysterectomy
cellulitis.
 Metronidazole is safe in pregnancy

 Chlamydia

 Retesting after treatment is not necessary in nonpregnant women. Consider


rescreening high-risk patients (such as young women and teens who have
previously been infected with Chlamydia) because of the high rates of
reinfection.
 The presence of gonorrhea must be ruled out and patients should be
treated for gonorrhea if chlamydia is found, and vice versa.
 Sexual partners should be treated as well, and women are advised to
abstain from intercourse for 7 days after they and their partner have
started treatment.

 Prevention of STIs

 For patients who are sexually active, consistent use of latex condoms offers
the best protection against STIs.
 Vaccination against HPV is recommended for all girls by age 12, and for
young women up to age 26. The three-dose vaccine Gardasil protects
against virus strains that cause most cervical cancer (types 16 and 18) and
genital warts (types 6 and 11). It is more effective if given before the
patient initiates sexual activity.

88
iSummary SHORT NOTES

 Although there is no evidence that IUDs have to be removed in women


diagnosed with acute PID, if the IUD is not removed CDC guidelines
mandate that close clinical follow-up be provided.
 Patients with PID should be seen within 3 days after initiation of therapy.

 CONTRACEPTION

 Missed pills

 Missed one pill


 Take the missed pill as soon as she remembers
 Keep taking remaining pills as schedule
 No backup method needed

 Missed more than one pill


 If ≥ 7 pills are left in the pack

 Take your pill now, take the rest as usual


 Use backup method (condom or spermicidal) for 7 days

 If < 7 pills left in the pack

 Take your pill now, take the rest as usual


 Start another pack immediately after finishing the current
one, without 7 days break.
 Use backup method (condom or spermicidal) for 7 days
 Benefits of COCs

 COCs reduce the risk of ovarian and endometrial cancer


 COCs also decrease the risk of benign breast disease, but not ductal
atypia
 By inhibiting ovulation, COCs also prevent ectopic pregnancies.
 There is also evidence to suggest that COCs may protect against
atherogenesis, osteoporosis, endometriosis, rheumatoid arthritis, and
lower incidence of ectopic pregnancy

 Some women consistently experience headaches during their placebo


week, associated with withdrawal of estrogen (menstrual
migraines)continuous COC dosing, taking active pills for three
continuous cycles, or by supplementing low-dose estrogen during the
placebo week.
 There is an increased risk of cerebral thromboembolism in women with a
history of pseudotumor cerebri or migraines with aura or focal neurologic
changes; therefore, OCPs are contraindicated in this group of women, they
also should be discontinued in women who have progressive headaches
while taking them.

89
iSummary SHORT NOTES

 POPs are preferable to COCs for lactating women, because estrogen can
significantly reduce breast milk production.

 TRANSDERMAL CONTRACEPTIVE PATCH applied for 3 weeks then


removed, Compared to COCs, the patch is associated with fewer
contraceptive failures because of better compliance. A good choice in
women who have a difficult time remembering to take COCs daily. The
patch might not be the best first choice for women with chronic skin
conditions.

 VAGINAL CONTRACEPTIVE RING  The woman inserts the ring and leaves
it in place for 3 weeks. She removes the ring for the fourth week. If the
ring is removed for any reason, it should be inserted again within 3 hours
to maintain contraceptive efficacy.

 DMPA decreases pain in women with endometriosis and decreases the


risk of (sickle-cell crisis, PID, ectopic pregnancy, frequency of seizures,
ovarian cancer, endometrial cancer menstrual cramps, anemia). The
manufacturer suggests that DMPA should not be used longer than 2 years
unless other birth control methods are inadequate. As with all women,
DMPA users should be encouraged to obtain adequate calcium and
vitamin D via diet or supplements, or both, and partake in regular weight-
bearing exercise.
 ETONOGESTREL IMPLANT Implanon is effective within 24 hours of
insertion and provides up to 3 years of highly reliable contraception.
provide a rapid return to fertility after removal

 The FEMALE CONDOM can be inserted up to 8 hours prior to intercourse.


It is designed for one-time use only, but it does not have to be removed
immediately after coitus. There is a higher risk of slippage than with the
male condom. It can offer superior STI protection for women than the
male condom.

 DIAPHRAGM The device may be inserted immediately before or up to 6


hours before intercourse, and it must remain in place for at least 6 hours
after intercourse. The diaphragm should not remain inside the vagina for
longer than 24 hours at a time, because this increases the risk of toxic
shock syndrome (TSS). There is a twofold to threefold increased risk of
urinary tract infections (UTIs) associated with diaphragm use. The
diaphragm can cause vaginal irritation, recurrent yeast infections, and
bacterial vaginosis.

 CERVICAL CAP The cervical cap is more effective in nulliparous than in


multiparous women. It can be left in place for up to 72 hours. Urinary tract

91
iSummary SHORT NOTES

infections occurred less frequently with FemCap compared with


diaphragms.

 SPERMICIDES (nonoxynol 9) A dose of spermicide remains effective for


approximately an hour after insertion. After intercourse, most
spermicides should be left in the vagina for at least another 6 hours.
Spermicides can be a useful, short-term contraceptive for women who
have experienced a temporary lapse in another method (e.g., pills), or are
waiting to start a different method (e.g., sterilization), in the case when a
couple has infrequent intercourse. Frequent exposure to nonoxynol 9
(when used several times a day, every day) can result in vaginal mucosal
damage and possibly increase the risk of STI transmission. Nonoxynol 9
spermicides can cause genital irritation and lead to yeast infections and
bacterial vaginosis.

 IUD
 It is FDA approved for up to 10 years of use, although the World
Health Organization (WHO) suggests it is effective for 12 years.
Fertility usually returns shortly after removal of the device.
 The copper IUD can be inserted at any time during the menstrual
cycle as long as pregnancy has been ruled out. Insertion may be
performed during menstruation to provide additional reassurance
that the woman is not pregnant. If insertion is planned during the
luteal phase, another nonhormonal contraceptive should be used until
completion of the next menses. It may be inserted immediately
postpartum or postabortion.
 T 380A IUD an acceptable contraceptive option for women with
previous ectopic pregnancies. However, in the very rare event that
a woman becomes pregnant while using this IUD, the pregnancy is
more likely to be ectopic.
 For women with complicated valvular heart disease, prophylactic
antibiotics prior to insertion should be considered to prevent
infective endocarditis.
 It is NOT contraindicated to place an IUD in a nulliparous patient or
a patient with a history of PID.

 EMERGENCY CONTRACEPTION

 Progestin-only regimen 0.75 mg of levonorgestrel 1 pill repeated


in 12 hours or 2 pills at once (preferred) Up to 120 hours
 The Yuzpe method, (combined hormonal)  consists of 100 μg of
ethinyl estradiol and 0.5 mg of levonorgestrel, repeated twice, 12
hours apart.  Up to 72 hours
 copper-T IUD Up to 5 days
 WHO states that the only contraindication to emergency
contraception is confirmed pregnancy because emergency
contraception will not be effective if a woman is already

91
iSummary SHORT NOTES

 Infertility

 Begin the workup for women under 35 years old after 12 months of
infertility and after only 6 months of infertility in:

 Women over 35 years old (infertility increases with age, doubling for
women between the ages of 30 and 35).
 patients with significant risks for infertility, like irregular menses or
PID
 Male infertility risk factors exist (e.g., a history of bilateral
cryptorchisism)

 In the evaluation of infertility, the man is usually evaluated first with a


sperm analysis, because the female evaluation may be more extensive.
 The male is evaluated with a complete blood count, urinalysis, and at least
two semen analyses at least 1 month apart.
 The starting dose for clomiphene is 50 mg per day PO on menstrual cycle
days 3 to 7. If a midluteal progesterone >10 pg/mL, continue the same
clomiphene dose. If the progesterone is <10 pg/mL, then increase the dose
of clomiphene by 50 mg per cycle until the patient is ovulating. This
medication does increase the risk of multiple pregnancy and ovarian
cancer. Ovulation should be expected 3 to 8 days after the treatment ends
and should be confirmed by a LH home detection kit and an elevated serum
progesterone on day 21. If ovulation does not occur despite clomiphene
therapy, consultation with an infertility specialist is recommended.
 The treatment of infertile women with endometriosisConservative
surgical treatment may enhance fertility by destroying endometrial implants
and endometriomas.
 PCOS-related infertility weight loss. If this is unsuccessful, clomiphene
citrate +/− metformin may be useful.
 intrauterine insemination (IUI) is the best treatment option for a cervical
factor

 Dysmenorrhea

 Primary dysmenorrhea pain beginning with the onset of menstruation


and lasting 12–72 hours and peaks on the day of the heaviest flow.
 Secondary amenorrhea symptoms beginning after age 20, lasting for 5–7
days, and progressive worsening of pain with time. These patients may also
report pelvic pain that is not associated with menstruation.
 Patients with abnormal findings on pelvic examination who do not respond
to therapy for primary dysmenorrhea or who have a history suggestive of
pelvic pathology should undergo pelvic ultrasound.

92
iSummary SHORT NOTES

 Menopause

 A woman is considered to be postmenopausal after 1 year of amenorrhea.


 Women on synthetic estrogen/progestin therapy at greater risk for heart
disease, strokes, invasive breast cancer, thromboembolic events,
gallbladder disease, and dementia.
 Benefits a reduction in fractures, osteoporosis, colorectal cancer, and
diabetes.
 In postmenopausal women with debilitating symptoms of menopause,
short-term use of hormone replacement therapy is appropriate.(SOR A)
 Indications for estrogen replacement therapy
 Moderate to severe vasomotor instability.
 Moderate to severe genital atrophy.
 Diminished quality of life secondary to menopausal symptoms.
 Osteoporosis prevention when other treatment options are
contraindicated and patient is at significant risk for fracture.
 Contraindications to estrogen replacement therapy
 Estrogen-dependent neoplasia (breast or endometrium).
 Undiagnosed vaginal bleeding.
 Past history of venous thromboembolism (deep venous thrombus,
pulmonary embolus) only absolute contraindication to HT
 Past history of arterial thromboembolism within 12 months
(cerebral vascular accident or coronary artery disease).
 Liver disease.
 Periodic follow-up visits (3 to 6 months) are needed to monitor women on
hormone replacement therapy.

 Screening tests for postmenopausal women

 Height and weight (periodically)


 Blood pressure (periodically)
 Vision (periodically)
 Mammogram/clinical breast examination (after 40 years; annually)
 Pap test (every 1 to 3 years until age 65 years; stop if postabdominal
hysterectomy, not because of malignancy)
 Colorectal (after 50 years; frequency depends on method)
 Fasting lipid profile (after 45 years; every 5 years if previous results were
normal)

93
iSummary SHORT NOTES

 Ovarian cancer

 CA-125 will be ↑ in the presence of malignancy and is useful for tracking


the course of disease, but it is nonspecific and therefore not useful for
screening asymptomatic women.
 Prevention  use of OCPs; 5 years of use has been shown to decrease the
risk of ovarian cancer. Women at increased risk (advancing age, family
history of breast or ovarian cancer, a personal history of breast cancer and
nulliparity) should be counseled on this benefit of OCPs.
 Routine screening for ovarian cancer by ultrasound, measurement of tumor
markers, and pelvic examination is not recommended.

 Cervical cancer

 Screening  begins at age 18 or at the time of first sexual intercourse, then


every 1 to 3 years, depending on risk factors, up to age 65.
 Routine screening is not recommended in the following:

 Women > 65 years of age with a history of adequate -ve screening and
who are otherwise not at high risk.
 Women who have had a hysterectomy for benign disease.

 PAP SMEAR EVALUATION FOR CERVICAL CANCER

94
iSummary SHORT NOTES

 AGUS (atypical glandular cells of undetermined significance): Do


endometrial sampling (eg, Pipelle) + colposcopy + HPV testing. Another
option is to do endometrial and endocervical sampling and if this is negative
to proceed to colposcopy.

95
iSummary SHORT NOTES

 Hyperprolactinemia

 MRI is the imaging modality of choice for the anatomic evaluation of the
hypothalamus and pituitary gland.
 Laboratory testing in a person suspected of hyperprolactinemia should
include testing for serum prolactin and creatinine levels and thyroid
function tests.

 Ovarian Mass

 Ultrasound is the most important diagnostic study.


 Prepubertal patientsall adnexal masses in prepubertal girls must be
evaluated with ultrasound and referral.
 Premenopausal women with cysts < 10 cm can be followed. Monophasic
OCPs can suppress symptomatic functional cysts. All solid adnexal masses
call for immediate surgical exploration, as does the presence of ascites.
 Postmenopausal women with asymptomatic ovarian cysts < 3 cm in
diameter and a normal serum CA-125 can be followed with serial
ultrasounds, as such patients have a very low risk for malignancy. Larger
cysts should be evaluated laparoscopically.
 Symptomatic patients with an ultrasonographically suspicious mass and ↑
serum CA-125 should be referred for surgical evaluation.

96
iSummary SHORT NOTES

PEDIATRIC
Disease Management
Chemoprophylaxis for ●1% tetracycline ophthalmic ointment
opthalmia neonatorum ●0.5% erythromycin ophthalmic ointment
●1% silver nitrate aqueous solution (penicillinase-
producing Neisseria gonorrhoeae)
Prevent vitamin K ●1.0 mg of vitamin K intramuscularly
deficiency bleeding
Mastitis ●emptying the breasts of milk, cold pack between
feedings, continue to breast-feed with the affected
breast
Antibiotic: cloxacillin or dicloxacillin. Erythromycin
or 1st cephalosporins for penicillin-allergic patients.
Candidal infection of the ●nystatin liquid twice a day
nipples
Febrile seizures ●If prolonged: Rectal diazepam, intravenous
phenytoin, Intravenous lorazepam
Primary varicella ●Antipyretics and antihistamines for pruritus
(chickenpox) ●Oral or intravenous acyclovir (complicated cases or
for immunocompromised children)
Measles ●Bed rest, adequate fluid intake and antipyretics for
(rubeola virus) fever.
●Vitamin A, 200,000 IU orally once daily for 2 days
Rubella ●Symptomatic and supportive
(German measles)
Roseola infantum ●Symptomatic and supportive
(exanthem subitum; sixth
disease)
Erythema infectiosum ●Supportive
(fifth disease) ●Immune globulin (0.4 mg/kg) in patients with
congenital immune deficiency
Kawasaki disease ●High-dose aspirin; IVIG.
(mucocutaneous lymph ●Warfarin for coronary artery aneurysms > 8 mm in
node syndrome) diameter.
●Corticosteroids are controversial, as they may be
associated with an ↑ incidence of aneurysms.
Meningococcemia ●Penicillin G, ampicillin, cephalosporins, and
chloramphenicol
●Prophylaxis rifampin. Ceftriaxone, ciprofloxacin -
Meningococcal vaccination
Infectious mononucleosis ●Symptomatic and supportive
in children ●Corticosteroids may benefit patients with
respiratory compromise or severe pharyngeal edema
●Contact sports should be avoided until the

97
iSummary SHORT NOTES

splenomegaly has resolved


Scarlet fever ●IM penicillin or oral erythromycin
Functional constipation ●Infants glycerin suppositories - Barley malt
extract, corn syrup, lactulose, or sorbitol - prune,
pear, apple juices
●ChildrenMineral oil, magnesium hydroxide,
lactulose, and sorbitol- Senna and bisacodyl
Encopresis ●Disimpaction followed by regular use of a stool
softener (Polyethylene glycol, Mineral oil, Psyllium,
and Milk of magnesia)
Enuresis ●Expectant management ●Behavioral therapy
●Alarm system
●Desmopressin - imipramine
Infantile regurgitation ●1. Thickened formula and a trial of a hypoallergenic
formula ●small feeds at shorter intervals
●H2RAs (trial for limited time) - antisecretory or
prokinetic therapy (>2y)
Cryptorchidism ●Hormonal or surgical treatment, or both; can start
at 6 mo of age and should be completed before 2 y
Hypospadias ●Circumcision contraindicated
●Urology referral, usually within 3–6 mo
Developmental dysplasia ●Abduction splints in infants < 6 mo; ●open or closed
of hips reduction more effective in those > 6 mo

 Fever with rash


Disease Infectivity Rash Rash Prevention
character distributio
n
Primary ●1-14 years ●Macule to ●face, ●immune
varicella ●Patients are papule to extremities, globulin:
(chickenpox) contagious from vesicle trunk, and pregnant, and
24 to 48 hours followed by possibly are exposed to
before onset of crusts mucous chickenpox,
rash until all membranes others1
vesicles are (lips and vulva) ●Vaccine: 2
crusted (3 to 7 doses at 12 to
days from onset). 15 months and
4 to 6 years
Measles ●Infants to young ●Purple-red ●Hairline, ●Vaccine: 2
(rubeola virus) adults coalescing forehead, and doses at 12 to
●The disease is maculopapular face. By the 15 months and
communicable 2 which blanches third day it 4 to 6 years
to 4 days before spreads to the ●Susceptible
the onset of the feet and it fades household
characteristic rash within 3 days in contacts within
●Infected the order of ˂72h of
individuals are appearance exposure 

98
iSummary SHORT NOTES

contagious up to ●The total vaccine


7 days after the duration of the ●Prolonged
onset of rash is exposure > 72
symptoms. approximately 7 h vaccine +
days immunoglobuli
n

Rubella ●Adolescent/ ●Pink-red ●Appear first


(German young adult lesions that are on the face and
measles) discrete and do spread rapidly
not coalesce downward to
the neck, arms,
trunk, and
lower
extremities.
●The total
duration is 3 to
4 days
Roseola ●Younger than 2 ●Pink macules ●Appear first
infantum years or on the trunk
(exanthem ●Abrupt onset of maculopapules and then
subitum; sixth fever, commonly 2 to 3 mm in spread to the
disease) 40 to 40.6°C, diameter that neck, face, and
which persists for blanch with upper and
3 to 5 days with a pressure. lower
rapid decline extremities.
Rash appears ●The total
after fever duration is 1 to
declines. 2 days.
Erythema ●3-12 years ●Marked ●Face and
infectiosum ●It is moderately erythema circumoral
(fifth disease) contagious, with (“slapped pallor. This is
outbreaks cheek” followed by a
occurring in appearance) generalized
families, daycare lace-like rash on
centers, and the trunk and
classrooms. extremities
Rash has a sudden ●The rash may
onset on the face last from a few
days to several
weeks.
Kawasaki ●Under 5 years ●Deeply ●Variable and
disease ●Fever (≥5 days), erythematous may be diffuse,
(mucocutaneou nonpurulent and truncal, or
s lymph node conjunctivitis, polymorphic limited to the
syndrome) erythema and and most extremities. It
fissuring of the commonly slowly fades
lips, induration of manifests as with resolution
the hands and pruritic of clinical
feet, enlarged plaques that illness.
lymph node mass vary from 2 to

99
iSummary SHORT NOTES

(>1.5 cm) 10 mm. They


may resemble
urticaria or the
target lesions
of erythema
multiforme
Infectious ●10 to 30 years ●erythematous ●Trunk and
mononucleosis ●fever, , macular and arms
in children tonsillopharyngitis papular, or It appears
, cervical morbilliform during the first
lymphadenopathy day and
, and disappears by
splenomegaly day 6 or 7
●The rash with amoxicillin or
EBV infection other
occurs in 10% to ●Antibiotic
15% of patients diffuse copper-
colored rash
Scarlet fever ●School age ●Diffuse ●Facial flushing
●Acute onset with erythema with with circumoral
fever, sore throat. sandpaper pallor; linear
texture erythema in
skin folds.
1
Full-term infant born to mother who has chickenpox <1 week before delivery, Every premature infant born to a
mother with active chickenpox (even if present longer than 1 week),
Newborns whose mothers had onset of varicella 5 days before delivery or within 2 days after delivery who are
exposed to varicella, Hospitalized premature infants (gestation of 28 weeks or more) whose mothers have no
history of chickenpox, Hospitalized premature infants (gestation of <28 weeks or ≤1,000 g) regardless of maternal
history

Disease Diagnosis Treatment


Breastfeeding ●Related to dehydration while ●More frequent breastfeeding
jaundice the mother’s milk supply is (feedings should be increased to
coming, resolves when feeding more than ten times/day)
is well established.
Breast milk ●Typically appears after the ●Continue to breast-feed or
jaundice first week and may persist for alternating breast and formula
many weeks. feeding for 2 to 3 days
Neonatal sepsis ●History - Lab ●Ampicillin with gentamicin
until culture results (blood, urine,
and CSF)
Influenza ●Clinical - nasopharyngeal ●Oseltamivir – zanamivir
swabs ●Immunization (best)
Common cold ●Clinical ●Acetaminophen
●Topical decongestants (old
children)
●Humidified air and nasal saline
drops then gentle bulb syringe
suction
GABHS Centor criteria - Rapid ●1. Penicillin 1. Erythromycin
pharyngitis streptococcal screen - Throat (penicillin-allergic patients)
culture Amoxicillin – Amoxicillin-

111
iSummary SHORT NOTES

clavulanate potassium-
Azithromycin – cephalosporin -
Ampicillin
Peritonsillar ●Clinical (unilateral) ●Needle aspiration
abscess ●1. Needle aspiration ●Antibiotic(penicillin, clindamycin,
cephalosporins, or metronidazole)
●Incision and drainage or
immediate tonsillectomy
(advanced)
Pneumonia ●Clinical - CXR ●Neonates ampicillin and
gentamicin, ± cefotaxime
3 weeks to 3 months and 5–15
years Macrolides
4 months and 4 years 1.
amoxicillin - withheld if virus
Bronchiolitis ●< 2 years (peak 6m) ●1.supportivefluids,
●Clinical  History (URTI) - antipyretics, head elevation and
Examination suctioning
●IV fluids ifcough, retractions,
or nasal flaring
●Tent with cool humidified oxygen
●Epinephrine (not in hospitalized
patients), ± albuterol
CROUP ●3 months to 6 years (peak ●Dexamethasone IM or Oral
6m:2y) ●Nebulized budesonide
●Clinicalbarking, "seal-like" ●Racemic epinephrine - L-
cough ●Epinephrine (in hospital)
●CXRsteeple" sign ●Humidified air
Epiglottitis ●2 to 7 years (peak 3.5y) ●Airway endotracheal
●Clinicalrapid onset – intubation OR cricothyrotomy or
drooling tracheotomy
●Lat CXRthumb sign ●Antibiotics (cefotaxime,
ceftriaxone, cefuroxime, oxacillin
or nafcillin, cefazolin, clindamycin
ampicillin plus sulbactam,
ampicillin plus chloramphenicol)
Gastroenteritis ●Clinical ●Rehydration and refeeding
●Stool analysis/culture(if ●Antibiotics according 2 the
bloody, high fever, persistent bacteria found
symptoms > 1 week,
tenesmus, or a history of
foreign travel,
Immunocompromised)
UTI ●Urine culture ●Hospitalization <2m or toxic,
are dehydrated, or are unable to
retain oral intake
●Antibiotic sulfonamide,
TM/SMZ,
amoxicillin/clavulanate or a
cephalosporin.
●1. Ceftriaxone if suspect

111
iSummary SHORT NOTES

pyelonephritis
Meningitis ●Clinical ●<1mampicillin +
●If Neisseria meningitidis ( cefotaxime/gentamicin
petechial rash) ●>1m Vancomycin +
●LP cefotaxime/ceftriaxone
Pyloric stenosis ●Nonbilious, often projectile, ●Longitudinal pyloromyotomy
vomiting shortly after feeding-
“olive-like mass"
●US or Barium swallow "string
sign"
Intussusception ●2m to 6y (peak 6:12m) ●Air enema
●Intermittent abdominal pain, ●Laparotomy
sausage-like mass, “currant
jelly" stool ●Air enema
Meckel’s ●Painless UG/LG bleeding ●Surgical resection of
Diverticulum symptomatic lesions
Lead poisoning ●Clinical ●Chelationdimercaprol and
●Lab: Levels >10 µg/dL edetate calcium disodium,
followed by penicillamine
Foreign body ●CXR-AXR ●Below the diaphragm
swallow (coins) observed until passing
lodged in the
●Esophagusendoscopy and
removal
Foreign body ●If in esophagus may be observed for up to 24 hours (exception of
swallow (blunt button batteries), If fails to pass into stomach removed or
objects) pushed into the stomach
●Objects lodged in esophagus for >24 hours or for an unknown
duration  removed endoscopically
●Swallowed button or disc batteries Early intervention

 Common bones complaints


Disease Diagnosis Investigation Treatment
Slipped capital ●Vague ●X-rays ●Surgical pinning
femoral progressive pain icecream scoop
epiphysis and limp slipping off its cone
overweight
adolescent boys
Septic arthritis ●Pain, fever, ●Drainage - culture ●Broad-spectrum IV
malaise, and ↓ antibiotics then oral
range of motion (total 2:3w)
(ROM)
Legg-calvé- ●Hip, groin, or ●Lateral lucency ●Expectant
perthes disease thigh pain and and fragmentation observation both
difficulty of the femoral clinically and
ambulating head radiographically
abduction casts -

112
iSummary SHORT NOTES

surgery
Osgood- ●Pain at the tibial ●Radiographs may ●Rest, ice, anti-
schlatter tubercle, reveal inflammatories
syndrome especially with fragmentation at
eccentric the tibial tubercle
exercises
Osteomyelitis ●Infant: fussiness ●↑ WBC, CRP, and ●Broad-spectrum IV
and ESR antibiotics with
pseudoparalysis of ●X-ray findings lag surgical debridement
a limb behind clinical
●Child: acute findings
tenderness and ●Aspiration of the
will refuse to bear bone for culture
weight on the
affected limb
Transient ●2- to 6-year, ●Joint aspiration, ●Mild symptoms
synovitis prior URTI ●Plain film observed without
limp or refusal to radiographs, ESR further investigation
walk, pain over and CBC with rest and observation,
the groin and/or differential NSAIDs
proximal thigh differentiate
transient synovitis
from septic
arthritis
Growing pain ●Active children ●Reassurance, rest,
aged 2–5 and short-term use
●Pain is of NSAIDs.
commonly
bilateral or/and
localized to the
calf, or felt at the
ankle, knee, or
thigh, no limping
occurs primarily at
night and is better
during the day

 Common infant problems

 Infants commonly lose up to 10% of their birth weight in the first few days
but should regain this weight by 2 weeks of age.
 A cephalohematoma is a result of bleeding in the subperiosteal space and
does not extend across a suture line. Cephalohematoma is not evident until
a few hours of age.

113
iSummary SHORT NOTES

 Caput succedaneum is caused primarily by subcutaneous edema and can


involve any amount of the scalp. Caput succedaneum is present at the time
of delivery

 Hemangiomas are not always present at birth and might not be visible until
1 month of age. Most hemangiomas spontaneously regress during
childhood and need no specific management. Periocular hemangiomas,
however, should be managed with ophthalmologic consultation and
aggressive therapy. Most are treated if they ulcerate or if they are impairing
a normal function, such as vision. Most ulcerated lesions are successfully
treated with topical antibacterial agents and nonadhesive dressings.
Hemangiomas on the head and neck or multiple hemangiomas should be
further evaluated with imaging studies.

 Absence of a retinal reflex can indicate a congenital cataract, glucoma and a


bright white or asymmetric reflex may be seen with retinoblastoma.
 Monocular cataracts should be corrected as soon as possible (within the
first 3 months of birth) so that vision can develop properly.

 The screening hip examination (Ortolani maneuver and the Barlow


maneuver) should be completed at birth, and infants with any positive or
equivocally positive result should be reexamined at 2 weeks of age. If at 2
weeks the examination remains positive, pediatric orthopedic consultation
should be obtained.

 When the infant has not passed a stool by 24 to 48 hours of age, a plain
radiograph of the abdomen, taken with the infant in the prone position,
often reveals the source of the problem.
 Failure to pass urine by 48 hours of age should lead to evaluation of the
kidneys, ureters, and bladder, including renal ultrasonography.

 With physiologic jaundice, which begins between the second and fourth
days of life, TSB levels are ≤15 mg/dL, direct bilirubin is ≤1.5 mg/dL. TSB
rises by ≤5 mg/dL in 24 hours and Bilirubin levels should peak on day 4 or 5
of life AND resolves by 1 week (in term infants) or by 2 weeks in preterm
infants).
 Phototherapy should be considered at TSB : ≥15 mg/dL at 25 to 48 hours of
age, ≥18 mg/dL at 49 to 72 hours of age, or ≥20 mg/dL in infants over 72
hours of age.
 Exchange transfusion is traditionally performed when the TSB exceeds the
threshold for phototherapy by 5 mg/dL or if phototherapy fails. (SOR C)

114
iSummary SHORT NOTES

 Breast feeding

 Sore nipples: With severe cracking there will occasionally be bleeding.


Breast-feeding can be continued with mild bleeding, but if severe bleeding
occurs the breast should be pumped and the milk discarded to prevent
gastrointestinal upset in the infant.
 Breast milk can be stored in the refrigerator for use within 2 days. Breast
milk intended for longer storage should be frozen as soon as possible. Once
the milk has thawed it can be kept in the refrigerator for 24 hours. Breast
milk should be kept at room temperature for 6 hours only. Warming in hot
water is a better way of reheating.
 New AAP guidelines recommend 200 IU/day of vitamin D from birth
through the teenage years, no exceptions. This can come either from
supplements or foods. Breast milk does not contain enough vitamin D.
 An infant urinates approximately 6–8 times a day. Parents may count
diapers in the first few weeks to confirm adequate feeding. The older child
usually voids 4–6 times daily.
 Breast-fed infants typically stool after each feeding, but some defecate only
two or three times a day. Bottle-fed infants generally have a lower
frequency of stooling. Occasionally, some infants may have only one stool
every 2 or 3 days without discomfort.

 Children with familial short stature have normal bone ages.


 Constitutional delay of growth, which is really a delay in reaching ultimate
height and sexual maturation, presents with delayed bone age and delayed
sexual maturation  Reassurance
 Hypothyroidism and growth hormone deficiency usually present with a
delayed bone age.

 Febrile seizures

 Simple seizures are generalized tonic-clonic events that last less than 15
minutes and do not recur within 24 hours.
 Complex seizures last longer than 15 minutes, demonstrate focal signs, or
recur within 24 hours or in a flurry.
 Recurrent febrile seizures may occur in 50% of children younger than 12
months and 30% of children older than 12 months at the time of their first
simple seizure.
 Epilepsy may develop in 1% to 2.4% of children with simple febrile seizures.

 Diagnosis of Rubella should be confirmed with serologic antibody testing for


rubella IgM antibody or paired sera for IgG antibody titers.

 According to the Centor criteria, the four classic features of streptococcal


pharyngitis are a temperature > 38°C, tender anterior cervical

115
iSummary SHORT NOTES

lymphadenopathy, the absence of cough, and the presence of pharyngotonsillar


exudates.
 In the presence of the Centor criteria:
 4 of 4: Treat empirically without a rapid test.
 2–3 of 4: Test and treat only if the rapid test is positive.
 0–1 of 4: No test, no antibiotics.

 Potential etiology of pneumonia


 Neonates younger than 20 days  group B streptococci, gram-negative
enteric bacteria, cytomegalovirus, and Listeria monocytogenes
 2–7 months  RSV - S pneumoniae
 4 months and 4 years  1. Respiratory viruses - S pneumoniae and
nontypeable H influenzae
 5–15 years  M pneumoniae

 An important item in the differential diagnosis of bronchiolitis is acute asthma,


as there may be many similarities in history and physical examination between
these two conditions.
 The presence of one or more of the following favors the diagnosis of
asthma: family history of asthma, sudden onset without a preceding URI,
repeated attacks, a markedly prolonged expiratory phase of respiration, and
response to one dose of epinephrine.
 Asthma is unusual in the first year of life, and the incidence of bronchiolitis
peaks at 6 months of age.

 The rotavirus has been implicated as a major cause of gastroenteritis in children.


Campylobacter species are the most common bacterial cause of childhood
diarrhea. Entamoeba histolytica is a common cause of diarrhea and dysentery in
developing countries. The most common cause of antibiotic-associated diarrhea
is ampicillin.

 UTI: On a noncatheterized specimen, bacterial counts greater than 50,000 are


consistent with an infection. Bag specimens are prone to contamination, and
are most useful to rule out UTI if the urinalysis and microscopy are negative.
 Practice guidelines from the American Academy of Pediatrics recommend
renal ultrasonography and voiding cystourethrography (VCUG) in all
children two months to two years of age with a documented first urinary
tract infection (UTI). After age two some controversy exists. Some
authorities recommend postponing workup for the first UTI in females.

 Vaccinations

 In preterm infants, immunizations should take place at the same designated


times as for term infants with no adjustments made for premature age. One
exception to this recommendation is that hepatitis B vaccination should be
delayed for 1 month if mothers are negative for hepatitis B surface antigen.

116
iSummary SHORT NOTES

 Infants born to mothers positive for hepatitis B surface antigen (HBsAg)


should receive hepatitis B vaccine and hepatitis B immune globulin within
12 hours of delivery.
 Stable neurologic conditions, or a family history of seizures, are not
contraindications to pertussis immunization.
 If the infant has diarrhea vaccinate OPV as usual and extra dose given one
month after the last dose.
 Contraindication to OPV: Anaphylactic reaction to neomycin, streptomycin,
or polymyxin B
 At least 4 weeks interval is needed between MMR and varicella vaccine.

 Egyptian accelerated vaccination scheme

 Follow the same sequence for Polio and DPT with 4 weeks minimum
intervals
 Measles can be given in the same sitting of Polio and DPT if the infant is
9 month or older.
 Hepatitis B vaccine : Give 1st dose then 2nd dose one month later then
3rd dose 2-6 months apart
 If the child has started vaccination but late for the 2nd or 3rd dose
,continue vaccination according to minimum time intervals ( 4 weeks)

 Apgar score
Sign Score
0 1 2
Heart rate Absent <100 beats/minute ≥100 beats/minute
Respirations Absent Irregular and slow Strong breaths,
(weak cry) crying
Muscle tone Limp Some flexion of Good flexion, active
extremities motion
Reflex irritability No response Grimace Cough, sneeze, cry
to tactile
stimulation
Color Blue or pale Blue extremities, Completely pink
pink body

117
iSummary SHORT NOTES

118
iSummary SHORT NOTES

PSYCHIATRY
Disease Treatment
Major ●Cognitive-behavioral therapy (CBT)
depression ●1.SSRIs

●TCAs

●SNRI
●Atypical
●MAOI
●Electroconvulsive therapy (ECT)
Dysthymia ●CBT
●SSRIs; other classes of antidepressants.
●Exercise
Postpartum ●Support and reassurance
blues

Postpartum ●Behavioral Individual or group therapy; couples therapy


major ●Pharmacologic SSRIs & TCA in moderate to severe PMD-
depression found in breast milk. Weigh the risks and benefits
Bipolar ●Mood stabilizers
disorder
●Behavioral Individual therapy, CBT, and interpersonal therapy
Generalized ●1.SSRIs
anxiety ●1 or 2 Buspirone
disorder ●Benzodiazepinesin chronic cases ●Rapid
, , ●Intermediate 

●TCAs ●MAOI ●AntihistaminicHydroxyzine


●β-Blockers(symptomatic)
●CBT

Panic disorder ●Reassurance


- Agoraphobia ●Pharmacologic for prevention- 1.SSRIs – Benzodiazepines
[ (relieve)- - (prevent) - 2.TCA
●MAOI
●CBT
Obsessive- ●1.CBT - SSRIs: – TCA:
compulsive
disorder
Post- ●CBT- Family Therapy - Eye movement and desensitization
traumatic reprocessing
Stress ●DepressionSSRIs (fl - ) - TCAs
Disorder ( )- Mood stabilizers (
)

119
iSummary SHORT NOTES

●Hypervigilance β-blockers ( )
●Anxiety benzodiazepines ( ) may cause
dependence
●Night mare α-blocker ( ) - atypical antipsychotics
Specific ●CBT: desensitization or repeated exposures
Phobias ●Benzodiazepines
Social phobia ●CBT ●paroxetine, sertraline, clonazepam, and ß-blockers
Conversion ●Brief psychotherapy ●Benzodiazepines
disorder
Hypochondria ●Consult with a psychiatrist, Regular visits - ●SSRIs
sis
Chronic pain ●Multidisciplinary pain clinic
syndrome
Insomnia ●Behavioral
●Short-acting nonbenzodiazepines

●Intermediate-acting benzodiazepines

●Long-acting benzodiazepines
●Trazodone
●SSRIs, mirtazapine, venlafazine, TCAs for comorbid
depression.
●Others Gabapentin, Diphenhydramine or Doxylamine,
●Promethazine, hydroxyzine
●Melatonin for jet lag, shift work, and delayed sleep phase
syndrome, as well as chronic insomnia
Obstructive ●Behavioral lose weight
sleep apnea ●Medications not effective, activating antidepressants may
reduce daytime drowsiness
●Surgery ●Referral to a sleep clinic –
●Nasal continuous positive airway pressure (CPAP)

Circadian ●Melatonin
rhythm sleep
disorder

Restless legs ●Dopaminergic Agents 


syndrome ●Dopamine Agonists
●Opioids
 if pain
●Benzodiazepines  if insomnia
●Gabapentin, Clonodine, iron supplements for patients whose
serum ferritin is below 50.
●Referral to a sleep disorder clinic
Periodic limb ●Clonazepam, temazepam
movement ●Referral to a sleep disorder clinic
disorder

111
iSummary SHORT NOTES

Narcolepsy ●Daytime drowsinessmodafinil, methylphenidate,


dextroamphetamine
●Cataplexy, sleep paralysis, and hallucinations fluoxetine,
paroxetine, sertraline, clomipramine, imipramine, nortriptyline ,
protriptyline, desipramine , venlafaxine, atomoxetine
●Insomnia-hypnotic medications
●Referral to a sleep clinic for diagnosis is indicated.
Anorexia ●Hospitalization hemodynamic or metabolic disturbance,
nervosa medication overdose, suicidal intent, and for the initiation of
nutritional restoration.
Bulimia ●Refeeding begins by increasing the daily caloric intake by 300
nervosa calories and reducing activity by 50%.
●Pharmacotherapy:
Prokinetic drugs delayed gastric emptying
Estrogens  osteoporosis
Trace minerals and vitamins  nutritional depletion,
Topical fluoride and bicarbonate rinses dental erosions
H2 blockers  gastric reflux
Bulk fiber supplements in constipation.
Antidepressants SSRI ( )
avoid TCA
● CBT
Schizophrenia ●1. Antipsychotics
●1.Atypical neuroleptics, or second-generation
antipsychotics
●Typical or first-generation antipsychotics 

●Psychosocial treatment CBT, individual therapy, group therapy,
family therapy, and social skills training
ADHD ●1. Stimulants
●Non stimulant
●Behavioral Therapy
Autism ●Psychosocial Treatment – Behavioral therapy
Oppositioinal ●Psychotherapy ●Amphetamines in ADHD
defiant
disorder

Conduct ●Psychosocial TreatmentCBT- Family interventions- Inpatient


disorder psychiatric hospitalization
●Psychopharmacotherapy lithium, antidepressants,
carbamazepine, propranolol, stimulants, clonidine, and antipsychotics
Alzheimer ●Cholinesterase inhibitors
disease ●N-methyl-D-aspartate (NMDA) antagonists
●Antipsychotics
 Psychosis, agitation
●Citalopram, Paroxetine, Fluoxetine  Depression, anxiety, and

111
iSummary SHORT NOTES

agitation
●Benzodiazepines  Anxiety
●Trazodone  Insomnia
●Behavioral Treatment
Delirium ●Identify and treat any underlying disorders
●Low-dose antipsychotics 
●Benzodiazepines: Last-line therapy to treat agitation
Parkinson ●Monoamine oxidase-B (MAO-B) inhibitors
disease
●Levodopa/Carbidopa
●Dopaminergic agonist

●Catechol-O-methyltransferase (COMT) inhibitors


●Anticholinergic

Disease Tips
MDD ●Major depressive disorder (MDD) is characterized by persistent
low mood and lack of interest and pleasure over at least 2
consecutive weeks.
●A strategy for the pharmacologic management of MDD is as
follows:

 Identify and treat causes unrelated to MDD (e.g., substance


abuse)
 Use single-agent pharmacotherapy as the first step.
 If there is no satisfactory response after 4 to 6 weeks and an
increase of the dose does not improve the patient's
condition, or if the patient cannot tolerate the first
drugswitch to a different drug that minimize the
troublesome side effects or is from a different medical class.
 If trials of two or three antidepressants are
ineffectiverefer to a psychiatrist for possible
augmentation or other intense treatments.

●Major side effects of SSRIs  gastrointestinal distress, decreased


libido and inhibited orgasm, tremor, insomnia, somnolence, dry
mouth, and small amount of weight gain. For SSRIs, most of the
adverse effects are transient and time limited, and most patients
can tolerate them.
●Major side effects TCAs dry mouth, urinary retention,
constipation, and blurred vision. They are more sedating than most
SSRIs and mixed reuptake inhibitors. Other serious side effects may
include orthostatic hypotension and cardiac conduction
abnormalities (eg, torsade de pointes). TCAs should be avoided in
people with cardiovascular disease, dementia, narrow-angle
glaucoma, and bowel obstruction

112
iSummary SHORT NOTES

●“activating” antidepressants like fluoxetine and bupropion for


patients with hypersomnia.
●Mirtazapine insomnia and anorexia.
●MDD is a recurrent disease: At 1 year following the start of
therapy, 33 % will be free of the disease, 33% will have had a
relapse, and 33% will still be depressed.
●Once the patient is asymptomatic for at least 6 months following a
depressive episode, recovery from the episode is declared.
Dysthymia Diagnostic criteria are as follows 2 of the following symptoms
must be present, most of the day, more days than not, for at least 2
years without interruption of symptoms for longer than 2 months:
●Poor appetite or overeating ●Insomnia or hypersomnia ●Low
energy, fatigue ●Poor concentration ●Low self-esteem
●Hopelessness
Grief or bereavement ●Reaction occurs in response to significant loss or separation.
(death of a loved one, marital separation, loss of a girlfriend or
boyfriend, or a move to a different and unfamiliar location )
The postpartum ●Transient, self-limited mood disturbance occurring within the first
“blues” 2 weeks after delivery.
Postpartum Major ●Onset can begin 24 hours to several months after delivery, lasting
Depression (PMD) several months to the second year postpartum
Bipolar Disorder ● A disorder characterized by manic or hypomanic behaviors that
are sometimes accompanied by a depressive disorder
●The symptoms of a manic episode of bipolar disorder are
expressed in the mnemonic DIG FAST:
 Distractibility. (i.e., attention too easily drawn to unimportant
or irrelevant external stimuli)
 Injudiciousness or Impulsivity (poor judgment—e.g., spending
sprees, sudden travel, sexual indiscretion, reckless driving).
 Grandiosity (↑ self-esteem).
 Flight of ideas (racing thoughts).
 Activities—psychomotor agitation; ↑ goal-directed activities
(e.g., socializing, hypersexuality, ↑ productivity).
 Sleep—↓ need for sleep. (e.g., feels rested after only 3 hours
of sleep)
 Talkativeness—pressured speech. More talkative than usual,
or pressure to keep talking
Adjustment Disorder ● Symptoms of anxiety occur in response to an identifiable stressor
with Anxious Mood within 3 months after the onset of the stressor and resolve within 6
months after the termination of the stressor
Generalized Anxiety ● Unrealistic or excessive worry about several life events or
Disorder activities
● The anxiety must be present for at least 6 months during which
the person has been bothered more days than not by these
concerns.
●Buspirone is not useful in the treatment of acute anxiety, because
it often takes several days to weeks (depending on the patient) to

113
iSummary SHORT NOTES

produce its therapeutic effect


Panic disorder ● Abrupt onset of intense fear that peaks within 10 to 30 minutes
of onset, and is associated with at least four autonomic symptoms,
including palpitations, sweating, trembling, dyspnea, choking,
chest pain, nausea, dizziness, depersonalization, paresthesias, hot
or cold flashes, and fear of dying.
●Diagnosis requires recurrent panic attacks and either 1 month of
behavior change in response to the attacks or persistent worry
about additional attacks or their consequences.
Agoraphobia ● Diagnosis requires the presence of anxiety in situations where
escape is difficult or help is unavailable
Post-traumatic Stress ●Patients must have witnessed or experienced a life-threatening or
Disorder (PTSD) severe injury-threatening event that elicited a response of intense
horror, hopelessness, and fear lasts for more than 1 month
Somatization ●4 pain symptoms: 2 gastrointestinal symptoms, 1 sexual symptom,
disorder 1 pseudoneurological symptom. Also, in order to diagnose
somatization disorder, the complaints must have started <30 years,
have no identifiable organic basis, and not be intentionally feigned
Conversion disorder ●Not intentionally produced ● self-limited symptoms that affect
voluntary motor or sensory systems during a period of psychosocial
stress and suggest a neurologic disorder but are not consistent with
anatomic structures.
Hypochondriasis ● A chronic preoccupation with or fear of having a serious medical
disease for 6 months or longer that is not relieved by appropriate
evaluation or reassurance.
Body dysmorphic ● Chronic preoccupation with an imagined defect in physical
disorder appearance
Chronic pain ● Pain without an identified organic cause is the central feature
syndrome
Malingering ● Motivated by external gain; symptoms are intentional with poor
cooperation in evaluation
Factitious disorder ● Motivated by assumption of the sick role, in which symptoms are
fabricated or self-inflicted. Histories are often vague, and patients
go from hospital to hospital seeking care.
Insomnia ● Insomnia, primary or comorbid with medical, psychiatric, or
neurologic disorders, is the inability to get adequate sleep even
under ideal sleep conditions.
● Insomnia is classified as acute—lasting from 1 night to less than 1
month, or chronic—occurring at least three times a week for more
than 4 weeks.
Restless legs ● RLS is an uncomfortable sensation of needing to move one’s legs
syndrome or arms when sitting or lying still
Periodic limb ● PLMD is the periodic and repetitive flexion of the legs and less
movement disorder frequently the arms that occurs every 20 to 90 seconds for minutes
to hours during sleep. And can lead to awakenings, which are
usually not noticed by the patient.

114
iSummary SHORT NOTES

Narcolepsy ●Narcolepsy is a disturbance in rapid eye movement (REM) sleep


causing periods of excessive daytime drowsiness and a tendency to
fall asleep at inappropriate times. Narcolepsy can be associated
with sleep paralysis, cataplexy, hypnagogic hallucinations, and
disturbed sleep
Eating disorders ● Bulimia nervosaRecurrent binge eating(large amounts) then
Recurrent compensatory acts to prevent weight gain (vomiting,
laxatives, diuretics, enemas, fasting, diet pills, excessive exercise)
● Anorexia Nervosa Intense fear of gaining weight or becoming
fat
● Menstruation usually resumes in women affected with anorexia
when the patient approaches 90% of ideal body weight
● Osteoporosis will not resolve when an adequate body weight is
regained
Substance abuse ● Flumazenil is a benzodiazepine antagonist treatment of
benzodiazepine overdose or the reversal of benzodiazepine sedation
●Verapamil and nitroglycerine  patients with cocaine-
associated chest pain.
●Sodium bicarbonate, benzodiazepines, and lidocaine 
management of cocaine-induced arrhythmias
● Alcohol abuse AST/ALT ratio is often 2:1
Schizophrenia ● Positive symptoms (delusions, hallucinations, disorganized
behavior, disorganized speech); negative symptoms (poverty of
speech, anhedonia, affective flattening, avolition, asociality); mood
symptoms (dysphoria, suicidal thoughts, hopelessness); and
cognitive symptoms (attention and memory deficits and difficulty
with abstract thinking).
● Haloperidol may cause prolongation of the QT interval; as a result,
electrocardiographic monitoring is necessary. Haloperidol should be
avoided if possible in patients with a known seizure disorder
ADHD ● Children ages 6 to 12
● Inattention symptoms have persisted for at least 6 months,
hyperactivity-impulsivity symptoms have persisted for at least 7
months, at 2 or more settings (home, school,)
● Stepwise therapy ●Begin first stimulant (usually
methylphenidate) ●Increase dose gradually ●End of dose failure:
Consider another one of the two recommended stimulants ●Failure:
switch to nonstimulant.
●Length of time and dosing ●Consider early morning and noon
dosing ●Consider "drug holidays" on weekends and vacations. ●Use
for as long as needed.
●Adverse events associated with stimulant use include decreased
appetite, weight loss, headache, sleep disturbances, motor tics, and
slowed growth velocity. Growth typically catches up during
adolescence with discontinuation of medication use.
Atomoxetine, a nonstimulant medicationshould be discontinued
in patients with jaundice or laboratory evidence of liver injury.

115
iSummary SHORT NOTES

Autism ● Presents between one and three years of age


● Early signs are poor eye contact, lack of stranger anxiety, and poor
attachment to parents
● Later signs include peculiar interests, stereotypic behaviors, and
low IQ (generally < 70)
● Most children are brought to their doctors because of poor
speech development.
● Lithium remains the preferred medication for the treatment of pregnant patients with
severe bipolar disease
● TCAs: Not associated with major congenital malformations
● Extensive research has shown no association between major congenital malformations and
fluoxetine
● Paroxetine is contraindicated in pregnancy
● Carbamazepine and valproic acid neural tube defects, neonatal hemorrhage
● ECTConsidered safe and effective during pregnancy.

 Treatment Caveats of anti-depression medications


Drug Treatment Caveats
Tricyclic Antidepressants
Imipramine Obtain electrocardiogram
Amitriptyline Establish treatment dose through blood levels
Nortriptyline Helpful in relieving neuropathic pain.
Doxepin Frequently cause sexual dysfunction.
SSRIs
Fluoxetine[*] Titrate gradually when starting.
Sertraline Taper gradually when stopping.
Paroxetine[*] Manage sexual side effects by weekend holiday (short half-life agents
Citalopram only); addition of bupropion SR 150 mg daily; dose reduction. Paroxetine
Escitalopram has the most anticholinergic activity. Citalopram and its stereoisomer,
escitalopram, have relatively clean profiles with no major
interactions with any of the cytochrome P450 enzymes. Sertraline is
also an attractive option if drug-drug interactions are a concern.
Venlafaxine[*] Titrate gradually when starting.
Taper gradually when stopping.
Manage sexual side effects by weekend holiday (short half-life agents
only); addition of bupropion SR 150 mg daily; dose reduction.
Side effects: Dizziness, dry mouth, sexual disturbance, and can increase
blood pressure.
Bupropion[*] Avoid in seizure disorder patients or those with active eating disorders
Bupropion is associated with vivid dreams and insomnia.
bupropion must be avoided in patients taking MAOIs.
No sexual side effects result
Mirtazapine Few sexual side effects are seen
Low doses are more sedating than high doses
Side effects: Sedation, orthostatic hypotension, weight gain
Duloxetine This is also indicated for certain neuropathic pain problems

116
iSummary SHORT NOTES

*
Available in extended-release forms.

At high therapeutic doses
SSRIs, selective serotonin reuptake inhibitors; S, serotonin transporter; N, norepinephrine transporter; D, dopamine
transporter; 5, 5-hydroxytryptamine (serotonin)2α; M, muscarinic; α, α1-adrenoreceptor (Catecholamine subtype receptors).

 Somatoform Disorders, Factitous Disorder, and Malingering


Symptom Type of Prevalence Voluntary Symptom
Presentation Symptoms and Control Duration
Gender over
Ratios Symptoms
Somatization Sees self as Multiple systems 0.2–2% in No Chronic,
disorder sickly; frequent or functions of females; < recurring
medical care several types, 0.2% in males and/or stable
including pain
and
pseudoneurologic
Undifferentiated Sees self as ill; Single system of Common No More than 6
somatoform frequent symptom months
disorder medical care
Conversion Onset after Single, 0.01–0.1%; No Sudden
disorder acute stress pseudoneurologic females much onset; short
more duration
common
Pain disorder Focus solely on Pain; low back, Common No Sudden
pain; pain neck, pelvic; onset;
behaviors emotional worsens with
changes time
Hypochondriasis Fearful of Multiple; normal 4–9% of No Long history,
disease; bodily sensations; medical worsens after
preoccupied may be vague practices; actual illness
with symptoms; equal
not reassured
Body Excessive Specific ? No Usually
dysmorphic concern about complaints of several years
disorder imagined defect defect
in appearance
Factitious Multiple Nonhealing and Rare Yes Chronic;
disorder with operations; unremitting multiple
physical infections admissions;
symptoms remits with
confrontation
Malingering Protest; demand Vague pain ? Yes Multiple
for medical help and/or paralysis episodes of
common same
problem

117
iSummary SHORT NOTES

 Clinical Features and Clusters of 10 DSM-IV-TR Personality


Disorders
Cluster Personality Clinical Features
Disorder
Cluster A: odd, Paranoid Suspicious; overly sensitive;
eccentric misinterpretations
Schizotypal Detached; perceptual and cognitive
distortions; eccentric behavior
Schizoid Detached; introverted, constricted affect
Cluster B: Antisocial Manipulative; selfish, lacks empathy;
dramatic, explosive anger; legal problems since
emotional, adolescence
erratic Borderline Dependent and demanding; unstable
interpersonal relationships, self-image,
and affects; impulsivity; micropsychotic
symptoms
Histrionic Dramatic; attention seeking and
emotionality; superficial, ie, vague and
focused on appearances
Narcissistic Self-important; arrogance and grandiosity;
need for admiration; lacks empathy; rages
Cluster C: Avoidant Anxiously detached; feels inadequate;
anxious, fearful hypersensitive to negative evaluation
Dependent Clinging, submissive, and self-sacrificing;
needs to be taken care of; hypersensitive
to negative evaluation
Obsessive- Preoccupied with orderliness,
compulsive perfectionism, and control

118
iSummary SHORT NOTES

COMMUNITY
Disease Management
Traveler’s ●The most common of which is enterotoxigenic E. coli (ETEC)
Diarrhea ● Fluid replacement
●Antibiotic1.azithromycin (The CDC no longer recommends
fluoroquinolones for traveler’s diarrhea because of resistance)
● Antimotility agents loperamide, which is safe in the
absence of bloody stools (dysentery)
Occupational ● Respiratory symptoms that occur within 1 hour after work
asthma begins or 6 to 8 hours later are consistent with but not
diagnostic of occupational asthma
●Pre-existing asthma made worse by the workplace exposure is
NOT considered occupational asthma
●Medication inhaled agents such as β-agonists,
corticosteroids, and mast cell membrane stabilizers.
●Noninhaled agents include leukotriene antagonists,
theophylline, and systemic corticosteroids
Hypersensitivity ● Agricultural workers are at particularly increased risk for
pneumonitis these disorders (eg, farmers’ lung, bagassosis,.. )
(extrinsic ● Acute  flulike signs and symptoms within 3 to 8 hours after
allergic exposure to an environmental antigen
alveolitis) ● Subacute and Chronic progressive shortness of breath and
chronic cough
● High-resolution CT ground-glass patchy infiltrates along
with a mosaic pattern in the lower zones of the lung
● Bronchoscopy with brochoalveolar lavage standard in the
diagnosis
●Treatment corticosteroids, oxygen, and bronchodilators
TOXIC ● Workers in textile, grain, livestock, and horticulture industries
PNEUMONITIS are at risk
(Organic ● Symptoms normally occur on the first day of returning to
Agents) work from some period of time off (Monday morning fever)
● flulike reaction (high fever) 4 to 12 hours after the initial
exposure
● Paracetamol or NSAIDs as needed
TOXIC ● Welders and metal trade workers are at increased risk,
PNEUMONITIS inhalation of heated zinc or exposure to fumes of heated
Chemical Agents Teflon, plastics...
● Flulike symptoms
● Anti-inflammatory medication as needed
Acute Chemical ● Caused by irritant gases, organic chemicals, metallic
Pneumonitis compounds, and complex mixtures
● irritation of eyes, nose, throat, cough, hoarseness, wheezing,
and chest pain

119
iSummary SHORT NOTES

● Steroids are used for severe conditions


Asbestosis ● Asbestos mining, asbestos removal, building demolition,
textile and tile manufacturing, shipbuilding, pipefitting, and
application of asbestos fireproofing and insulation ●The highest
prevalence was found among insulation workers
● The latency period for the disease is generally 15 to 20 years.
●Patients present with cough, shortness of breath, thoracic
pain, or hemoptysis
●Diseases associated with asbestos are carcinoma of the lung,
malignant mesothelioma, and benign pleural lung disease
● Only supportive care, as there is no effective treatment
Coal Worker’s ● Occupations including underground mining, face working,
Pneumoconiosis roof bolting, and tunnel drilling are at increased risk of
exposure
● Patients present with productive cough and shortness of
breath with exertion
● Standard bronchodilator therapy may be useful
Silicosis ● Industrial sources of free silica include mining, stonework,
sandblasting, foundry work, and glass manufacturing
● The disease is usually seen after 10 to 30 years of exposure.
● Productive cough
● Eggshell calcification of the hilar nodes is characteristic of
silicosis, End-stage silicosis  fibrotic masses in the upper lung
fields
●Treatment of silicosis is supportive

 The only vaccination required by international health regulations is the yellow


fever vaccine for travel to certain parts of sub-Saharan Africa and tropical South
America. The meningococcal vaccine is required by the Saudi Arabian
government for travel during the Hajj.

 Relative risk The incidence of a disease in exposed individuals divided by the


incidence of disease in unexposed individuals.
 Odds ratio The odds that an individual with a specific condition has been
exposed to a risk factor divided by the odds that a
control has been exposed.
 Three years after smoking cessation, the risk of recurrent MI ↓ to that of a
nonsmoker
 Types of prevention
 1° prevention: Defined as disease prevention measures taken before
disease develops. Includes counseling for at-risk
behaviors, immunizations, and chemoprevention.
 2° prevention: Early detection and treatment of asymptomatic disease,
including screening and risk assessment.
(disease is already present)
 3° prevention: Management of chronic diseases to prevent or minimize
complications.

121
iSummary SHORT NOTES

 Adequate fluoride intake is important. Fluoride supplementation should


continue until age 16 years for adolescents living in areas with inadequate
fluoride in the water supply
 The optimal schedule of health maintenance examinations for adults is unclear.
Most physicians recommend annual or biannual health maintenance visits
 NOF lists the following major risk factors for postmenopausal osteoporosis and
related fractures:
 Personal history of fracture as an adult (especially after age 45 years; not
including fingers, toes, and skull)
 History of fragility fracture in a first-degree relative (especially maternal hip
fracture)
 Low body weight (< 127 pounds - 57.6 kg)
 Current cigarette smoking
 The USPSTF found no evidence that screening for HCV infection in adults at high
risk leads to improved long-term health outcomes
 USPSTF recommend that any woman with a family history suggesting the
possibility of a BRCA1 or BRCA2 gene mutation should be offered genetic
counseling and testing (Recommendation B).

 Screening & prevention in birth: 10 years


 Measure the height and weight of children (B).
 Screen for amblyopia, strabismus, and visual acuity defects in all children
younger than 5 years (B).
 Screen all newborns for hemoglobinopathies, phenylketonuria, and congenital
hypothyroidism (A).
 High-risk children should be screened for anemia when the infant is 6 to 12
months old (B).
 Screen for lead levels at age 12 months (B).
 Do not screen children with urinalysis (D).
 High-risk children should be screened for tuberculosis by a purified protein
derivative (PPD) test (A).
 Infants born to mothers who are positive for human immunodeficiency virus (HIV)
or mothers at high risk for HIV infection but with unknown status should be
screened for HIV immediately after birth (B).
 Antibiotic ointment should be applied to newborn's eyes within 1 hour of birth
(A).
 Fluoride supplementation should be recommended in areas where there is
insufficient fluoride supply in the water (B).
 AAP, AMA Screen asymptomatic children during well-child visits for
hypertension beginning at 3 years of age. (recommended I by USPSTF)
 Adolescents

• Measure the height and weight of adolescents to screen for obesity (B).
• Screen sexually active female adolescents for chlamydial infection (A).
• Screen high-risk female adolescents for gonorrhea (B).

121
iSummary SHORT NOTES

• Screen with Pap smears within 3 years of the onset of sexual activity (A).
• Do not screen asymptomatic adolescents for scoliosis (D).
• Screen for rubella susceptibility in women of childbearing age (A).
 ADULT
 Screen all adults for hypertension (A).
 Do not screen the general adult population for coronary artery disease (D).
 Do not screen adults for peripheral vascular disease (D).
 Screen men between 65 and 75 years who have ever smoked for abdominal
aortic aneurysm (B).
 Screen adults for obesity by means of the body mass index (B).
 Screen men older than 35 years and women older than 45 years for
hyperlipidemia (A). Begin screening for hyperlipidemia at age 20 for those with
other risk factors for heart disease (B).  USPSTF
 The National Cholesterol Education Panel (2001) recommends that all adults
older than 20 years be screened for hyperlipidemia every 5 years
 Screen hyperlipidemic and hypertensive adults for diabetes mellitus (B).
 The American Diabetes Association (2005a, 2005b) recommends screening for
diabetes and impaired fasting glucose beginning at age 45 or sooner for patients
at high risk for diabetes or its complications
 The American Heart Association recommends consideration of screening
beginning at age 20 for patients at increased risk.
 Screen for osteoporosis at age 65 for women of average risk and at age 60 for
women at increased risk (B). They recommend a screening interval of no less than
2 years, and they suggest no age at which to stop screening
 The National Osteoporosis Foundation (NOF) expert panel recommends all
women 65 years of age or older be screened for osteoporosis regardless of the
presence or absence of risk factors. Younger postmenopausal women with one
or more risk factors (other than being white, postmenopausal, and female)
should be screened for BMD
 Do not screen for thyroid dysfunction in asymptomatic patients (D).
 Screen for Chlamydia in all sexually active women younger than age 25 and
continue to screen high-risk women older than age 25 (A).
 Screen all women of childbearing age for immunity to rubella (B).
 Screen all adults for depression, provided that the resources exist to treat
depression after it has been identified (B).
 Screen all adults for alcohol misuse (B).
 Screen all women older than age 40 for breast cancer (B).
 UPDATE  The USPSTF recommends biennial screening mammography for
women aged 50 to 74 years
 Screen all sexually active women with a cervix for cervical cancer and its
precursors. Begin screening within 3 years of the onset of sexual activity or by age
21 (A).
 Do not continue to screen for cervical cancer and its precursors in previously
screened, low-risk women older than 65 years or in women who have undergone

122
iSummary SHORT NOTES

a hysterectomy for benign disease (D).


 Screen all patients older than age 50 for colorectal cancer (A). They make no
recommendation for the use of any particular screening strategy.
 Discuss the risks and benefits of and the gaps in scientific knowledge regarding
prostate cancer screening with all men older than 50 years (I).
 Do not screen for ovarian, testicular, pancreatic, or bladder cancer (D).
 Question elderly adults about hearing difficulties (B).
 Screen elderly adults for decreased visual acuity (B).
 Counsel patients who use tobacco to quit (A).
 Counsel patients who are at increased risk for diet-related diseases regarding
healthy eating and weight loss (B).
 Discuss the risks and benefits of aspirin prophylaxis for patients > 40 YEARS at
increased risk for heart disease (A).
 Discuss the risks and benefits of tamoxifen or raloxifene for women at
significantly elevated risk for breast cancer (B).
 Recommend against the use of β-carotene by smokers (D).
 Recommend against the use of hormone replacement therapy for the prevention
of chronic disease (D).
 Immunize adults against tetanus and diphtheria every 10 years.
 Immunize all adults older than 50 years and younger patients with risk factors
against influenza annually.
 Immunize adults older than 65 years against pneumococcus with a single
immunization.
 Immunize adults without a history of infection or vaccination against varicella.

 Contraindications to the DPT vaccine include the following:

 Previous anaphylaxis to the vaccine


 Moderate or severe illness
 Previous encephalopathy within 7 days after DPT injection
 Progressive neurologic problem that is undiagnosed
 Fever higher than 105°F (40.5 C) after previous DPT
 Continuous crying lasting 3 hours or more after previous DPT
 Seizure within 3 days after previous DPT
 Previous collapse, limp, or pale episode with previous DPT

123
iSummary SHORT NOTES

Audit cycle: first step is to set standard

124
iSummary SHORT NOTES

 DEMOGRAPHIC PRINCIPLES AND MEDICAL STATISTICS

Mortality statistics

 Crude death rate

 Neonatal mortality rate

 Infant mortality rate

 Post neonatal mortality rate

 Children mortality rate

 Under 5 mortality rate

 Cause specific mortality rate from a specific disease

 Maternal mortality ratio

 Maternal mortality rate

125
iSummary SHORT NOTES

( )

 Proportionate death rate

 Case fatality rate

Fertility statistics

 Crude birth rate

 General fertility rate

 Fecundity rate

 Age specific fertility rate

( )
( )

126
iSummary SHORT NOTES

 Test parameters

127
iSummary SHORT NOTES

SURGERY
Disease Management

Cyclical breast pain ●Reassurance


●Medicationsacetaminophen and ibuprofen – Danazol -
Tamoxifen - Bromocriptine and gonadotropin-releasing hormone
agonists - primrose oil - low-dose estrogen and 19-nor progestins
Hyperthyroidism ●Thionamides  and or )
●β-Adrenergic antagonists( ) or diltiazem and
verapamil/ Clonidine as alternatives
●Radioactive iodine (I-131)
●Surgerysubtotal thyroidectomy
Thyroid storm ●High doses of PTU ●High doses of β-blockers ●Hydrocortisone -
●Potassium iodide (after the initial administration of antithyroid
drugs)
Hypothyroidism ●Levothyroxine
Subacute thyroiditis (de ●NSAIDs for pain
Quervain thyroiditis) ●Prednisone If no resolution of pain occurs within 1 week
●β-adrenergic antagonists symptomatic treatment.
● Thyroid replacement  if hypothyroidism develops.

Radiation/ trauma-induced ●NSAIDs or prednisone  pain and


thyroiditis ●β-adrenergic antagonists symptomatic relief.

Acute suppurative thyroiditis ●Appropriate antimicrobial therapy with possible surgical drainage.

Painless or lymphocytic ●Beta-blockers hyperthyroid symptom


thyroiditis (silent thyroiditis ●Levothyroxine for clinical hypothyroidism.
or, postpartum thyroiditis).

Riedel thyroiditis (Fibrous ●Glucocorticoids or other immunosuppressive agents. ●Tamoxifen


thyroiditis) ●Surgical intervention relieves compression of the esophagus and
trachea.
Thyroid nodules ●Malignant thyroid nodules Surgical excision
●Benign thyroid nodule  levothyroxine (suppressive doses)
●Very large nodulessurgery
Anal fissure ●Pain local anesthetics ( 2% jelly) , topical
corticosteroid
●Nifedipine 0.3% with lidocaine ointment 1.5%, nitroglycerin
ointment, diltiazem gel
●Surgery lateral internal sphincterotomy
●Botulinum toxin A  in chronic fissure
Anorectal abscesses ●Incision and drainage
●Antibiotics immunocompromised or high fever, valvular heart
disease, extensive cellulitis, or diabetes
Anorectal fistulas ●Fistulotomy
●Chronic fistulas TTT of underlying cause e.g., Crohn's disease
Anal warts, polyps ●Topical podofilox, topical 5% 5-fluorouracil (5-FU), interferon

128
iSummary SHORT NOTES

injections ●Cryosurgical destruction, electrosurgical ablation, surgical


excision, or laser ablation
Pruritus ani ●Behavioral ●Short course of topical steroids ( 1%)
●Intradermal injection of 30 cc of 0.5% methylene blue intractable
idiopathic cases
Thrombosed external ●1. symptomatic measures analgesics, stool softener ●2 Excision
hemorrhoids
Internal hemorrhoids ●Grade 1,2,3 Analgesics, Stool softener(docusate sodium), sitz
bath,  water or fluid, Psyllium or methylcellulose powder, short
course of topical hydrocortisone
●Rubber band ligation(1,2,3) ●Surgical excision(3,4)
DVT/PE ●Oxygen
●Heparin Unfractionated heparin or Low-molecular-weight
heparin (for 5 days)
●Warfarin
●Thrombolytic agents extensive embolism and/or hemodynamic
instability.
Superficial thrombophlebitis ●Heat, elevation, and NSAIDs

Peripheral vascular disease ●Behavioral Therapy Smoking cessation, Exercise


●Antiplatelets drugs Aspirin or Clopidogrel
●Cilostazol, Pentoxifylline, Ginkgo biloba
●Statins, ACEI
●Percutaneous transluminal angioplasty
●Arterial bypass surgery (Gold standard)
Varicose veins ●Reassurance, stockings ●Avoid standing for prolonged periods and if
standing, don't stand still; walk regularly; ↓ weight (if obese).
●Any complications or severe symptoms Refer for surgical
assessment
Urolithiasis ●Pain Narcotics or NSAIDs ●CCB and alpha blockers
facilitate passage of stone
●Shock wave lithotripsy (ESWL)stone is less than 2 cm in diameter
●Ureteral stones 1.Wachful waiting 2.●ESWL for upper 2/3
●Endoscopic surgery lower stones
●Renal stones ●ESWL for stones less than 2 cm and located in the
upper pole of the kidney ● combination of ESWL and percutaneous
nephrolithotomy If stones larger than 1 cm are located of the lower
pole of the kidney
Hematemesis ● Resuscitation Vital signs, Intravenous access, Crystalloids (normal
saline or Ringer solution) or fresh frozen plasma until blood is
available. Blood (in elderly patients and those intolerant of hypoxia),
IV reduce the risk for rebleeding when endoscopy is also performed.
● Nasogastric (NG) tube no longer recommended as a first-line
intervention, If endoscopy is to be scheduled in the next several
hours, then NG placement is not necessary
Solitary intraductal ●Ages of 30 and 50 ●Surgical removal
papillomas ●Bloody nipple
discharge
●Palpable mass that is <
1 cm but that can be as
large as 4 to 5 cm

129
iSummary SHORT NOTES

●Rarely increase the


patient's risk of
developing breast
cancer
Fibrocystic breast ●25% of premenopausal ●No treatment is necessary
changes (FBD) women and up to 50% unless the woman is
(Mammary dysplasia) of postmenopausal. symptomatic
●Cyclical pain ●Supportive measures –
●Bilateral, in the upper Calcium – weight loss if
outer quadrants, a few BMI<30
days prior to ●Primrose oil
menstruation,  with ●Oral contraceptives (most
the onset of menses, effective treatment)
and may be associated ●Danazol (FDA approved)
with an increase in ●Tamoxifen
breast size and yellow- ●Thyroid replacement if TSH
green breast discharge. is 
● Cysts are smooth, ●Bromocriptine and
regular, rubbery, and luteinizing
easily movable lumps or ●Hormone-releasing agents
areas of local ●Thiazide diuretics
tenderness without a ●Surgery (subcutaneous
discrete mass. Cysts can mastectomy; oophorectomy)
range in size from 1 mm
to many centimeters.
Compression causes
tenderness. Larger cysts
are more common as
women age.
Fibroadenoma ● Most common in ● Aspiration of the ● FNA biopsy or an excisional
Breast mouse women younger than 25 mass should be biopsy is both diagnostic and
years attempted. curative
● Painless mass ● A fine-needle
● A well-defined, biopsy may be
rubbery, mobile, diagnostic
nontender, completely
round, 1 to 5 cm mass
can generally be
palpated in an upper
quadrant.
Asymptomatic ●Discovered incidentally ●Ultrasonography ●Cholecystectomy once
cholelithiasis symptoms begin, for patients
with a calcified (“porcelain”)
gallbladder or who undergo
laparotomy for unrelated
conditions
Acute cholecystitis ●RUQ or epigastric ●RUQ ●Intravenous antibiotics,
tenderness and ultrasonography analgesics and bowel rest
guarding ●Abdominal ●Cholecystectomy
●Murphy sign radiographs
●Palpable Gallbladder (10:15% visible)

131
iSummary SHORT NOTES

20% - Jaundice 20% ●HIDA scan


Acute acalculous ●RUQ pain and ●Ultrasonography ●Cholecystectomy
cholecystitis tenderness ●Cholescintigraphy
(HIDA scan)
Chronic cholecystitis ●History of multiple ●RUQ ●Dissolution therapyoral
episodes of RUQ pain, ultrasonography bile acids such as or
absence of RUQ ●Abdominal  poor operative
tenderness, and radiographs candidates or who decline
gallstones on (10:15% visible) surgery
ultrasonography ●CT and oral ●Elective cholecystectomy
●Murphy’s sign is not cholecystography
present (OCG)
Choledocholithiasis ● Charcot triad consists ●Elevated serum ●Preoperative ERCP with
and acute of RUQ pain, bilirubin, alkaline sphincterotomy
cholangitis fever/chills, and phosphatase, and ●Percutaneous transhepatic
jaundice. transaminases drain
●Ultrasonography ●Cholecystectomy(later)
(common bile duct
dilation >8 mm in
diameter)
●MRI
cholangiopancreato
graphy (MRCP)
●Endoscopic US
●ERCP (gold
standard)
●Intraoperative
cholangiography
Diverticulosis ●Usually asymptomatic. ●Barium enema ●High-fiber diet;
mild tenderness in the segmental spasm bulking agents (psyllium);
LLQ and muscular education and reassurance;
thickening that ●Surgical resection for
narrows the lumen, massive hemorrhage or to
giving it a rule out carcinoma
sawtoothed
appearance
Diverticulitis ●Acute onset of LLQ ●AXR Reveals ●Expectant
pain free air in the managementNPO, NG tube
presence of a placement, IV fluids. Broad-
perforation. Shows spectrum antibiotics with
ileus and partial colonoscopy 4–6 weeks after
obstruction +/− LLQ the acute attack.
mass. ●Surgical management for
●CT The best perforation or failure of
initial study expectant management;
●May reveal ●Severe cases laparotomy
pericolic fat and possible colectomy
stranding, an
abscess, or a fistula.
●Barium enema:
Contraindicated

131
iSummary SHORT NOTES

Sigmoid volvulus ●Intermittent cramp ●AXR A single, ●Endoscopic decompression


like pain and greatly distended ●Emergent operation (partial
obstipation. loop of bowel or total colectomy)
●Abdominal distention “megacolon” Performed if strangulation or
●Barium enema perforation is present or if
“bird’s beak” or attempts at decompression
“ace of spades” are unsuccessful.
deformity
Testicular cancer ●20-40y ●Ultrasound ●Orchiectomy
●Hard, heavy, firm, exploration through
nontender testicular an inguinal incision
mass that does not
transilluminate
Testicular torsion ●Adolescent boys and ●Testicular Doppler ●Surgical exploration within 6
young men ultrasonography hours
●Acute pain and a firm, ●Orchiopexy
tender mass-unilateral
usually during activity
absence of the
cremasteric
Torsion of the ●Scrotal pain ●Ultrasound ●Conservative therapyrest,
appendix testis NSAIDs, ice or surgical
excision
Cancer bladder ● Hematuria(most ●
common) cystourethroscopy,
●Flank pain, suprapubic urine cytology, and
pain, hypogastric pain, CT with urography
and perineal pain or IVP with renal
●Irritative or ultrasound
obstructive voiding
symptoms

 Breast lump

 A negative mammogram should never preclude biopsy of an appropriate


palpable lesion. Although 85% of all breast cancers are documented by
mammography, as many as 15% of women with breast cancers have a
normal mammogram.
 Limitations include very dense breasts, such as in young women,
breast-feeding women, and those on hormone replacement therapy,
for whom both sensitivity and specificity are reduced
 Greater than 90% of palpable breast masses in women between 20 and 55
are benign.
 A woman with a clinically suspicious lesion should undergo mammography
and/or ultrasound, and biopsy. Between 9% and 22% of palpable breast
cancers will not be detected with imaging studies, with lobular carcinoma
particularly difficult to detect on mammography.
 Characteristics of a mass that are concerning for malignancy include:

132
iSummary SHORT NOTES

 Single lesion, hard, fixed/immobile, irregular border, and size greater


than 2 cm.
 Any asymmetry, skin dimpling, nipple discharge, and lymphadenopathy
must also be assessed.

 Fibrocystic changes are the most common benign condition of the breast.
There is no increased risk of cancer in women with fibrocystic changes in a
woman younger than 50 years unless proliferative or hyperplastic lesions
with atypical epithelial cells are present on biopsy.
 Fibroadenoma, the most common solid benign breast tumor

 Breast cysts

 Ultrasound may be used to differentiate solid versus cystic masses and


aspiration may be both diagnostic and therapeutic.
 Cysts should be surgically biopsied if they contain bloody fluid, fail to
resolve completely after drainage, or recur after 4 to 6 weeks. It is not
necessary to send aspirated fluid for cytologic examination, however.
 Nonpalpable cysts identified during routine mammography do not require
further evaluation or treatment.

 Nipple discharge

 Characteristics of pathologic discharge include unilaterality; presence from


a single duct; association with an underlying mass; spontaneous,
intermittent, and persistent occurrence in a postmenopausal woman; and
bloody to serosanguinous color.
 The characteristics of the fluid may aid in diagnosis:
 Green, black, creamy, or mucoid  FBD.
 Straw-colored  papilloma
 Bloody or serosanguinous  malignancy, but may also represent
bleeding papilloma or fibrocystic change with an intraductal
component.
 Bloody or brown-green discharge  old blood, should be investigated
further.
 Cheesy  duct ectasia.
 Purulent  mastitis.
 Gynecomastia appearing during mid-to-late puberty requires only a history
and physical examination, including careful palpation of the testicles and, if
the results are normal, reassurance and periodic follow-up. In most boys,
the condition resolves spontaneously within a year and no further
evaluation is necessary.
 Gynecomastia associated with pain, asymmetry, rapid onset or progression,
galactorrhea, and/or erectile dysfunction requires further workup.
Association with precocious puberty is also a concerning sign.

133
iSummary SHORT NOTES

 Breast cancer

 The lifetime probability that a woman will develop invasive breast cancer is
13.36%; breast cancer will eventually develop in one of every nine or seven
women.
 BRCA1 and BRCA2 genes are responsible for 10% of breast cancers, and
women who have these mutations have a cumulative risk of developing
breast cancer up to age 70 years of 55% to 85%.

 Mammography is the major screening tool for early detection of breast


cancer in women over 40.
 When a mass is detected by breast self-exam, CBE, or mammography,
definitive diagnosis depends on tissue sampling.
 The most important factor in prognosis of patients with breast cancer is the
axillary node status.
 If cancer is found during the first or second trimester, a mastectomy and
axillary dissection should be carried out. Third-trimester patients can be
observed until delivery and then receive prompt therapy.

 Hyperthyroidism

 Graves's disease is the most common cause of hyperthyroidism in iodine-


sufficient areas usually affecting women of reproductive age.

 Atrial fibrillation is the most common complication and can occur in up to


20% of patients of hyperthyroidism, some of whom have no other
symptoms of hyperthyroidism.
 After confirmation of the hyperthyroidism by the TSH and T4 tests,
radioactive iodine uptake is the next most appropriate test to narrow the
differential diagnosis of the hyperthyroidism.
 TSI (thyroid-stimulating immunoglobulin) or TRab (thyroid receptor
antibody) (the most specific) are positive in 80% of patients with Graves
disease and are diagnostic of this disorder
 The RAIU is elevated in Graves' disease, toxic adenomas, toxic multinodular
goiters, TSH-secreting pituitary adenomas, metastatic follicular thyroid
carcinomas, and trophoblastic tumors.
 The RAIU is low in subacute thyroiditis, lymphocytic thryoiditis, exogenous
hyperthyroidism, recent iodine load (contrast dye, diet), and struma ovarii.
Low RAIU disorders are the cause of hyperthyroidism only 5% of the time.
 Ultrasonography may be used to evaluate thyroid nodules and goiters.
Graves' disease may demonstrate an altered blood flow that can also be
demonstrated via ultrasound.
 The two basic treatments for Graves' disease are antithyroid drugs (PTU or
methimazole) and radioactive iodine therapy.

134
iSummary SHORT NOTES

 For adults the treatment of choice is radioactive iodine, for children and
adolescents antithyroid drugs remains first-line treatment, with radioactive
iodine being second-line treatment.
 Propylthiouracil is preferred for pregnant patients. Surgery is often
considered in the second and third trimester of pregnancy if reasonable
doses of antithyroid drug therapy are not working.
 Treatment duratuion of hyperthyroidism with Thionamides (propylthiouracil
and methimazole or carbimazole 12 to 18 months and then a trial
discontinuation is initiated. After initiation of therapy, testing free T4 and T3
levels every 4 to 6 weeks until stable is indicated
 Monitor patients for relapse every 4 to 6 weeks for the first 3 to 6 months,
and then every 3 months for the first year following cessation of the
antithyroid drugs (ATD). If the patient remains euthyroid, annual monitoring
is continued indefinitely. If relapse of hyperthyroidism occurs, alternative
therapy is recommended.
 If you have a patient on PTU with a sore throat, check a CBC.
Agranulocytosis is the most serious side effect
 Radioactive iodine is contraindicated in women who are pregnant (because
it can ablate the gland of the fetus in utero), plan to become pregnant
within 6 months, and those who breastfeed.

 Hypothyroidism

 The most common cause of primary hypothyroidism is Hashimoto


thyroiditis (due to an antibody and cell-mediated destruction of the thyroid
gland).
 If the hypothyroidism is due to Hashimoto thyroiditis, antithyroid
peroxidase antibodies (positive in 95% of patients) are present in the
serum.
 Most healthy adult patients with hypothyroidism require about 1.6 mcg/kg
of thyroid replacement, with requirements falling to 1 mcg/kg for the
elderly.
 The serum TSH level is the most important measure to gauge the dose, and
a free T4 estimate may be included as well.
 The dosage can be increased if necessary after analyzing thyroid levels 6 to
8 weeks after the initial dose. If the TSH level remains elevated, the dose is
increased. If the TSH level is suppressed, the dose is decreased.
 Once the TSH level reaches the normal range (0.3 to 2.5 mIU/L), the
frequency of testing can be decreased. Each patient's regimen must be
individualized, but the usual follow up after TSH is stable is at 6 months
 Subclinical hypothyroidism
 Treatment with levothyroxine is recommended if the TSH levels are
greater than 10 μ per L.
 If TSH levels are between 4.5 and 10 μ per L, no firm guidelines direct
treatment. Patients can follow thyroid hormone levels every 6 to 12
months for an elevation in TSH levels.

135
iSummary SHORT NOTES

 Treatment of subclinical hypothyroidism in pregnancy is generally


recommended.
Overt hypothyroidism Low T4, high TSH
Subclinical hypothyroidism Normal T4, high TSH
Hypothyroidism secondary to Low T4, normal or borderline low TSH
hypopituitarism
Euthyroid Normal T4, normal TSH
Subclinical hyperthyroidism Normal T4, low TSH
Hyperthyroidism High T4, low TSH
Isolated T3 hyperthyroidism Normal T4, low TSH, High T3

 Thyroid nodule

 Solitary thyroid nodules are malignant in approximately 10% of these cases.


 A fine-needle aspiration biopsy (FNAB), TSH, and free T4 and T3 testing are
recommended as initial diagnostic tests in most adults presenting with a
thyroid nodule.
 After RAIU study, Cold nodules require further investigation because they
carry a 5% to 15% risk of malignancy. Hot nodules unlikely to be malignant
and do not require FNA.
 In children younger than 13 surgical excision is currently the mainstay of
treatment.

 Thyroid cancers

 Papillary carcinoma is the most common form of thyroid cancer; it has the
best prognosis
 Follicular carcinoma is less likely to have regional spread but is more likely
to be more aggressive and to have distant metastases. Prognosis is slightly
worse
 Anaplastic thyroid carcinoma has the worst prognosis
 60% to 70% of patients with “silent stones” never develop symptoms. The
risk of developing biliary colic or other complications is about 1–2% per year
(this rate decreases over time). Therefore, the incidental finding of
asymptomatic gallstones should generally not prompt surgical referral,
except in:
 Cases in which an elevated risk for gallbladder carcinoma exists
(gallstones associated with a calcified, or "porcelain," gallbladder).
 Possibly those with stones > 3 cm).
 10% of persons with gallstones develop acute cholecystitis.
 HIDA scan is test of choice in patients who have gallstones seen on US but
atypical presentation, or in patients with typical presentation but
nondiagnostic US.
 Pregnant patients with gall stones should generally be treated
conservatively, because most attacks will abate after the birth of the infant.

136
iSummary SHORT NOTES

However, in severe attacks or in acute cholecystitis, laparoscopic


cholecystectomy is safe. The best time to perform surgery is during the
second trimester, thus avoiding potential teratogenic effects during the first
trimester and preterm labor in the third trimester.
 Approximately 25% of persons with gallstones eventually develop chronic
cholecystitis.

 Variceal Bleeding  Noninvasive pharmacologic treatment aimed at


causing splanchnic vasoconstriction and thus reducing portal pressure
(vasopressin, somatostatin, and propranolol). Recurrent bleeding may be
prevented by periodic endoscopic sclerotherapy. Propranolol given twice
daily (at a dose that reduces the heart rate by 25%) decreases portal
pressure, although the response is nonuniform.
 All patients except those at low risk, as well as those with continued active
bleeding, should have endoscopy within 12 hours after resuscitation and
stabilization. In stable patients, the gold standard confirmation of UGIB is
upper endoscopy
 colonoscopy as the preferred modality for the evaluation and treatment of
LGIB
 Anal Fissures off the midline (lateral) should prompt consideration of other
medical disorders such as inflammatory bowel disease, anal carcinoma,
acquired immune deficiency syndrome (AIDS), tuberculosis, occult
abscesses, leukemic infiltrates, herpes, or syphilis. Screening laboratories
may be helpful in these instances.
 Hemorrhoids You should consider further evaluation (eg, flexible
sigmoidoscopy, colonoscopy, etc) in patients <50 years who present with
rectal bleeding, even if hemorrhoids are present and are the likely source of
bleeding. In patients >50 years with rectal bleeding, a full colonoscopy is
routinely recommended to rule out any cancerous process.
 Appendicitis pain on flexion of the hip (psoas sign), pain on internal
rotation of the hip (obturator sign), or pain on the right side when pressing
on the left (Rovsing’s sign). None of the above signs is specific for
appendicitis.

 DVT

 D-Dimer not specific for DVT, useful to rule-out DVT


 Duplex venous  the first-line noninvasive diagnostic procedure. If an
initial ultrasound study is negative and the clinical suspicion of DVT is high
 repeat study should be obtained 5 to 7 days later. If results from the
second study are equivocalvenography can be used.
 Contrast venography  The gold standard

137
iSummary SHORT NOTES

 Pulmonary embolism

 V/Q scan: Ventilation-perfusion scanning is the test of choice in diagnosing


PE in patients with normal chest x-rays and absence of underlying
pulmonary disease.
 Spiral CT with contrast effective for the diagnosis of large, central
pulmonary embolisms. Can detect peripheral thrombi up to fifth-order
branches
 Pulmonary angiography The gold standard

 HEPARIN
 Unfractionated heparin (UFH): Administered intravenously. Drawbacks
include the need for hospitalization and the risk of thrombocytopenia
from heparin.
 Low-molecular-weight heparin: Suitable for lower-risk patients in place
of unfractionated heparin. Can be used at home without the need for
monitoring. LMWH is as effective as UFH and is preferred initial
therapy for DVT. Use of LMWH may be limited in the case of renal
insufficiency.
 LMW heparin is the agent of choice for treating DVT in pregnant
women and patients with cancer.
 LMW and UFH are both acceptable for prevention of DVT and PE in the
postoperative period. The optimal length of time that patients require
prophylaxis for venous thromboembolism after surgery is not known.
Aspirin and low-dose warfarin (target INR 1.5) are also effective but less
so than heparin or enoxaparin.
 The major side effect of heparin is bleeding. If needed, protamine
sulfate may be administered to rapidly reverse heparin's anticoagulant
effect; in most cases, this measure is unnecessary and the
discontinuation of heparin is adequate

 WARFARIN
 Treatment duration is typically six months for a first episode when
there is a reversible risk factor; 12 months after a first-episode
idiopathic thrombus; and 6–12 months to indefinitely in those with
recurrent disease or nonreversible risk factors

 Peripheral vascular disease

 ABI (ratio of the systolic ankle blood pressure to the systolic brachial artery
pressure) ●ABI is 0.9 to 1.3  normal ●ABI <0.9 PAD ●ABI
<0.4 pain at rest and ulceration ●ABI <0.2 ischemic gangrenous
extremities.
 Angiography and magnetic resonance angiography (MRA) are considered
the gold standard as diagnostic tools and in the planning for
revascularization.

138
iSummary SHORT NOTES

 Lymphadenopathy

 Patients with lymphadenopathy can be observed for 3–4 weeks unless there
is a suggestion of malignancy (eg, fever, night sweats, weight loss, age
greater than 40 years, hard texture, fixed lymph nodes, and supraclavicular
location). A 3–4 week delay makes no difference in patient outcome if the
node does turn out to be malignant.
 Certain clinical features suggest the need for an early biopsy. These features
include:
 Diameter of greater than 2 cm.
 Hard and fixed consistency.
 Lack of pain or tenderness on palpation.
 Patient age older than 40 years.
 Abnormal chest x-ray result (e.g., adenopathy or infiltrate).
 Associated signs and symptoms (e.g., weight loss or
hepatosplenomegaly).
 Absence of upper respiratory tract symptoms.
 Enlargement of a supraclavicular node, or a cervical node in a smoker.
 During follow-up for undiagnosed lymphadenopathy, nodes that remain
constant in size for 4 to 8 weeks or fail to resolve in 8 to 12 weeks
should be biopsied.
 A bone marrow examination is indicated for patients with severe anemia,
neutropenia, thrombocytopenia, or a peripheral smear with malignant blast
cells.

 Urolithiasis

 The most sensitive test for the diagnosis of metabolic abnormalities


associated with nephrolithiasis is the 24-hour urine collection for
analyzing of calcium, uric acid, citrate, oxalate, sodium, creatinine, and
urine pH
 The current gold standard for diagnosis of a kidney stone is helical
computed tomography (CT) scan without contrast.
 Intravenous pyelogram (IVP) is the next best choice; however, it is
neither as sensitive nor as specific, and it is often a slower test.
 Kidney-ureter-bladder (KUB) radiography will not demonstrate the
radiolucent stones such (uric acid, cysteine, and magnesium ammonium
phosphate [struvite]) stones.
 Renal ultrasound and IV urography may be helpful for radiopaque stones
(calcium oxalate and calcium phosphate) and pregnant women.
 Size is the single best predictor if a stone will pass, Stones ≥6 mm will
pass spontaneously only 10% of the time and those 4–6 mm 50% of the
time. Stones <4 cm pass the great majority of the time. Stones in the
distal ureter are also more likely to pass than those proximally.
 Prevention
 Moderate calcium intake (1 g/day)

139
iSummary SHORT NOTES

 Restriction of oxalate in the diet. Low sodium diet


 Increased fluid intake (Water and Lemonade and orange juice)
 Reduce meat, fish, and poultry consumption
 Hypercalciuria is an indication for prophylaxis with thiazide diuretics
 Patients with uric acid stones respond to urinary alkalinization with
potassium citrate or sodium bicarbonate
 Hypocitraturia may be treated with potassium citrate.
 Cysteine Urine alkalinization with potassium citrate or potassium
bicarbonate. Na+ restriction

 Pregnancy Stones will often pass secondary to ureters dilation

 Scrotal complaints

 Testicular trauma produces acute testicular pain and swelling similar to that
associated with torsion or infection. However, if the pain following trauma
lasts more than 1 hour, one must consider the possibility of trauma-induced
torsion.
 A right-sided varicocele or suddenly appearing left-sided varicocele requires
further evaluation because of the possibility of venous obstruction or renal
carcinoma. In such cases, an intravenous pyelogram or renal
ultrasonography is indicated.
 Whenever a testicular malignancy is suspected, exploration should be
conducted through an inguinal incision. Transscrotal biopsy may cause
spillage of tumor into the scrotum and areas of lymphatic drainage.
 Most hydroceles and cystic lesions do not require surgical therapy, but the
patient should be instructed to return if the enlargement becomes
uncomfortable or interferes with intercourse.

 Classification of Burn

 First degree (Superficial burns): Red, warm, painful tissue involving the
epidermis that blanches with pressure. They usually take 3 to 6 days to heal
without scarring. Sunburn is a classic example of this type of burn.

 Second degree (Partial-thickness burns): Red, wet, painful tissue with or


without blisters involving the epidermis and portions of the dermis.

 Third degree(Full-thickness burns): Dry, insensate, waxy, leathery tissue


with or without overlying blisters involving the epidermis, the dermis, and
possibly underlying subcutaneous fat tissue, muscle, and bone. It is usually
painless but the patient complains of pain, which is usually a result of
surrounding second-degree burns.

141
iSummary SHORT NOTES

 UGIB Triage

 Discharge: Healthy patients with normal hemodynamics, an aspirate that


clears with NG lavage, and normal labs may be discharged. Endoscopy
should be scheduled as an outpatient procedure. The risk of rebleeding is
low if the patient presents < 48 hours after the event.
 Admit to the hospital: Everyone else must be admitted for observation
and/or endoscopy.
 ICU admission: Active bleeders, those with > 5 units of blood loss, and those
with advanced liver disease or serious comorbidities must be admitted to
the ICU.

141
iSummary SHORT NOTES

ENT
Disease Management
Otitis externa ●Clean out exudates and debrisby gently suctioning or
swabbing
●Acetic acid drops or antibiotics and a steroid drops,
drying agents, alcohols
●Pain control topical anesthetic (benzocaine, antipyrine,
and dehydrated glycerin), or acetaminophen, ibuprofen
● Systemic antibiotics if infection spreads to the concha
or to the preauricular or infra-auricular area
● Fluoroquinolone if perforation to tympanic membrane
● Cotton wicks for a severely swollen EAC

Otomycosis ● Thorough cleaning of the ear canal


● Topical antifungal clotrimazole, miconazole, nystatin
Cerumen impaction ● Cerumenolytic ear drops followed by irrigation
Chronic Suppurative ● Removing the debris
Otitis Media ● Amoxicillin clavulanate ± ciprofloxacin
● Topical otic suspensionneomycin, polymyxin B, and
hydrocortisone or gentamicin otic drops
● Frequent suctioning and intravenous antibiotics for 6
weeks
● Tympanomastoid surgery
Bell's palsy ● Antivirals acyclovir or valacyclovir
●Steroids prednisone
●Eye protectionMoisturizing eye drops and nightly
lubrication
Vasomotor rhinitis ● Ipratropium bromide ●Intranasal steroids, oral
decongestants, Intranasal antihistamines ,regular nasal
saline washes, or a combination of these
Rhinitis ● Stop topical decongestant
medicamentosum ● Oral decongestants or a short course of a topical nasal
steroid
● short course of systemic steroids
Atrophic rhinitis ● Guaifenesin + intranasal saline ● Systemic estrogens

Epistaxis ● 1.Assessment of hemodynamic stability


● Sustained compression of the nose with the patient
leaning forward for 5 to 20 minutes
● Vasoconstrictors  oxymetazoline or phenylephrine
● Topical anesthetic lidocaine or cocaine
● Chemical cautery if easily visualized bleeding vessels
● Silver nitrate sticks no > 5 minutes

142
iSummary SHORT NOTES

● Persistent anterior bleeding nasal packing coated


with topical antibiotic
Nasal Polyps ● Intranasal steroids ●Systemic steroids ● Surgical
removal if sever obstruction or recurrent sinusitis

Disease Diagnosis Investigati Management


on
Mastoiditis ●Fever, pain, erythema, aural ●Sever ●Ceftriaxone ±metronidazole
discharge, and swelling mastoid ●Surgical drainage (for a
behind the ear. tenderness, subperiosteal abscess)
●Symptoms appear 2 or 3 lateral
weeks after an episode of displacement
acute suppurative OM of the pinna,
and
postauricular
mastoid
swelling
Meniere's ● Recurrent episodes of ● Salt restriction, avoidance of
disease vertigo (20min:24h), caffeine and alcohol
fluctuating low-frequency ●low-dose hydrochlorothiazide
Sensorineural hearing loss, ●Antiemetics.
tinnitus, and aural fullness in ●Benzodiazepine
the affected ear ●Surgical decompression of
the endolymphatic system
for patients who do not
respond to medical
management.
Vestibular ● Current RTI ● Rest, reassurance, and
neuronitis ● Spontaneous sever vertigo antiemeticsdroperidol,
(24-48h) with associated chlorpromazine, or
ataxia and nausea and dimenhydrinate
vomiting. ●Corticosteroids
●There is no hearing loss,
aural pain, or other
symptoms
Benign ● Dix-Hallpike maneuver ● Repositioning maneuvers
paroxysmal nystagmus
positional ●Positional vertigo for 1-2
vertigo min
Labyrinthitis ● Follows otitis media or an ● Rest
URTI ●Antibioticsif the etiology is
● Features are similar bacterial
vestibular neuronitis, except ● Amoxicillin ± clavulanic
 significant Sensorineural acid prophylaxis if serous
hearing loss and sever labyrinthitis
vertigo that lasts several ● Surgical drainage

143
iSummary SHORT NOTES

days.
●Fever may accompany the
illness.
Sinusitis ● Start with URI symptoms ● CT scan ● Initial treatment 
● Sinus pain/pressure, imaging conservative measures and
maxillary toothaches, nasal modality of decongestants. cool steam
obstruction, high fever, choice and  oral intake of water,
headache, halitosis, Oral/topical decongestants,±
hyposmia /anosmia, nausea, Guaifenesin
and vomiting. ●Antibiotic1. Amoxicillin,
● Purulent rhinorrhea TMP/SMZ, Doxycycline
● Clarithromycin,
azithromycin, or quinolones
for penicillin/cephalosporin
allergies
●Poor response amoxicillin-
clavulanate and quinolones
● Chronic sinusitis
cloxacillin, cephalexin,
cefadroxil, erythromycin,
clarithromycin, amoxicillin-
clavulanate, and cefuroxime
axetil for 3 weeks
Allergic ● Rhinorrhea, sneezing fits, ● Allergen skin ● Environmental control
rhinitis pruritus of the nose and testing ●1. Steroid nasal sprays
eyes, nasal congestion, and a ● ● Antihistamines (oral/nasal)
sensation of "sinus pressure" Radioallergoso ●Topical Decongestant
● Nasal crease rbent (RAST) ● Antihistamine–decongestant
testing combinations
● Mast cell–
stabilizing(cromolyn
sodium)pregnancy
● Anticholinergic agents
(ipratropium)
● Nasal saline and
montelukast
● Immunotherapy

144
iSummary SHORT NOTES

 Comparative Features of Peripheral Vestibular Disorders


Feature BPPV Meniere's Vestibular Perilympha
Disease Neuronitis tic Fistula
Hearing No Low frequency Ultrahigh Variable
loss initially frequency
(greater than
8kHz) or none
Tinnitus No Yes Predominantly Variable
no
Type of Positional Spontaneous Spontaneous Variable
vertigo
Duration of 1-2 min 20 min to 24h 24-48h Variable
vertigo
Physical Positive Dix- Nystagmus Nystagmus 66% of
examinatio Hallpike test present in acute present in acutepatients have a
n stage only stage only positive fistula
test
Treatment Repositioning Diuretic as Diazepam during Bed rest
maneuvers; maintenance acute stage; initially;
surgery if therapy; vestibular surgery if
dizziness diazepam exercises 72h dizziness
persists (Valium) for after disease persists
intractable onset
vertigo
BPPV, benign paroxysmal positional vertigo.

 Otitis externa

 Pseudomonas aeruginosa is the most common pathogen


 The polymyxin, neomycin, hydrocortisone preparation and the
aminoglycosides have the potential of being ototoxic and should not be
used unless the tympanic membrane is intact.
 Cotton wicks After 48 to 72 hours, the wick can usually be removed, with
continuation of drops for the full 7 to 10 days.

 Otitis media

 Streptococcus pneumoniae (25% to 50%), nontypable Haemophilus


influenzae (15% to 30%), and Moraxella catarrhalis (3% to 20%) are the
most commonly pathogens of acute otitis media.
 Influenza vaccine has been shown to decrease the number of cases of AOM
in immunized patients compared to controls (SOR A)
 A diagnosis of AOM requires:
 History of acute onset of signs and symptoms.

145
iSummary SHORT NOTES

 The presence of middle ear effusion (MEE).


 Signs and symptoms of middle ear inflammation erythema of the
tympanic membrane OR otalgia

 The presence of MEE is indicated by any of the following:


 Bulging of the tympanic membrane (confirmed by Tympanometry.
Pneumatic otoscopy)
 Limited or absent mobility of the tympanic membrane
 Air-fluid level behind the tympanic membrane
 Otorrhea

 Otitis media most often begins with a URI, and as many as 93% of children
with AOM have typical symptoms of URI.
 TympanocentesisThe gold standard for diagnosis of a MEE
 If treating with antibiotic  amoxicillin is still the drug of choice

Features Treatment
Low-risk patients
Older than 6 years, no antimicrobial therapy within Amoxicillin 40-50 mg/kg/day
past 3 months, no otorrhea, not in daycare, and in divided doses for 5 days
temperature <38° C (<100.5° F)
High-risk patients
Younger than 2 years, in daycare, treated with Amoxicillin 80-90 mg/kg/day
antimicrobials within past 3 months, otorrhea, or in 2 divided doses for 10 days
temperature >38° C (>100.5° F)

 If failure of initial antibiotic therapy:


 Amoxicillin-clavulanate: 90 mg/kg/day of amoxicillin, with 6.4 mg of
clavulanate in two doses for 5 to 10 days
 Ceftriaxone: 50 mg per kg (up to 1 g IM) daily for 3 days, and If the child
is vomiting.

 Treatment after Failure on Amoxicillin/Clavulanate


 Ceftriaxone: 50 mg/kg/day up to 1 g IM in a single dose for 3
consecutive days if the child is vomiting or unable to keep down oral
medication.
 Clindamycin: 8-25 mg/kg/day in three or four doses for 5 to 10 days

 For patients with a penicillin allergy



 Cefdinir: 14 mg/kg/day in one to two doses, (to a maximum of 600
mg/day)
 Cefpodoxime: 10 mg/kg/day in one dose (to a maximum of 400
mg/day)
 Cefuroxime: 30 mg/kg/day in two doses (to a maximum of 100 mg/day)

146
iSummary SHORT NOTES

 Azithromycin: 10 mg/kg/day on day 1 and then 5 mg/kg/day for 4 days


in a single dose or 10 mg/kg/day every 24 hr for 3 days
 Clarithromycin: 8-25 mg/kg/day in three or four doses for 5 to 10 days
 Ceftriaxone: 50 mg/kg/day (up to 1 g) for 3 consecutive days, IM or, if
vomiting, IV
 Sulfamethoxazole/trimethoprim: ml susp/10kg (up to 20 mL) per dose
PO bid for 5 to 10 days

 Antibiotic prophylaxis if >3 episodes in 6 months or 4 episodes in 1 year


 Amoxicillin 20 mg/kg/day for 3 to 6 months is the first choice
 sulfisoxazole 50 mg/kg/day sensitive to penicillin or who have failed
amoxicillin prophylaxis

 Otitis media with effusion

 OME  persistent middle-ear fluid without pain, fever, or redness of the


tympanic membrane.
 Resolution of MEE can take up to 3 months, so scheduling a return visit
sooner than 4 weeks is not cost effective.
 Otitis media with effusion is best diagnosed with pneumatic otoscopy
 If otitis media with effusion persists for 3 months,
 If the hearing test shows ≥40 dB in the affected earimmediate
referral to an otolaryngologist is recommended.
 If the decibel loss is between 21 and 39 (mild hearing loss)the
physician has the option of surgery referral or continuing to monitor
the patient at 3- to 6-month intervals.
 A hearing loss of ≤20 dB repeated hearing evaluation and assessment
for MEE in 3 to 6 months.
 The results of hearing tests also must be communicated to the referral
physician.
 Persistent MEE in children younger than 3 years without risk factors
does not require immediate insertion of PE tubes (pressure-
equalization) because developmental outcomes do not differ when the
procedure is postponed for 6 months
 children at risk for speech, language, or learning problems (Down
syndrome, autism, visual impairment, cleft palate, developmental
delays, and permanent hearing loss independent of MEE)  intervene
even before 3 months
 Antibiotics, decongestants, antimicrobials, and corticosteroids not
recommended for routine management of OME

 The most common cause of SNHL in adults is presbycusis; the most common
cause of CHL in adults who have normal-appearing tympanic membranes is
otosclerosis. Asymmetric SNHL requires an MRI to rule out retrocochlear
pathology (vestibular schwannoma [acoustic neuroma]), which requires further
treatment.

147
iSummary SHORT NOTES

 Drugs: Cisplatin, aminoglycoside antibiotics, and loop diuretics can all cause
hearing loss, as can salicylates (eg, aspirin) and some of the other NSAIDs (eg,
ibuprofen, diflunisal) and chloroquine.

 The distinguishing feature between Bell's palsy and CNS lesions (e.g., strokes,
tumors) is that Bell's palsy involves the entire face (including muscles of the
forehead), whereas CNS lesions tend to affect the face below the eyes and
other areas including the arms and legs.
 CANCER OF THE LARYNX presents with Hoarseness. Sore throat and referred
otalgia can exist without hoarseness
 Any patient with an ulcer that is not significantly improving within 3wk. of
presentation to exclude malignancy.

 Rhinosinusitis

 Acute sinusitis symptoms lasting 4 weeks.


 Subacute sinusitis symptoms lasting 4 to 12 weeks.
 Recurrent acute sinusitis  > 4 episodes of acute sinusitis per year lasting
at least 7 days with complete resolution of symptoms between bouts.
 Chronic sinusitis persists for ≥3 months
 The organisms most commonly implicated in acute bacterial sinusitis
include Streptococcus pneumoniae, Haemophilus influenzae, and
Moraxella catarrhalis.
 Rhinitis medicamentosum Abusers of topical nasal decongestants (the
most popular being oxymetazoline) are at risk (decongestant used more
frequently than every 3 hours or for longer than 3 weeks)
 CT scanning is indicated when:
 Medical therapy has failed to establish the diagnosis of chronic
sinusitis in equivocal cases before starting long-term antibiotic therapy
or when complications are suspected.
 For patients whose symptoms fail to respond to antibiotics or who
have recurrent infections. In such patients, a normal computed
tomography scan is particularly helpful in avoiding repeated courses of
antibiotics.
 In patients suspected of having intracranial or intraorbital
complications, but these are rare complications.
 Antibiotics for acute sinusitis should be reserved for patients with
 Persistent rhinorrhea.
 Daytime cough lasting more than 10 to 14 days without improvement
after initial treatment.
 Severe illness (severe facial pain, fever with purulent nasal discharge,
periorbital swelling), regardless of duration. (SOR B )

 The most common bleeding disorder associated with epistaxis is von Willebrand
factor (vWF). Packing removal should occur 3 to 5 days after placement.
Posterior bleeding that does not respond to anterior nasal cavity packing is
considered an otolaryngologic emergency

148
iSummary SHORT NOTES

149
iSummary SHORT NOTES

OPHTHALMOLOGY
Disease Management
Corneal ●Foreign bodies lodged within the cornea a TB or 25-gauge
abrasion and needle, an attempt can be made to remove it by irrigation
foreign body ●Foreign body removed with a cotton-tipped applicator or
25-gauge needle
●Atropine, homatropine, two drops in the affected eye to
reduce pain symptoms from ciliary spasm.
●Oral nonnarcotic analgesics or topical NSAIDs for pain relief
●Topical antibiotic solution reduces the risk of secondary
infection and is recommended for all abrasions

Strabismus ●Esotropia is more serious and often requires surgery


●Exotropia  initially be treated with patching and exercises
but may also need surgical correction
● Accommodative esotropia corrected with glasses

 Ophthalmia Neonatorum
Disease Diagnosis Management
1.Chlamydial ● Mild unilateral or ● Tetracycline ointment or
Infection bilateral mucopurulent erythromycin
conjunctivitis with ● Alternative oral
moderate lid edema, erythromycin or doxycycline for
chemosis, and conjunctival 3 weeks.
injection. ● Both parents treated with
●Systemic involvement oral tetracycline or azithromycin
rhinitis, vaginitis, and
otitis media.
2.Gonococcal ● Swollen lids, purulent ● Referral to an ophthalmologist
conjunctivitis exudates, beefy red is critical.
conjunctiva, and
conjunctival edema.
● Corneal perforation
● Less common
3.Bacterial ● Chemosis, purulent ● A topical fluoroquinolone
Conjunctivitis discharge, lid edema, and before culture results
injection ●Gram-negative organisms
●Associated systemic tobramycin or a topical
septicemia can occur fluoroquinolone.
●Cultures should be ● Mild conjunctivitis respond
prepared on blood and to erythromycin or bacitracin
chocolate agar. ointment
● Systemic antibiotics when

151
iSummary SHORT NOTES

there is evidence of systemic


disease.
Acute ● Pain, tearing, redness, ● Systemic antibiotics for the
Dacryocystitis and discharge acute stage.
● Culture testing and Gram ● The ophthalmologist should be
staining ifchild is febrile, consulted immediately
Chronic ● Chronic history of tearing ● Topical fluoroquinolone
Dacryocystitis and crusting with a chronic ● The mother should be taught to
yellow discharge compress or massage the lacrimal
sac 4 to 6 times a day
● If failure referral and the
nasolacrimal duct system
irrigated between 6 and 10
months of age.
●Before age 14 months, a single
probing is curative in about 90%
of cases.
Congenital ● Excessive tearing, ● Immediate consultation 
Glaucoma photophobia, intense Surgical treatment
blinking or lid spasm ● follow by an ophthalmologist
● Enlarged cornea or for the rest of their lives
corneal clouding
● Corneal
edemablindness

Disease Diagnosis Management


Blepharitis ●Staphylococcal ● Lid hygiene
blepharitis ●Staphylococcal blepharitis 
asymptomatic initially, lid scrubs with dilute baby
then, foreign body shampoo and topical bacitracin,
sensation, matting of the erythromycin, or sulfacetamide
lashes, and burning. Lid
crusting, discharge,
redness, and loss of lashes
are observed
●Seborrheic blepharitis
seborrhea of the scalp,
lashes, eyebrows, and ears.
greasy, dandruff-like scales
on the lashes.
Stye (Acute ● Acute boil-like lesion ● Warm compresses for 15
Hordeolum) ● Swollen, tender, red minutes 4 times /day
eyelid ●Topical antibiotics (not
effective)
●Systemic antibiotics  not
indicated unless there is a

151
iSummary SHORT NOTES

preseptal cellulitis component.


●Drainage using a needle may
be helpful.
Chalazion ● Chronic (weeks or ● Frequent warm compresses
months) swelling of the and massage
eyelids not associated ● Intralesional injection of
with conjunctivitis steroids or incision and drainage
● Rubbery, cystic, and is warranted if persistent >4:6
nontender on palpation. weeks
●When the upper lid is ● Recurrent chalazia biopsied
involved, vision is often and sent for pathologic testing.
temporarily blurred

Bacterial ● Severe purulent ● Tobramycin ointment


Conjunctivitis dischargeculture of the ● Ciprofloxacin and ofloxacin
conjunctiva is mandatory ● Gatifloxacin and moxifloxacin
● Subconjunctival (better)
hemorrhage ●Referral if not improve within
2 to 3 days or the worsening
symptoms
Viral ● Less prominent discharge ● Symptomatic treatment
Conjunctivitis that is usually watery artificial tears 4–8 times per
● Palpable preauricular day, cool compresses, and strict
lymph nodes hygiene
●Referralif persists or there is
any pain or change in vision

Allergic ● Itching, redness, and ● Environmental control


Conjunctivitis swelling in both eyes ● Cold compresses, saline
●Clear discharge irrigation, and ocular lubricants
4:8 times daily
● Symptomatic treatment 
artificial tears, and OTC
antihistamines.
● Topical antihistamine-
decongestant combinations
● Cromolyn sodium 4% and
olopatadine hydrochloride
●Topical NSAIDs
● Corticosteroids severe cases
Subconjunctival ● Bright red eye, normal ● No therapy, except reassurance
Hemorrhage vision, and no pain that the blood will clear within 2
● History of coughing, to 3 weeks
sneezing, or straining ●Referral if trauma is
before the hemorrhage is suspected
present

152
iSummary SHORT NOTES

Primary Herpes ●Clear watery discharge, ●Referral if dendrites are


Simplex skin vesicles on the lids, present
Infection and preauricular nodes.
●Associated vesicles and
ulcers on the oral mucosa
and skin
●Corneal involvement also
may occur with single or
multiple dendrites
●A lesion at the tip of the
nose involvement of the
cornea
Orbital Cellulitis ● Pain on movement of the ● Emergent CT scanning to
eye, conjunctival edema, rule out orbital cellulitis.
and limited extraocular ●If orbital cellulitis is
movements. diagnosedimmediate
●Cultures should be hospitalization with IV AB and
obtained from the ophthalmologic consultation
nasopharynx, conjunctiva,
and blood
Iritis ● Redness, pain, and ● Initial treatment dilation
photophobia. with homatropine and
●No discharge, pupil is loteprednol etabonate for
constricted. patient comfort
●Circumcorneal (ciliary) ●Immediate referral to an
injection may occur. ophthalmologist
●Intraocular pressure is
normal or low.
Angle-Closure ● Severe ocular pain in one ● Ophthalmology consult is
Glaucoma eye, redness, blurred urgently indicated
vision, rainbow-colored ●Initial emergent treatment to ↓
halos around lights, and IOP topical beta blockers, IV
sometimes abdominal pain, and oral carbonic anhydrase
nausea and vomiting inhibitors (e.g., acetazolamide),
●Red, tender, inflamed, α2-adrenergic agonist (eg,
pupil mid-dilated and brimonidine, apraclonidine)and
poorly reactive, and hyperosmotic agents
intraocular pressure ● Topical glycerin clear the
markedly  corneal edema
●On palpation, the involved ● Peripheral laser iridectomy
eye often feels significantly ● Other eye should be treated
harder than the uninvolved prophylactically with a laser
eye iridotomy

Keratitis ● Eye pain, photophobia, ● Referral to all cases


tearing, and blurred vision ● If bacterial gatifloxacin or
● Perilimbal redness with moxifloxacin

153
iSummary SHORT NOTES

slight focal whitening of the ● If viraltopical trifluridine


cornea and/or oral valacyclovir
●± contact lens
Episcleritis ● Sore, red, and tender ● All cases referred to determine
eye. Photophobia if uveitis is involved
● Diffuse or localized ●No treatment, self-limited,
injection of bulbar resolve spontaneously in 1 or 2
conjunctiva ●Freely mobile weeks
nodule may be present ● If pain is a factorartificial
●± associated uveitis tears and/or topical
corticosteroids
Scleritis ●Sudden severe boring ● Urgent ophthalmologic
ocular pain consultation
●Photophobia and ● Systemic corticosteroids are
watering are prominent.↓ mandatory
vision may be seen. ● Immunosuppressive agents are
●Complete physical is used in severe cases
mandatory
●Ocular injection is often
bluish red, tenser globe,
inflammation may be
segmental or diffuse
Uveitis ● Sudden, rapidly ●Urgent ophthalmologic
progressive, painful ocular consultation
injection with severe ● Topical corticosteroids
photophobia, Lacrimation is ● oral prednisone  in severe
prominent cases
● ↓ vision may be present, ● Mydriatics  to relieve pain
± floaters
● The pupil on the effected
side is smaller,
●Circumcorneal Redness
● Lack of purulent
discharge is important
Primary open ● Early asymptomatic ●1. Beta blockers, Prostaglandins
angle glaucoma ● IOP > 21 mmHg ● Adrenergic agonists, Carbonic
● Optic nerve cup-to-disk anhydrase, Cholinergic agonists
ratio is >0.5 ● Surgery (Laser surgery
(trabeculoplasty) or Open
trabeculotomy or destruction of
the ciliary body) if medical
therapy is ineffective at or
compliance is poor

154
iSummary SHORT NOTES

 Red Eye Differential Diagnosis


Parameter Conjunctivitis, Iritis Keratitis Acute
Bacterial Glaucoma
Vision Normal Blurred Blurred Marked
blurring
Pain None Moderately Sharp, severe Severe;
severe; sometimes
intermittent nausea and
stabbing vomiting
Photophobia None Moderate Moderate Moderate
Discharge Usually significant None None to mild None
with crusting of
lashes
Conjunctival Diffuse Circumcorneal Circumcorneal Diffuse
injection (surrounding
the cornea)
Appearance Clear Clear Cloudy Cloudy
of cornea
Pupil size Normal Constricted Normal Dilated
Intraocular Normal Normal or low Normal Elevated
pressure Caution—do not
measure with
discharge present

 Inequality of the pupil may indicate iritis, with which the affected pupil is
typically partially constricted. In acute angle-closure glaucoma, the pupil is
usually partially dilated and may not be round
 intraocular pressure  in iritis and traumatic perforating ocular injuries,  in
acute angle-closure glaucoma
 Ophthalmia Neonatorum  caused by chemical conjunctivitis, Neisseria
gonorrhoeae, and chlamydial infection
 Bacterial Conjunctivitis in children The most common gram-positive bacteria
that are causative agents of conjunctivitis include Staphylococcus aureus
 Chronic Dacryocystitis Approximately 80% of these inflammations resolve
spontaneously by 6 months of age.
 Staphylococcal blepharitis most common inflammation of the external eye.
 Generally, the stye drains spontaneously within several days. If resolution does
not occur with 2 weeks, the patient should be referred.
 A hordeolum (stye) affects the anterior lid margin glands which become acutely
plugged. A Chalazion affects the posterior lid margin glands which become
plugged and chronically inflamed.

 Topical steroids or antibiotic-steroid combinations for conjunctivitis or other


causes of red eye should not be used unless the patient is under the care of an

155
iSummary SHORT NOTES

ophthalmologist. Topical corticosteroids have four potentially serious ocular


side effects and are contraindicated for conjunctivitis:

 Steroids can facilitate penetration of an undetected corneal herpetic


infection to the deeper corneal layers and cause corneal perforation.
 Prolonged local use of the corticosteroids (usually longer than 2 weeks)
can cause chronic open-angle glaucoma.
 Prolonged use of topical corticosteroids can cause cataracts.
 Topical corticosteroids are capable of potentiating the development of
fungal corneal ulcers.

 Management of Gonococcal conjunctivitis consists of Gram stain of purulent


material to document gonococcus, culture, ceftriaxone IM/IV (or spectinomycin
if cephalosporin-allergic), eye irrigation, and treatment for possible concurrent
chlamydia infection.

 Viral Conjunctivitis Adenovirus is most common cause of conjunctivitis. It is


self-limiting, usually lasting 1 to 3 weeks. The condition is highly contagious, and
handwashing is important to avoid infection. Topical steroids are
contraindicated.
 Orbital cellulitis, most commonly caused by an extension of infection from the
ethmoid sinus. It is the most common cause of exophthalmos in children.

 Risk factors for corneal ulcerspatients with corneal erosions, persistent
epithelial defects, impaired immunologic mechanisms, contact lenses, chronic
topical or systemic steroid use, diabetes, and alcoholism.
 Acute glaucoma should always be suspected in a patient who is older than 5o
years and has a painful red eye.
 Up to one third of patients with Episcleritis have systemic collagen vascular
condition
 Scleral inflammation  frequently part of a systemic immune-medicated
collagen vascular disease
 Patients with episcleritis do not have the severity of pain noted with scleritis,
and the inflammation and erythema are generally not as intense.
 Uveitis may have systemic collagen vascular disease associated
 The first, and most important, step in ocular trauma is to look for evidence of an
open or ruptured globe
 Corneal abrasion and foreign body ciliary injection is a warning sign that a
deep penetration may have taken place, and an ophthalmologic consultation
should be sought immediately. A purely localized conjunctival inflammation
pattern is generally associated with superficial foreign bodies
 Patching may also be used PRN for patient comfort but is unnecessary and
may increase pain and increase healing time. It may increase the risk of
infection.
 Topical anesthetics should be avoided if no foreign body is present because
they delay corneal epithelial healing

156
iSummary SHORT NOTES

 If symptoms resolve in 48 hours, no follow-up is necessary. If symptoms


persist, re-examination is indicated to rule out retained foreign body or
infection.

 Contact Lens Overwear


 Contact lens overwear is managed similarly to corneal abrasion. Patients
need reassurance that the condition is usually not serious, even though the
pain is severe.
 Patients should be seen the next day and referred if they have not
improved. Contact lens wear may be resumed only after the corneal
epithelium is well healed.
 Metallic foreign bodies in the cornea may leave a rust ring around the area of
the foreign body which can be removed using an ocular burr
 Chemical and thermal burns  Immediate irrigation with normal saline (or
water if saline is unavailable) is the initial step, Any particulate matter should be
removed prior to irrigation if it is a reactive substance Patch the affected eye
and refer the patient to an ophthalmologist
 Cataract There is no strict visual acuity which determines the appropriate
timing of cataract surgery. If there are significant lifestyle limitations secondary
to visual disability from a cataract, then cataract surgery is indicated.

 Primary open angle glaucoma


 POAG causes gradual, asymptomatic loss of peripheral vision in a
characteristic pattern, followed by central vision loss if untreated.

 Strabismus
 Intermittent esotropia may occur in normal infants up to 5–6 months of
age. To confirm suspected strabismus, check the following:
 Light reflex: The corneal light reflex from a penlight held along a toy
that the child focuses on should appear symmetric. If there is an
esotropia, the corneal light reflex will appear to be temporal in one eye.
If there is an exotropia, the reflex will appear to be nasal in one eye.
 Cover test: In an abnormal test, when the dominant eye is covered, the
weaker eye will move to focus on an object.
 Any form of strabismus may result in vision loss, and for this reason, it
is important to treat strabismus early in life.
 Any unexplained new onset strabismus mandates an evaluation.

 Amblyopia: Amblyopia is subnormal visual acuity in one or both eyes despite


correction of refractive error.
 Indications for treatment of thyroid ophthalmopathy include diplopia,
abnormal head position, a large horizontal or vertical strabismus, and loss of
vision
 Prophylactic laser photocoagulation for diabetics with proliferative diabetic
retinopathy.
 Age-related macular degeneration

157
iSummary SHORT NOTES

 Central vision is impaired, whereas peripheral vision remains intact. These


patients should screen their central vision daily with an Amsler grid.
 Limited treatment exists for the atrophic type; early laser photocoagulation
surgery may delay or reverse visual loss in the exudative type.
 Patients with the acute onset of flashes or floaters should be referred to an
ophthalmologist.
 Seeing flashes of light is most consistent with signs and symptoms of retinal
detachment

 In central retinal artery occlusion, there is sudden painless profound loss of


vision to hand movements or no light perception. The patient may have had
previous episodes of amaurosis fugax (loss of vision in one eye due to a
temporary lack of blood flow to the retina) with fleeting blindness lasting only
seconds.
 Central or branch retinal vein occlusion causes a sudden painless decrease in
visual acuity.

158
iSummary SHORT NOTES

DERMATOLOGY
Disease Diagnosis Management
Tinea barbae ● Facial hair of adult ● Oral antifungalsgriseofulvin,
males terbinafine, or itraconazole) are all
used long enough, sometimes up to
4 weeks
Tinea capitis ● Scalp and hair shafts ●Oral antifungals1. Oral
● black dot alopecia griseofulvin for 4 to 6 weeks.
●Otheralternative antifungal
ketoconazole, itraconazole,
terbinafine, and fluconazole
Tinea corporis ● Trunk and extremities ● Topical antifungalmiconazole,
● Lesions are annular clotrimazole, ketoconazole,
with central clearing and itraconazole and the allylamines 
a scaling border and may naftifine, terbinafine twice daily
be pruritic for a minimum of 2 weeks
Tinea pedis ● Interdigital ● Topical antifungalterbinafine is
better than imidazole clotrimazole
● Oral antifungals  in refractory
infections
Tinea cruris ● Groin ● Topical antifungal
● low-dose corticosteroid for the
first few days
● Oral antifungals  in refractory
infections
Tinea versicolor ● Hypopigmented or ● Topical agentsselenium sulfide,
hyperpigmented sulfacetamide sodium, and ciclopirox
macules and patches ● Topical antifungalsazoles, and
on the chest and the allylamines
back
Tinea unguium ● Nail bed, matrix, or ● Ciclopirox
(Onychomycosis plate ● AntifungalTriazole, allylamine
) ● Toenails are affected > ● Terbinafine, itraconazole, and
fingernails fluconazole
Diaper ● Sparing in the folds ● Topical antifungalclotrimazole,
dermatitis econazole, ciclopirox, miconazole,
Intertrigo ● Dark moist areas, such ketoconazole, and nystatin
as the groin or fat folds ● Oral antifungals  in extensive
infections, immunocompromised
patients
Herpes simplex ● HSV-1fever, sore ● Oral antivirals, including acyclovir,
throat and valacyclovir , and famciclovir
submandibular or ● Suppressive therapy in patients

159
iSummary SHORT NOTES

cervical with recurrent genital herpes


lymphadenopathy. ● Cesarean delivery if active
Painful vesicles on the lesions are present at the onset of
lips, oral mucosa, or labor
tongue. The lesions
ulcerate and heal within
2 to 3 weeks
● HSV-2 painful
vesicular lesions on
penis, anus or perineum;
cervix and on the
external labia bilaterally,
vagina, perineum,
buttocks in women
● The diagnosis of
genital herpes is best
established by viral
culture
Herpes zoster ● Pain and parasthesias, ● Pain and pruritus  cool water
(shingles) then appearance of compresses, calamine, Analgesics
small groups of vesicles ● Antiviralacyclovir, valacyclovir,
on an erythematous and famciclovir within 3 to 4 days
base in a dermatomal of rash
distribution. The rash
rarely crosses the
midline and is usually
confined to a single
dermatome.
● Lesions usually resolve
in 2 to 3 weeks.
● If lesions appear on
the tip of the
nosesuspect Corneal
involvement
● postherpetic neuralgia
Verruca (warts) ● Facial warts ● Flat ● Most warts spontaneously regress
warts ● Plantar warts over a period of many months to
● Condylomata (STI) years
in intertriginous areas ● Cryotherapy, topical fluorouracil
and on mucous cream, keratolytic agents or
membranes, commonly podophyllin, and laser ablation
in the genital and rectal ● Genital HPV infections
areassmall papules, cryotherapy, podophyllin, or
Vulvar and anogenital imiquimod for external lesions
condylomata generally
exhibit a cauliflower-like
appearance

161
iSummary SHORT NOTES

Molluscum ● Asymptomatic, firm, ● Resolve spontaneously over


contagiosum smooth, round papule months or years with no residual
with central scarring
umbilication ● Lesion destruction mechanical
● In children, on the (curettage, laser, or cryotherapy with
extremities, trunk, and liquid nitrogen or nitrous oxide
face. cryogun), chemical (trichloroacetic
●In sexually transmitted acid, tretinoin), or immunologic
cases, on the lower (imiquimod)
abdomen and in the
genital region
Impetigo ● Red macule or papule, ● Topical antibiotics 1.Mupirocin
then into a fragile vesicle ●Oral antibioticscephalosporin,
or pustule that becomes semisynthetic penicillin, or beta-
a honey yellow crusting lactam–beta-lactamase inhibitorif
papule, with minimal extensive impetigo or with refractory
surrounding erythema infection.
● Typicallyaround the ●Tetracyclines, trimethoprim
nose and mouth as well sulfamethoxazole, or clindamycin 
as other exposed body if MRSA
surfaces. The palms and
soles are spared.

Erysipelas ● Fiery red, indurated, ● Penicillin orally or intramuscularly


tense, and shiny plaque ● Erythromycin or azithromycin if
with sharply allergy to penicillin
demarcated margins ● Hospitalization for severe cases
and for infants, older adult patients,
and immunocompromised patients
●Facial erysipelas  dicloxacillin or
nafcillin
●Surgical débridement only in
severe infections with necrosis or
gangrene

Cellulitis ● Red, hot, swollen, and ● Oral antibiotics in


tender area of skin. the immunocompetent hosts
borders are not ● Intravenous antibiotics Severe
elevated nor sharply cases or patients with comorbid
demarcated conditions such as cardiac, renal, or
hepatic failure or
immunosuppression
Lymphangitis ● Erythematous linear ● Afebrileoral antibiotics, close
streaks extend from the follow up
primary infection site ● Fever, chills, and myalgias 
toward draining regional parenteral therapy
nodes ● Analgesics and anti-inflammatories

161
iSummary SHORT NOTES

Folliculitis ● Yellowish pustules at ●Antibacterial soap


the bases of hairs, ●If persist  topical or systemic
particularly on the scalp, antibiotic
back, legs, and arms ●If a pseudomonal infection is
● Usually self-limited. suspected and lesions persist >5 days
without treatmentciprofloxacin
should be considered
Furuncles and ● Furuncles, or boils ●Drainage of the lesion.
carbuncles more aggressive ●Antibioticif the furuncle is not
forms of folliculitis yet fluctuant, there is evidence of
painful, fluctuant surrounding cellulitis or
swelling in a non– lymphadenitis, or the lesion is on the
weight-bearing area, face.
most commonly areas of ●Carbuncles Surgical drainage and
friction, the nasal area, resection of the lesions are often
or the external ear. necessary
● Carbuncle collection
of furuncles and usually
occurs on the back of
the neck
Erythrasma ● Intertriginous spaces ● Oral erythromycin, plus daily
● Skin color changes in cleansing with soap and water
the infected area. The ● Keep the affected area dry to
reddish brown and may prevent recurrence
be slightly raised, with
some central clearing
Scabies ● small pruritic papules, ● Permethrin 5% below the neck
pustules, and burrows ● All family members should be
●Lesions are intensely treated, regardless of the presence
pruritic, especially at or lack of symptoms
night
● Classically located in
the web spaces of the
hands. It is also common
in the axillae,
antecubital fossa, gluteal
crease, feet, genitalia,
nipples, and waistband
Pediculosis ● May affect the head, ● Permethrin 1% cream rinse. Oral
body, or genital region agents  ivermectin and TM–SMZ
● asymptomatic or may ● Removal with a gloved hand or a
present with pruritus fine-toothed comb
● Diagnosed by direct ●All household members with active
visualization infestation should be treated
simultaneously
●Manual removal only  in children
< 2 years

162
iSummary SHORT NOTES

Psoriasis ● Well-demarcated ● Topical treatment topical


erythematous plaques corticosteroids, a vitamin D analog
covered by waxy silvery- (calcipotriene), topical retinoids.
white scales. ● Systemic treatment
● Bilateral involvement Phototherapy (UVB),
of the extensor surfaces, photochemotherapy (PUVA), oral
scalp, palms, and soles. retinoids, methotrexate,
Sometimes pruritic cyclosporine, biologicals (efalizumab,
● Nail pitting is seen in infliximab).
half of cases ●Guttate psoriasis Penicillin VK
or erythromycin is used to treat strep
throat
Pityriasis rosea ● Herald patch then ● Spontaneous resolution usually
generalized rash, occurs in 1–2 months.
“Christmas tree” ● Topical or systemic steroids and
distribution antihistamines for itching
Lichen Planus ● Purple, polygonal, ● Moderate- to high-potency topical
pruritic papules steroids
affecting the flexor ● Oral lesions steroid-containing
surfaces, mucous paste
membranes, and
genitalia
● Associated with
chronic HBV and HCV
● self-limited
ACNE ROSACEA ● Erthyema, edema, ● Topical solutions of metronidazole,
papules and pustules, sodium sulfacetamide with sulfur,
and telangiectasia and azelaic acid gel
● central face area ● Tretinoin cream ± topical
nose, forehead, antibiotics in papular or pustular
cheeks, and sometimes lesions
periorbital and eyelid ● Oral antibioticstetracycline,
areas doxycycline, erythromycin, and
minocyclinefor nodular rosacea or
in ocular involvement.
● Oral clonidine and beta-blockers
 in flushing associated with
rosacea
● Vascular laser therapy and light
therapy in telangiectasias
● Broad-spectrum sunscreen should
be applied daily
●Referral if ocular rosacea is
suspected or response to topical and
oral therapy is inadequate
ATOPIC ● Itch that rashes ●1. chronic treatmentprevent

163
iSummary SHORT NOTES

DERMATITIS ● Younger infants, the excessive drying with short, less


cheeks and neck are frequent baths and showers,
involved. As they begin nonirritant soaps and lotions, and
to crawl, their extensor frequent moisturizing
surfaces are involved. ● Elimination of exacerbating factors
The diaper area, (foods, Airborne allergens...)
because it is moist, is ●Acute flares potent topical
not usually involved. corticosteroid for 7 to 10 days. Oral
● In older childrenthe corticosteroids for severe flares
flexural areas, like the not responding to topical therapy
antecubital and popliteal ●Mid-potency steroid ointments
fossae, are involved. (triamcinolone) the mainstay of
●AD tends to worsen in pharmacotherapy for AD.
the dry winter months ● Antihistamines control pruritis
● PUVA or UVB phototherapy if
resistant to conventional measures
● Topical immunosuppressants
tacrolimus for a limited time in
children > 2 years of age
Allergic and ● Acute pruritic eruption ● Avoiding contact with allergens
irritant contact ● The acute phase ● Vesicles and erythemacold
dermatitis erythema, edema, compresses, topical corticosteroids.
and vesiclescoalesce ● ItchingHydroxyzine and
to form bullae. Vesicles diphenhydramine
papules, crusting ● If dermatitis is severe or
and scaling. widespread, involves mucous
As the papulovesicular membranes, or is unresponsive to
lesions resolve, the initial therapysystemic
chronic stage involving corticosteroids
lichenification and
scaling occurs.
● Patch testing gold
standard for diagnosis of
ACD
Seborrheic ● Scalp, face, and upper ● Avoid emotional stress
dermatitis trunk, under mustaches ● KeratolyticsSalicylic acid, zinc
and beards, eyebrows, pyrithione, and tar shampoos scalp
and nasolabial folds and face
● In infants, seborrhea ● Anti-inflammatory topical agents
commonly affects the and shampoos (fluocinolone
scalp “cradle cap.” shampoo and cream).
● Adultsitchy red ●Topical steroid once or twice a day
rash± history of with a shampoo.
“dandruff” ● Topical calcineurin inhibitors
● In infantsthick ● Ketoconazole and selenium sulfide
scales and erythema of shampoos
the vertex of the scalp ● Seborrheic blepharitis cleansing

164
iSummary SHORT NOTES

of the eyelashes with baby shampoo


and cotton-tipped applicators
Discoid ● Papules, scaling, ● High-potency topical steroids
(Nummular) crusting, and serous ● Systemic corticosteroids for
Eczema oozing. ● Singular or more severe cases.
multiple, circular in
shape.
● Over the trunk and
lower extremities (the
head is spared)
Urticaria ● Multiple, palpable, ● Local therapies symptomatic
circumscribed, relief cool compresses, topical
erythematous, antipruritics (doxepin cream, and 1%
blanchable, pruritic menthol in aqueous cream)
papules (wheals) and ● Nonsedating antihistamines
plaques ● Corticosteroids given as an oral
taper
● Chronic urticaria 
immunosuppressive therapy,
Androgens, corticosteroids,
cyclosporine, intravenous
immunoglobulins, and
plasmapheresis
Acne vulgaris ● Face, back, shoulders, ●Mild to moderate acne●1.
and anterior chest wall benzoyl peroxide ●add topical
● Redness, dry skin, tretinoin ●add topical erythromycin
pruritus, and nodules or topical clindamycin ● Add
systemic tetracycline
●Moderate to severe nodular
pustular acne 
●systemic antibiotic 1. tetracycline
or erythromycin OR minocycline,
doxycycline, clindamycin, and
trimethoprim-sulfamethoxazole
● Oral isotretinoin should be used
before significant scarring occurs or if
symptoms are not promptly
controlled by antibiotics
Alopecia areata ● Localized loss of hair in ● Intralesional glucocorticoid
round areas without any injection limited scalp
visible inflammation. involvement. Brow and beard areas
● The condition can may also be injected. New hair
range in severity from a growth usually is visible in about 4
few small bald patches weeks.
to total loss of scalp and ● Topical minoxidil scalp,
body hair, alopecia eyebrows, and beard area. Minoxidil
universalis solution is applied twice daily and

165
iSummary SHORT NOTES

stimulates hair growth within 12


weeks. Maximal response is seen by
1 year, and treatment must be
maintained until full remission
occurs.
● Anthralin cream safe in children
Alopecia ●Diffuse loss of scalp, ● Treat the underlying cause
telogen axillary, and pubic hair ● Reassurancerecovery is
effluvium occurring 2–4 months spontaneous and occurs within 6
after an inciting event months
(psychological stressors,
major surgery,
postpartum, crash diets,
endocrine disorders).
Androgenic ● Hereditary; between ● Topical minoxidil; finasteride
alopecia 12 and 40, with gradual
progression of hair loss
over the temple and
crown area
Atypical nevi ● Benign growths ● Excisional biopsy only if melanoma
sharing features of is suspected
melanoma.
●Typically large (> 6
mm), hyperpigmented,
and asymmetric, with
irregular “fuzzy” borders
(“fried egg”
appearance).
Melanoma ● Changing mole ● Excisional biopsy with narrow (eg,
enlarge suddenly, 2 mm) margins (shave biopsy is not
begin to bleed, begin to recommended)
itch, or become painful. ● Sentinel lymph node dissection for
May occur anywhere on melanomas > 1 mm thick to
the body, but more determine if adjuvant therapy is
common in sun-exposed needed
areas
Basal cell ● Shiny “pearly” papule ● Excision or destruction
carcinoma (bcc) with an umbilicated (electrodesiccation and curettage) of
center and the lesion
telangiectasias (small
blood vessels on the
skin)
●Sun exposed areas
(especially the central
face and ears)
● Shave biopsy
Squamous cell ● Hyperkeratotic lesion ● Surgical excision with clear margins

166
iSummary SHORT NOTES

carcinoma with crusting and


ulceration.
●May occur anywhere,
but most commonly
seen in sun-exposed
areas
Erythema ● Tender red nodules, ● Treat underlying cause
Nodosum most frequently on the ● Bed rest, leg elevation, and
pretibial area or lower NSAIDSs
legs. ● Spontaneous resolution occurs
within 4 to 6 weeks in most patients

 Until tinea capitis is cured, an imidazole or ciclopirox cream should be applied


to the scalp to prevent spread, especially to other children, and selenium sulfide
2.5% shampoo should be used daily. Children can attend school during
treatment, and the risk of transmission is low.
 Tinea versicolor the condition may be chronic but is not contagious because
the causative fungus is a normal skin flora. Tinea versicolor does not leave any
permanent color changes or scarring after treatment. Unfortunately, recurrence
is common.
 Onychomycosis because fungi are responsible for only approximately one-half
of nail dystrophies, the diagnosis of onychomycosis may need to be confirmed
by potassium hydroxide preparation, culture, or histology. Psoriasis, lichen
planus, contact dermatitis, trauma, nail-bed tumor, and yellow nail syndrome
may be mistakenly diagnosed as onychomycosis.
 HSV-1 is the most common cause of oral herpes infection, and HSV-2 is the
primary pathogen in sexually transmitted genital herpes. Both serotypes can be
present at oral or genital sites. In genital herpes recurrent episodes are less
likely with the HSV-1 serotype than with the HSV-2 serotype
 Condylomata are associated with cervical carcinoma and penile cancer. Among
the many subtypes of HPV, types 16 and 18 are especially associated with the
development of carcinoma
 Individuals with impetigo from streptococcal infections can develop
glomerulonephritis as a rare complication. Oral antibiotics may not prevent the
development of renal complications of cutaneous streptococcal infections.
 ERYSIPELAS Routine complete blood count with differential is indicated,
because neutrophilic leukocytosis is common. Blood, wound, and
nasopharyngeal cultures are recommended.
 Guttate psoriasis generally follows a streptococcal infection (e.g., strep
throat) and presents with an acute symmetrical eruption of “droplike” lesions on
the trunk and limbs
 ACNE ROSACEA After initial treatment, follow-up is required in 6 to 8 weeks
to determine treatment efficacy.
 Allergic and irritant contact dermatitis Patch testing is indicated when
inflammation persists despite avoiding the offending agent.

167
iSummary SHORT NOTES

 Urticaria Patients should be advised to avoid aspirin, and other NSAIDs agents
and opioid narcotics. Angioedema patients may require intubation. Epinephrine,
first-generation antihistamines, and corticosteroids are used. Fresh frozen
plasma has been effective in ACEI-induced angioedema. Complement deficiency
associated angioedema is treated with androgens

 Acne vulgaris

 Classifications Mild (noninflammatory): primarily comedones; moderate


(inflammatory): primarily papules and pustules; severe (nodulo-cystic):
primarily nodules and cysts. Most patients will have a mixture of both
inflammatory and non-inflammatory lesions
 Bacterial colonization of the trapped sebum with Propionibacterium
acnes
 Allow 6 to 8 weeks for treatment to work before deciding to try another
regimen or add another agent.
 When prescribing topical tretinoin remember to tell your patient to
expect redness and irritation of the face, especially in the initial period. Urge
the patient not to discontinue the product because of this side effect.
 Isotretinoin teratogenic and must not be used in pregnancy. Women
should avoid becoming pregnant for 1 month after discontinuing treatment.
As both tetracycline and isotretinoin can lead to pseudotumor cerebri, they
should not be used together. vitamin A supplements should be avoided as
the combination may lead to vitamin A toxicity
 Retinoid products of any kind is photosensitizing, and sunscreen is
mandatory with the concurrent use of these products.
 Follow upExpect 4 to 8 weeks to see significant improvement with most
treatments. Recommend follow-up about every 6 weeks until the optimal
regimen has been determined, then every 2 to 3 months for maintenance
and adjustments.

 Alopecia areata

 Intralesional glucocorticoid injection New hair growth usually is visible in


about 4 weeks. Localized skin atrophy may occur if the injections are
administered too deeply into the dermis or into the fat.
 Topical minoxidil Minoxidil solution is applied twice daily and stimulates
hair growth within 12 weeks. Maximal response is seen by 1 year, and
treatment must be maintained until full remission occurs.
 Anthralin cream New hair growth may occur within 2 to 3 months after
initiation of topical anthralin therapy.

 The incidence of melanoma is ↑ in patients with atypical nevi.


 MelanomaTumor thickness (Breslow’s classification) and lymph node spread
are the most important prognostic factors. The best prognosis is achieved with a

168
iSummary SHORT NOTES

Breslow depth ≤0.75 mm. Ulceration is associated with more aggressive cancers
and a poorer prognosis. Color does not correlate with prognosis

169
iSummary SHORT NOTES

MALE REPRODUCTIVE
SYSTEM DISORDERS
Disease Diagnosis Investigation Management
Epididymitis ● Acute painful, swollen ● NSAIDs, scrotal
scrotum elevation, and cold
● Dysuria and urinary packs
frequency ● AB cover N.
● Swollen, tender mass gonorrhoeae and C.
attached to the testicle trachomatis if sexual
transmission is
suspected; TMP-SMX
or a fluoroquinolone if
enteric gram negative
organisms or staph are
suspected
Prostatitis ● Perineal, rectal, ● UA ● Acutely
and/or low back pain bacteriuria, illhospitalization
● Urinary urgency, hematuria, and IV antibiotics.
frequency, retention, pyuria Ampicillin and an
nocturia, and dysuria ● Urine culture aminoglycoside
● Painful ejaculation and sensitivity ● Oral TMP-SMX,
● Prostate feels boggy, identifies the amoxicillin, or a
swollen, warm, and causal organism. fluoroquinolone.(for
tender 21:30 days)
● analgesics and stool
softeners for comfort
● If chronic symptoms
persist and cultures
are negative
symptomatic
(analgesics, anti-
inflammatory agents,
α-blockers, and sitz
baths)
Benign ● Obstructive flow ● Prostate- ● α-blockers
prostatic symptomshesitancy specific antigen doxazosin,
hypertrophy and slow, weak stream, (PSA) may be ↑ terazosin, prazosin,
which in turn can lead but is tamsulosin and 5α-
to irritative nonspecific reductase inhibitors
symptomsfrequency,  finasteride,
urgency, and nocturia dutasteride
● Smooth and
symmetrically enlarged ● Transurethral
on DRE resection of the

171
iSummary SHORT NOTES

prostate (TURP) and


other minimally
invasive surgical
techniques

●Open prostatectomy
is the treatment of last
resort.

Prostate ● Asymptomatic, or ● If PSA or DRE ● Depends on the


cancer may → urinary raises concern stage at diagnosis and
obstruction with ultrasound- on the patient’s
symptoms of prostatism guided biopsy comorbidities and may
● Bony metastases can includesurgery,
→ vertebral or hip pain radiation, hormonal
● DRE  areas of therapy, or watchful
induration or nodules if waiting
the tumor is on the
posterior aspect of the
prostate. Anterior
tumors will produce a
normal exam.

Orchitis ● Painful, swollen ● Ultrasound ● Analgesics, scrotal


testicle support, and cold
● If the infection is packs
caused by ● If symptoms do not
mumpsparotitis, improveconsider
fever, and malaise. abscess or other
causes of testicular
mass

 The most common cause of acute prostatitis is E. coli


 Chronic prostatitis is the most common cause of recurrent UTI in men.
 Hydrocele and spermatocele usually do not require treatment. If they become
large or uncomfortable, surgical repair is offered
 Varicocele repair is considered part of infertility treatment. If the mass was
detected incidentally, repair may be advised to prevent infertility.

171
iSummary SHORT NOTES

ORTHOPEDIC
 Fractures
Type Diagnosis Complication Management
Skull ● Cerebrospinal ● Intracranial ●Scalp lacerationscontrol
otorrhea or injury, infections, bleeding
rhinorrhea, and seizures ● Open fractures cleaned and
periorbital repaired, AB
ecchymosis
(raccoon eyes)
● Skull
radiographs if –
ve do CT scan
Nasal ● If there is good ● Septal ● Drain septal hematoma acutely
fractures cosmesis, a hematomas and the pack the nose
radiograph is septal cartilage can ● Consultation in 5 to 7 days
unnecessary just necrose, leading to when edema has resolved
to document a a perforated
fracture septum
Orbital ● A Waters view ● Check Visual acuity
fractures of the orbit or a ●emergency ophthalmic
CT evaluation consultation
Neck ● Lateral cervical ●maxillofacial surgeon should be
fracture radiograph consulted
THORACOL ●Plain ●Hospital admission
UMBAR radiographs - ●CT
Rib ● Flail chest ●First or second ●Oxygen, ventilatory support IF
fractures ●CXR ribs injury to the pulmonary contusion
great vessels, ●Uncomplicated fractures
cervical spine, conservatively
head, and brachial
plexus
Sternal ●Chest ●Myocardial ●(ECG) should be obtained
fractures radiographs with contusion serially
lateral views of
the sternum
Clavicle ●Clinical ●Neurovascular ●Nondisplaced  sling
injury displacement figure-of-8 brace
●Fracture medial ●Consultation severely
third  displaced fractures, complicated
intrathoracic fractures, or fractures of the
injuries or late- medial or distal third
onset arthritis
fractures of distal
third  disrupt the
coracoclavicular
ligament and
nonunion may
occur without

172
iSummary SHORT NOTES

operative repair
Scapula ●Coexisting injuries ●Immobilization with a sling, ice,
to the ipsilateral and analgesics, with ROM
lung, thoracic cage, exercises
and shoulder girdle ●Surgical management
Proximal ●Fall on an ●Examine ●ICA & Referral - Early mobility
humerus outstretched sensation over the
hand skin of the deltoid
that is supplied by
the axillary nerve
Humeral ●Neurovascular ●ICA & Referral closed
shaft injuries are a fractures
common ●Surgery pathologic fractures,
associated with neurovascular
injuries, or very proximal or very
distal humerus fractures
Intercondyl ●Severe soft-tissue ●Joint reduction, elbow
ar injuries immobilized and urgent
fractures orthopaedic referral made

Supracond ●Radiographs ●Neurovascular ●Posterior splint for 2 weeks


ylar may reveal a fat- injuries are Suspected occult or nondisplaced
fractures pad sign common radial head fractures
posteriorly or (radial,median, Undisplaced fractures casting
anteriorly anterior displaced fractures  surgical
interosseus fixation
nerve,ulnar)
●Volkmann
ischemic
contracture
Olecranon ●Involve ulnar ●Undisplaced fractures (<2 mm
fractures nerve injury or displacement) are immobilized
radial head and with the elbow in 45 degrees of
neck fractures flexion with orthopaedic follow-up
within a week.
●Displaced fractures surgically
Forearm ●The ●Compartment ●Fractures of both the radius and
fractures radiographic syndrome ulna surgical fixation
evaluation should ●A long arm cast minimally
include views of displaced or undisplaced fractures
the elbow and ●Isolated undisplaced ulnar or
wrist in addition radial fractures immobilized in
to a long-arm cast with close follow-
anteroposterior up;
(AP) and lateral ●Displaced fractures open
views of the reduction and internal fixation.
forearm
Colles ●Distal radial ●Median nerve and ●Fractures should be reduced
fracture metaphysis radial artery Immobilization should involve a
fracture injuries long arm cast

173
iSummary SHORT NOTES

falls on the ●Unstable fractures surgical


outstretched intervention
hand
Scaphoid ●Fall on a ●Avascular ●Suspected occult fractures
fracture hyperextended necrosis and immobilized in a short-arm
wrist nonunion thumb spica splint or cast and re-
●The evaluated in 2 weeks.
radiographic ●Displaced or unstable fractures
evaluation should placed in a long-arm thumb
include AP, spica splint or cast
lateral, oblique,
and scaphoid
views
●MRI or bone
scan should be
considered.
Pelvic ●A standard AP ●Hemorrhage ●Single pubic or ischial ramus
fractures view of the pelvis fracture OR Iliac wing fractures
should be conservative
obtained in all ●Coccyx fractures bed rest, sitz
victims of serious baths, laxatives, and cushions for
trauma. sitting
●Acetabular fractures
reduction of the dislocation,
hospital admission, and early
orthopaedic consultation
Hip ●Radiographs - ●Intracapsular ●Orthopaedic consultation
fractures ●MRI avascular necrosis emergently intracapsular
thromboembolic fractures, urgently 
events and failure extracapsular fractures
to regain normal
function
Distal and ●Hemorrhage ●Mid-femur: orthopaedic
mid-femur Distal femoral consultation urgently
fractures injury to peroneal ●Distal femoral analgesics,
nerve (innervates immobilization, and emergent
space between the referral
first and second
toe) and the
popliteal artery
Tibia and ●Complications ●Nondisplaced, single plateau
fibula chronic pain, fractures immobilized and
knee dysfunction, outpatient orthopaedic referral
and degenerative made
arthritis ●Other plateau fractures
referred quickly
●Orthopaedic consultation
Patella ●AP, lateral, and ●Traumatic ●A nondisplaced patellar fracture
skyline views chondromalacia with intact extensor mechanism
and avascular knee immobilizer, ice,
necrosis elevation, and analgesics

174
iSummary SHORT NOTES

followed by long leg casting.


other types of patellar
fracturesconsultation
Ankle ●AP, lateral, and ●Traumatic ●Disruption of the ankle at only
mortise views arthritis, chronic one place conservative means
talar instability, (posterior splinting, nonweight
chronic pain and bearing, and edema control)
swelling, and ●If two or more disruptions of the
osteochondral ankle immobilization and
fractures emergent referral
●Unilateral avulsion fractures of
the distal tip of the malleolus
treated like second-degree sprains
Metatarsal ●Nondisplaced shaft fractures
s and conservatively with a walking cast
phalanges or orthopaedic shoe
●Displaced shaft fractures
closed reduction, casting, and
nonweight bearing for 6 weeks
neck fractures open fixation for
displaced fractures
Fifth ●Jones fracture ●Jones fracture6 weeks of
metatarsal malunion or casting and nonweight bearing
fractures nonunion avulsion fracture cast shoe
Ankle ●Grade I and II sprains external support, such as an air or gel splint, ice
sprain application, and elevation; early mobilization
●Grade III sprain orthopedist for evaluation
ICA: immobilization, ice, anagelsics

 Knee injuries
Disease Diagnosis Treatment
Meniscal tears ●McMurray’s sign ●Arthroscopic partial
●MRI, Radiographs to meniscectomy
rule out bony conditions degenerative tears or in the
absence of mechanical
symptoms conservative
measures (e.g., rest, ice, NSAIDs,
corticosteroid injection) can initially
be tried.
Anterior cruciate ●Hearing a pop - ●Rehabilitation alone (old patient)
ligament (ACL) ●Lachman test ●Reconstruction (young patient\0
tear ●Radiographs
●MRI confirmation ,
evaluation of associated
injuries
Collateral ●Direct blow to the ●Hinged knee brace and ROM
ligament (MCL lateral aspect of the ●Exercises for isolated MCL or LCL
and LCL) injuries knee injuries

175
iSummary SHORT NOTES

●LCL injuries evaluate


for accompanying
injuries
Patellofemoral ●Aching anterior knee ●Activity modification and an
pain syndrome pain that worsens with exercise program, NSAIDs
(runner's knee) activities
Iliotibial band ●Lateral knee pain - ●ITB stretching, foam roller
syndrome ●Ober’s test exercises for soft tissue work to
relax the ITB, rest, ice, NSAIDs,
iontophoresis
Quadriceps and ●Radiographs patella ●Immediate surgical repair
patellar tendon alta (patellar tendon cylinder cast in full extension for 4–
ruptures rupture) or patella baja 6 weeks Partial tears
(quadriceps rupture)
●MRI to confirm the
diagnosis, surgical
planning
Knee dislocations ●Highly associated with
injury to the popliteal
artery
Osteochondritis ●Gradual onset of vague ●Younger patients with smaller,
dissecans knee pain and stable lesions conservative
intermittent swelling measures.
after running and sports ●Activity modification (no running
activities or jumping; possible crutch use)
●Plain radiographs minimizes shear forces and allows
(especially the tunnel for new bone formation.
view). ●If the overlying articular cartilage
●MRI is disrupted, a loose body is
present, or the patient is skeletally
mature, operative management

 Overuse injuries
Disease Diagnosis Treatment
Rotator cuff ●Pain with shoulder ●Nonoperative ice, a limited course
tendinopathy abduction of NSAIDs, and avoiding aggravating
(Hawkin or Neer signs) factors. A subacromial corticosteroid
●Radiograph, MRI injection
●Operative referral for surgery

Epicondylitis ●Clinical ●Initial treatment ice with friction


massage, relative rest and a short
course of NSAIDs.

176
iSummary SHORT NOTES

●Corticosteroid injection at the site of


maximal tenderness may control
symptoms.
Rehabilitation - Surgeryrarely
required
De quervain ●Pain reproduced by ●Initial treatment ice, relative rest,
tenosynovitis sharp ulnar deviation NSAIDs, and a thumb spica splint.
of the hand with the Iontophoresis may also be useful as an
thumb either extended early modality.
or flexed (Finkelstein ●If symptoms persistlocal
maneuver). corticosteroid
●Operative When nonoperative
treatments fail to alleviate symptoms
Trigger finger ●pain at the PIP joint, ●Tendon sheath injection with a
the problem is located corticosteroid
at the palmar surface ●Persistent or recurrent symptomsa
of the MCP joint second injection with application of a
inflammation at the A1 trigger-finger splint is helpful.
pulley surgical release of the pulley

Anserine ●There is tenderness ●corticosteroid injection into the bursa


bursitis at the anserine bursa,
located on the
proximal medial aspect
of the tibia
Prepatellar ●Chronic inflammation ●Protective padding in recurrent cases.
bursitis of the prepatellar ●When infection is
“housemaid’s bursa results from suspectedantibiotics after cultures
knee” recurrent trauma, such are obtained.
as repetitive kneeling. ●Chronic inflammation may respond to
a corticosteroid injection
surgical excision of the bursa
Shin splints ●Diffuse pain and ●Initial treatment relative rest, ice
tenderness at the massages, and NSAIDs.
posteromedial aspect
of the tibia
less commonly, the
anterolateral aspect of
the tibia
●Radiographs rule out
tibial or fibular stress
fractures
Chronic ●Pain with activity, ●Initial treatment rest and
compartment worsening with analgesics. With persistent symptoms,
syndrome increased activity and compartment pressure testing will be
relieved by rest. necessary.

177
iSummary SHORT NOTES

●Anterolateral leg and ●Surgical fasciotomy


dorsum of the foot,
posteromedial and
instep pain
Plantar ●Pain is typically most ●Initial measures relative rest, A heel
fasciitis intense upon arising in cup or silicon pad, Ice or heat, NSAIDs,
the morning, with appropriate shoe support including
symptom shoe inserts, and calf stretching.
improvement with ●As a second step, custom-made night
activity splints, physical therapy modalities, and
steroid injection may be helpful.
●Extracorporeal shock therapy, Surgery

Achilles ●Pain and swelling in ●Ice, NSAIDs, relative rest, and gentle
tendonitis the Achilles tendon  stretching of the calf
with activity ●Early referral for physical therapy
●Dorsiflexion of the
foot or local palpation
reproduces the
symptoms

Disease Diagnosis Investigatio Management


n
Adhesive ●Loss of both ●NSAIDs, physiotherapy, and
Capsulitis active and local steroid injection
(Frozen passive ●Restricted movements are
Shoulder) motion in all slow to returnorthopaedic
directions of referral
the shoulder
OSTEOPOROS ●Asymptomat ●Central DXA ●Ca 1.5g –Vit D 400:800 IU
IS ic until testing: T-score: ●Bisphosphonates
fractures more than 2.5 sd and
occur below mean (T-
●Vertebral score < −2.5) ●Raloxifene (SERMs)
compression ●Calcitonin, Teriparatide
fractures treatment only
(most
common)
Carpal ●History ●Phalen's test ●Conservative therapy
Tunnel ●Tinnel's test (modify activities, NSAIDs,
Syndrome ●Electrophysiolo rest, vitamin B6, nocturnal
gic studies wrist splinting)
●Local corticosteroid
injections
●Surgery

178
iSummary SHORT NOTES

Nerve injury
MEDIAN NERVE Inability to oppose the thumb - and loss of sensation over the
DAMAGE lateral side of the hand
Radial nerve Wrist drop
Ulnar nerve Claw hand deformity
Axillary nerve Paralysis of arm abduction
Long thoracic winging of the scapula
nerve
Femoral nerve No patellar reflex

 Shoulder dislocations injury to the axillary nerve


 Elbow dislocations injury to the median nerve and brachial artery

 Osteoporosis
 ACR recommends repeat DXA testing every 6 to 12 months in patients on
chronic corticosteroids
 DEXA scanning can be used in observation of those at risk or to monitor
progress in treatment. A person must change more than 5% to be sure a
change is not merely machine imprecision; therefore, testing is usually done
at intervals of 2 or more years.
 If a woman at age 65 has normal bone density, further monitoring is
unnecessary unless her risk profile changes.
 College of Rheumatology (ACR) recommends starting a bisphosphonate in
patients initiating chronic corticosteroid treatment (>5 mg/day for > 3
months), even prior to obtaining a DXA. ACR also recommends initiating
bisphosphonate therapy in patients already on chronic corticosteroid
therapy if their DXA T score is -1.

 Calcium supplementation for osteoporosis prevention

 Calcium is not sufficient to prevent OP, but is an important adjuvant.


Recommended daily intake is:
 500 mg for children aged 1 to 3;
 800 to 1,300 mg for ages 4 to 18;
 1,000 mg for ages 19 to 50; and
 1,200 or 1500 mg thereafter.
 1500 mg for patients with established osteoporosis.

 The diagnostic study of choice in monoarthritis is synovial fluid analysis


(Arthrocentesis).

179
iSummary SHORT NOTES

 Gout
 pseudogout, for calcium pyrophosphate deposition disease Rhomboid-
shaped, positive birefringement
 Gout Needle-shaped, negative birefringement
 NSAIDs drugs of choice for acute gouty attacks. Corticosteroids are
normally used in cases when NSAIDs or colchicine cannot be tolerated or
are ineffective.
 Colchicine terminates most acute gout attacks within 6 to 12 hours;
however, it is limited by its GI side effects and is often poorly tolerated by
elderly people.
 Allopurinol is indicated for prophylaxis of gout when hyperuricemia is
documented, but its use in the acute setting is inappropriate.
 For patients with recurrent gouty attacks, renal stones, renal damage, or
asymptomatic uric acid levels greater than 12 mg per dL or those who are
undergoing cancer chemotherapy or taking cyclosporine after
transplantation, uric acid-lowering therapy should be initiated.

 Differential diagnosis of Osteoarthritis


Osteoarthritis Gout Rheumatoid
Arthritis
Key Pauciarticular. Pain with Monoarticular. Polyarticular.
presenting movement, improving Abrupt onset. Pain Gradual, symmetric
symptoms with rest. Site of old at rest and involvement.
injury (sport, trauma). movement. Morning stiffness.
Obesity. Occupation. Precipitating event Hands and feet
(meal, physical initially involved
stress). Family more than large
history. joints. Fatigue,
poorly restorative
sleep.
Key Infrequent warmth, Podagra. Swelling, Symmetric swelling,
physical effusion. Crepitus. warmth. Exquisite tenderness. MCP,
findings Enlargement/spur pain with MTP, wrist, ankle
formation. movement. Single usually before larger,
Malalignment. joint (exceptions— proximal joints.
plantar fascia, Rheumatoid nodules.
lumbar spine).
Tophi.
Key Few characteristic Synovial fluid with Elevated ESR/CRP.
laboratory, (early). Loss of joint uric acid crystals. Rheumatoid factor.
x-ray space, spur formation, Elevated serum Anemia of chronic
findings malalignment (late). uric acid. 24 h disease. Early
urine uric acid. erosions on x-ray,
osteopenia at
involved joints.

181
iSummary SHORT NOTES

MISCELLANEOUS

181
iSummary SHORT NOTES

 Treatments for specific Toxins


Toxin Treatment
Acetaminophen N-acetylcysteine
Amphetamines Benzodiazepines
Beta blockers Glucagon, vasopressors
Calcium blockers Calcium, vasopressors
Digoxin Digoxin-specific antibody fragments
Cyclic antidepressants Alkalinization up to pH 7.5
Methanol Ethanol or fomepizole
Lithium IV fluids, oral polyethylene glycol solution,
hemodialysis.
Opioids Naloxone
Organophosphates Atropine
Salicylates Activated charcoal, urine Alkalinization

 Opiates (Morphine, heroin, codeine, oxycodone, etc)  Pinpoint pupils,


hypotension, hypopnea, coma, hypothermia
 Sympathomimetic (Cocaine, ecstasy, methamphetamine) Tachycardia,
hypertension, elevated temperature, dilated pupils (mydriasis)

182
iSummary SHORT NOTES

183
iSummary SHORT NOTES

184

You might also like